Harrisons Principles of Internal Medicine Self-Assessment and Board Review 18th Edition [18 ed.] 0071771956, 9780071771955

Add the unmatched authority of Harrison’s to your board review or test preparation 1,100 review questions! Based on the

245 89 12MB

English Pages 512 [521] Year 2012

Report DMCA / Copyright

DOWNLOAD PDF FILE

Recommend Papers

Harrisons Principles of Internal Medicine Self-Assessment and Board Review 18th Edition [18 ed.]
 0071771956, 9780071771955

  • 0 0 0
  • Like this paper and download? You can publish your own PDF file online for free in a few minutes! Sign Up
File loading please wait...
Citation preview

18th Edition

Principles of

HARRISON’S

®

INTERNAL MEDICINE SELF-ASSESSMENT AND BOARD REVIEW

Editorial Board

DAN L. LONGO, md Professor of Medicine, Harvard Medical School Senior Physician, Brigham and Women’s Hospital Deputy Editor, New England Journal of Medicine Boston, Massachusetts ANTHONY S. FAUCI, md Chief, Laboratory of Immunoregulation Director, National Institute of Allergy and Infectious Diseases National Institutes of Health Bethesda, Maryland DENNIS L. KASPER, md William Ellery Channing Professor of Medicine Professor of Microbiology and Molecular Genetics Harvard Medical School Director, Channing Laboratory Department of Medicine, Brigham and Women’s Hospital Boston, Massachusetts STEPHEN L. HAUSER, md Robert A. Fishman Distinguished Professor and Chairman Department of Neurology, University of California San Francisco, California J. LARRY JAMESON,

md, phd Robert G. Dunlop Professor of Medicine Dean, University of Pennsylvania School of Medicine Executive Vice-President of the University of Pennsylvania for the Health System Philadelphia, Pennsylvania

JOSEPH LOSCALZO, md, phd Hersey Professor of the Theory and Practice of Medicine Harvard Medical School Chairman, Department of Medicine Physician-in-Chief, Brigham and Women’s Hospital Boston, Massachusetts

18th Edition

HARRISON’S INTERNAL MEDICINE Principles of

®

SELF-ASSESSMENT AND BOARD REVIEW

For use with the 18th edition of HARRISON’S PRINCIPLES OF INTERNAL MEDICINE

EDITED BY CHARLES M. WIENER, MD Dean/CEO Perdana University Graduate School of Medicine Selangor, Malaysia Professor of Medicine and Physiology Johns Hopkins University School of Medicine Baltimore, Maryland

CYNTHIA D. BROWN, MD Assistant Professor of Medicine Division of Pulmonary and Critical Care Medicine University of Virginia Charlottesville, Virginia

ANNA R. HEMNES, MD Assistant Professor, Division of Allergy, Pulmonary,   and Critical Care Medicine Vanderbilt University Medical Center Nashville, Tennessee

New York   Chicago   San Francisco   Lisbon   London   Madrid   Mexico City Milan   New Delhi   San Juan   Seoul   Singapore   Sydney   Toronto

Copyright © 2012, 2008, 2005, 2001, 1998, 1994, 1991, 1987 by The McGraw-Hill Companies, Inc. All rights reserved. Except as permitted under the United States Copyright Act of 1976, no part of this publication may be reproduced or distributed in any form or by any means, or stored in a database or retrieval system, without the prior written permission of the publisher. ISBN: 978-0-07-177196-2 MHID: 0-07-177196-4 The material in this eBook also appears in the print version of this title: ISBN: 978-0-07-177195-5, MHID: 0-07-177195-6. All trademarks are trademarks of their respective owners. Rather than put a trademark symbol after every occurrence of a trademarked name, we use names in an editorial fashion only, and to the benefit of the trademark owner, with no intention of infringement of the trademark. Where such designations appear in this book, they have been printed with initial caps. McGraw-Hill eBooks are available at special quantity discounts to use as premiums and sales promotions, or for use in corporate training programs. To contact a representative please e-mail us at [email protected].

International Edition ISBN 978-0-07-178847-2; MHID 0-07-178847-6. Copyright © 2012. Exclusive rights by The McGraw-Hill Companies, Inc., for manufacture and export. This book cannot be re-exported from the country to which it is consigned by McGraw-Hill. The International Edition is not available in North America. Notice Medicine is an ever-changing science. As new research and clinical experience broaden our knowledge, changes in treatment and drug therapy are required. The authors and the publisher of this work have checked with sources believed to be reliable in their efforts to provide information that is complete and generally in accord with the standards accepted at the time of publication. However, in view of the possibility of human error or changes in medical sciences, neither the authors nor the publisher nor any other party who has been involved in the preparation or publication of this work warrants that the information contained herein is in every respect accurate or complete, and they disclaim all responsibility for any errors or omissions or for the results obtained from use of the information contained in this work. Readers are encouraged to confirm the information contained herein with other sources. For example and in particular, readers are advised to check the product information sheet included in the package of each drug they plan to administer to be certain that the information contained in this work is accurate and that changes have not been made in the recommended dose or in the contraindications for administration. This recommendation is of particular importance in connection with new or infrequently used drugs.

TERMS OF USE This is a copyrighted work and The McGraw-Hill Companies, Inc. (“McGraw-Hill”) and its licensors reserve all rights in and to the work. Use of this work is subject to these terms. Except as permitted under the Copyright Act of 1976 and the right to store and retrieve one copy of the work, you may not decompile, disassemble, reverse engineer, reproduce, modify, create derivative works based upon, transmit, distribute, disseminate, sell, publish or sublicense the work or any part of it without McGraw-Hill’s prior consent. You may use the work for your own noncommercial and personal use; any other use of the work is strictly prohibited. Your right to use the work may be terminated if you fail to comply with these terms. THE WORK IS PROVIDED “AS IS.” McGRAW-HILL AND ITS LICENSORS MAKE NO GUARANTEES OR WARRANTIES AS TO THE ACCURACY, ADEQUACY OR COMPLETENESS OF OR RESULTS TO BE OBTAINED FROM USING THE WORK, INCLUDING ANY INFORMATION THAT CAN BE ACCESSED THROUGH THE WORK VIA HYPERLINK OR OTHERWISE, AND EXPRESSLY DISCLAIM ANY WARRANTY, EXPRESS OR IMPLIED, INCLUDING BUT NOT LIMITED TO IMPLIED WARRANTIES OF MERCHANTABILITY OR FITNESS FOR A PARTICULAR PURPOSE. McGraw-Hill and its licensors do not warrant or guarantee that the functions contained in the work will meet your requirements or that its operation will be uninterrupted or error free. Neither McGraw-Hill nor its licensors shall be liable to you or anyone else for any inaccuracy, error or omission, regardless of cause, in the work or for any damages resulting therefrom. McGraw-Hill has no responsibility for the content of any information accessed through the work. Under no circumstances shall McGraw-Hill and/or its licensors be liable for any indirect, incidental, special, punitive, consequential or similar damages that result from the use of or inability to use the work, even if any of them has been advised of the possibility of such damages. This limitation of liability shall apply to any claim or cause whatsoever whether such claim or cause arises in contract, tort or otherwise.

CONTENTS Preface

Introduction to Clinical Medicine

SECTION II

Nutrition

SECTION III

Oncology and Hematology

SECTION IV

Infectious Diseases

SECTION V

Disorders of the Cardiovascular System

SECTION VI

Disorders of the Respiratory System

SECTION VII

Disorders of the Kidney and Urinary Tract

SECTION VIII

Disorders of the Gastrointestinal System

SECTION IX

Rheumatology and Immunology

SECTION X

Endocrinology and Metabolism

SECTION XI

Neurologic Disorders

SECTION XII

Dermatology



Questions Answers Questions Answers Questions Answers

Questions Answers Questions Answers Questions Answers Questions Answers Questions Answers Questions Answers Questions Answers Questions Answers Questions Answers

Color Atlas

1 25 83 87 97 114 153 193

CONTENTS

SECTION I

vi

265 280 299 310 331 336 343 355 377 388 409 424 453 466 497 501 507 v

SECTION I

PREFACE This is the third edition of Harrison’s Self-Assessment and Board Review that we have had the honor of working on. We thank the editors of the 18th edition of Harrison’s Principles of Internal Medicine for their continued confidence in our ability to produce a worthwhile companion to their exceptional textbook. It is truly inspirational to remind ourselves why we love medicine broadly, and internal medicine specifically. The care of patients is a privilege. As physicians, we owe it to our patients to be intelligent, contemporary, and curious. Continuing education takes many forms; many of us enjoy the intellectual stimulation and active learning challenge of the question-answer format. It is in that spirit that we offer the 18th edition of the Self-Assessment and Board Review to students, housestaff, and practitioners. We hope that from it you will learn, read, investigate, and question. The questions and answers are particularly conducive to collaboration and discussion with colleagues. This edition

contains over 1100 questions that, whenever possible, utilize realistic patient scenarios including radiographic or pathologic images. Similarly, our answers attempt to explain the correct or best choice, often supported with figures from the 18th edition of Harrison’s Principles of Internal Medicine to stimulate learning. All of the authors have physically left the Osler Medical Service at Johns Hopkins Hospital. However, our experiences with colleagues and patients at Hopkins have defined our professional lives. In the words of William Osler, “We are here to add what we can to life, not to get what we can from life.” We hope this addition to your life stimulates your mind, challenges your thinking, and translates to your patients. Of course, none of this would be possible without the loving support of our families, for which we are truly thankful. They were patient and encouraging as we transformed (often not quietly) a mountain of page proofs into this book.

PREFACE

SECTION I Introduction to Clinical Medicine

QUESTIONS DIRECTIONS: Choose the one best response to each question.

I-1.  Which of the following is the best definition of evidencebased medicine? A. A summary of existing data from existing clinical trials with a critical methodological review and statistical analysis of summative data B. A type of research that compares the results of one approach to treating a disease with another approach to treating the same disease C. Clinical decision making support tools developed by professional organizations that include expert opinions and data from clinical trials D. Clinical decision making supported by data, preferably from randomized controlled clinical trials E. One physician’s clinical experience in caring for multiple patients with a specific disorder over many years I-2.  All of the following are part of the informed consent process EXCEPT: A. Alternatives and likely consequences of the alternatives to the procedure B. Ascertainment of understanding by the patient C. Discussion of the details of the procedure D. Outlining the patient’s wishes if he or she becomes unable to make decisions E. Risks and benefits of the procedure

I-4.  In high-income countries, what category of disease accounts for the greatest percentage of disability-adjusted life years lost? A. B. C. D. E.

Alcohol abuse Chronic obstructive pulmonary disease Diabetes mellitus Ischemic heart disease Unipolar depressive disorders

I-5.  What is the leading cause of death in low-income countries? A. B. D. D. E.

Diarrheal diseases Human immunodeficiency virus Ischemic heart disease Lower respiratory disease Malaria

I-6.  You are working with the public health minister of Malawi in a project to decrease malarial deaths in children younger than 5 years of age. All of the following strategies are part of the World Health Organization Roll Back Malaria plan EXCEPT: A. B. C. D. E.

Artemisinin-based combination therapy Early treatment with chloroquine alone Indoor residual spraying Insecticide-treated bed nets Intermittent preventive treatment during pregnancy

I-3.  Which of the following is the standard measure for determining the impact of a health condition on a population? A. B. C. D. E.

Disability-adjusted life years Infant mortality Life expectancy Standardized mortality ratio Years of life lost 1

SECTION I

I-7.  A 38-year-old woman is evaluated for chest pain. She has no risk factors for coronary artery disease, but a stress test is ordered by a physician in the emergency department. You are called for a cardiology consult when an exercise ECG stress test result is positive. You estimate that the pretest probability of coronary artery disease is 10% and determine that this is most likely a false-positive stress test with a low posttest probability of coronary artery disease. This is an example of which of the following principles used in medical decision making?

Introduction to Clinical Medicine

A. B. C. D. E.

Bayes’ theorem High positive predictive value High specificity Low negative predictive value Low sensitivity

I-8.  A new diagnostic test for predicting latent tuberculosis is introduced into clinical practice. In clinical trials, it was determined to have a sensitivity of 90% and a specificity of 80%. A specific clinical population of 1000 individuals has a prevalence of tuberculosis of 10%. How many individuals with latent tuberculosis would be correctly identified in this population? A. B. C. D. E.

10 80 90 100 180

I-9.  In the above scenario, how many individuals would be erroneously told they have latent tuberculosis? A. B. C. D. E.

10 90 180 720 900

I-10.  A receiver operating characteristic (ROC) curve is constructed for a new test for disease X. All of the following statements regarding the ROC curve are true EXCEPT: A. One criticism of the ROC curve is that it is developed for testing only one test or clinical parameter with exclusion of other potentially relevant data. B. The ROC curve allows the selection of a threshold value for a test that yields the best sensitivity with the fewest false-positive test results. C. The axes of the ROC curve are sensitivity versus 1 - specificity. D. The ideal ROC curve would have a value of 0.5. E. The value of the ROC curve is calculated as the area under the curve generated from the true-positive rate versus the false-positive rate. I-11.  Which of the following values is affected by the disease prevalence in a population? A. B. C. D. E. 2

Number needed to treat Positive likelihood ratio Positive predictive value Sensitivity Specificity

I-12.  Drug X is investigated in a meta-analysis for its effect on mortality after a myocardial infarction. It is found that mortality drops from 10 to 2% when this drug is administered. What is the absolute risk reduction conferred by drug X? A. B. C. D. E.

2% 8% 20% 200% None of the above

I-13.  How many patients will have to be treated with drug X to prevent one death? A. B. C. D. E.

2 8 12.5 50 93

I-14.  When considering a potential screening test, what endpoints should be considered to assess the potential gain from a proposed intervention? A. Absolute and relative impact of screening on the disease outcome B. Cost per life year saved C. Increase in the average life expectancy for the entire population D. Number of subjects screened to alter the outcome in one individual E. All of the above I-15.  A 55-year-old man who smokes cigarettes is enrolled in a lung cancer screening trial based on performance of yearly CT scans over a period of 5 years. At year 2, he is found to have a 2-cm right lower lobe lung nodule that is a non–small cell lung cancer upon surgical removal. At that time, there were no positive lymph nodes. The cancer recurs, and the patient subsequently dies from lung cancer 6 years after his initial diagnosis. A person with a similar smoking history who is not participating in the trial is discovered to have a 3-cm lung nodule that is also non–small cell lung cancer. Upon surgical resection, one lymph node is positive. This person also dies from lung cancer after a period of 3 years. What conclusion can be made about the use of the CT screening for lung cancer in these patients? A. CT screening for lung cancer improves mortality in smokers. B. It is unable to be determined if CT screening for lung cancer led to any difference in survival because one cannot determine if lag time bias is present. C. It is unable to be determined if CT screening for lung cancer led to any difference in survival because one cannot determine if lead time bias is present. D. Selection bias may cause apparent differences in survival in this trial, and one should be cautious in making conclusions with regards to CT screening for lung cancer. E. The radiation received as part of the CT scan screening led to lung cancer in the initial patient and contributed to the first patient’s overall mortality.

I-17.  Which preventative intervention leads to the largest average increase in life expectancy for a target population? A regular exercise program for a 40-year-old man Getting a 35-year-old smoker to quit smoking Mammography in women age 50–70 years Pap smears in women age 18–65 years Prostate-specific antigen (PSA) and digital rectal examination for a man older than 50 years old

A. Digitalis-specific antibody (Fab) fragments alone B. Digitalis-specific antibody fragments plus hemodialysis C. Digitalis-specific antibody fragments plus hemoperfusion D. Plasmapheresis alone E. Volume resuscitation and observation

I-18.  All of the following patients should receive a lipid screening profile EXCEPT:

I-22.  A 48-year-old woman with a generalized seizure disorder has been taking phenytoin for the past 10 years with good control of her disease. She also has a history of hepatitis C virus infection acquired via a blood transfusion received after an automobile accident in her teens. She currently takes phenytoin 100 mg tid, lactulose 30 g tid, and spironolactone 25 mg daily. She is brought to the emergency department by her husband, who reports that she has had increasing lethargy for the past week. On examination, her blood pressure is 100/60 mmHg, heart rate is 88 beats/min, respiratory rate is 20 breaths/ min, and oxygen saturation is 98% on room air. She is afebrile. She is minimally responsive to voice and follows no commands. There is no nuchal rigidity. Her abdomen is distended with a positive fluid wave but without tenderness. She has spider angiomata, caput medusa, and palmar erythema. She does not appear to have asterixis. She does have horizontal nystagmus on examination. Her laboratory values include Na, 134 meq/L; potassium, 3.9 meq/L; chloride, 104 meq/L; and bicarbonate, 20 meq/L. Creatinine is 1.0 mg/dL. The white blood cell count is 10,000/μL with a normal differential. Her liver function tests are unchanged from baseline with the exception of an albumin that is now 2.1 g/dL compared with 3 months ago when her level was 2.9 g/dL. Ammonia level is 15 μmol/L, and her phenytoin level is 17 mg/L. A paracentesis shows a white blood cell count of 100/μL that is 80% neutrophils. What test would be most likely to demonstrate the cause of the patient’s change in mental status?

A. B. C. D. E.

A. A 16-year-old boy with type 1 diabetes B. A 17-year-old female teen who recently began smoking C. A 23-year-old healthy man who is starting his first job D. A 48-year-old woman beginning menopause E. A 62-year-old man with no past medical history I-19.  A 43-year-old woman is diagnosed with pulmonary blastomycosis and is initiated on therapy with oral itraconazole therapy. All of the following could affect the bioavailability of this drug EXCEPT: A. B. C. D. E.

Coadministration with a cola beverage Coadministration with oral contraceptive pills Formulation of the drug (liquid vs. capsule) pH of the stomach Presence of food in the stomach

I-20.  A 24-year-old woman with cystic fibrosis is admitted to the hospital with an exacerbation. She is known to be colonized with Pseudomonas aeruginosa and is started on intravenous therapy with cefepime 1 g IV every 8 hours and tobramycin 10 mg/kg IV once daily. You want to ensure that the risk of nephrotoxicity is low. When should the tobramycin level be checked? A. B. C. D. E.

30 minutes after the first dose 2 hours after the first dose 2 hours before second dose Immediately before the fourth dose There is no need to check drug levels if the patient has normal renal function

A. B. C. D. E.

QUESTIONS

A. Every 5 years beginning at age 30 years B. Once at age 30 years C. Once at age 30 years and again in 10 years if the test result is normal D. Periodically E. There is no recommended screening for thyroid disease recommended by the U.S. Preventive Services Task Force

I-21.  A 68-year-old man with ischemic cardiomyopathy has been treated with digoxin 250 μg daily for the past year. He has chronic kidney disease with a stable baseline creatinine of 2.1 mg/dL. He is initiated on an oral amiodarone load for new-onset atrial fibrillation with rapid ventricular response. Over 1 week, he develops increasing nausea, vomiting, and fatigue. On presentation to the emergency department, he is lethargic and difficult to arouse with a heart rate of 45 beats/min and a blood pressure of 88/50 mmHg. His laboratory values demonstrate a potassium of 5.2 meq/L, creatinine of 3.0 mg/dL, and a digoxin level of 13 ng/mL. His ECG shows complete heart block. What is the most appropriate treatment for this patient?

SECTION I

I-16.  According to the U.S. Preventive Services Task Force, what is the recommended screening interval for thyroid disease in women older than the age of 30 years?

CT scan of the head Electroencephalogram (EEG) Free phenytoin level Gram stain of ascites fluid Gram stain of cerebrospinal fluid (CSF)

3

SECTION I

I-23.  A 55-year-old Japanese woman is found to have a 3-cm mass in the right lower lobe of the lung. She is a lifelong nonsmoker. The mass is positive on positron emission tomography scan as are contralateral and ipsilateral lymph nodes in the mediastinum. A biopsy demonstrates the mass to be a moderately differentiated adenocarcinoma, and a left hilar lymph node also demonstrates adenocarcinoma. Clinically, this places the patient as a stage IIIB non–small cell lung cancer, and the patient and her oncologist decide to treat with chemotherapy. Molecular testing demonstrates an exon 19 deletion in the tyrosine kinase domain of the epidermal growth factor receptor and no mutation in k-ras. What is the best choice for initial chemotherapy in this patient?

Introduction to Clinical Medicine

A. B. C. D. E.

Carboplatin plus paclitaxel Carboplatin and paclitaxel plus erlotinib Docetaxel alone Erlotinib alone Gemcitabine plus docetaxel

I-24.  A 26-year-old woman received an allogeneic bone marrow transplant 9 months ago for acute myelogenous leukemia. Her transplant course is complicated by graftversus-host disease with diarrhea, weight loss, and skin rash. She is immunosuppressed with tacrolimus 1 mg bid and prednisone 7.5 mg daily. She recently was admitted to the hospital with shortness of breath and fevers to 101.5°F. She has a chest CT showing nodular pneumonia, and fungal organisms are seen on a transbronchial lung biopsy. The culture demonstrates Aspergillus fumigatus, and a serum galactomannan level is elevated. She is initiated on therapy with voriconazole 6 mg/kg IV every 12 hours for 1 day, decreasing to 4 mg/kg IV every 12 hours beginning on day 2. Two days after starting voriconazole, she is no longer febrile but is complaining of headaches and tremors. Her blood pressure is 150/92 mmHg, up from 108/60 mmHg on admission. On examination, she has developed 1+ pitting edema in the lower extremities. Her creatinine has risen to 1.7 mg/dL from 0.8 mg/dL on admission. What is the most likely cause of the patient’s current clinical picture? A. B. C. D. E.

Aspergillus meningitis Congestive heart failure Recurrent graft-versus-host disease Tacrolimus toxicity Thrombotic thrombocytopenic purpura caused by voriconazole

I-25.  A 45-year-old man is diagnosed with primary syphilis after development of a penile ulcer. Results of a rapid plasma reagin and fluorescent treponemal antibody absorption tests are both positive. He is treated with benzathine penicillin G 2.4 million units intramuscularly as a one-time dose. Ten days after the injection, the patient presents to the emergency department complaining of fevers, rash, and diffuse joint pains with muscle aches. On physical examination, the patient has a temperature of 38.3oF, heart rate of 110 beats/min, and blood pressure of 112/76 mmHg. His HEENT, chest, cardiovascular, and 4

abdominal examination findings are normal. He has an urticarial rash on trunk, back, and extremities. There is swelling and warmth of the knees, wrists, and metacarpophalangeal joints bilaterally. In addition, there is pain with palpation of the tendinous insertions of the Achilles tendons and patellar tendons bilaterally. The penile ulcer has a dry base and has decreased in size compared with previously. Laboratory studies show a white cell count of 10,100/μL (80% neutrophils, 15% lymphocytes, 3% monocytes, and 2% eosinophils). The erythrocyte sedimentation rate is 55 seconds. Antinuclear antibodies and rheumatoid factor results are negative. A urethral swab is negative for Chlamydia trachomatis and Neisseria gonorrhea. What is the most likely diagnosis? A. B. C. D. E.

Disseminated gonococcal infection Inadequate treatment of secondary syphilis Jarisch-Herxheimer reaction Seronegative rheumatoid arthritis Serum sickness caused by benzathine penicillin

I-26.  Which of the following classes of medicines has been linked to the occurrence of hip fractures in elderly adults? A. B. C. D. E.

Benzodiazepines Opiates Angiotensin-converting enzyme inhibitors Beta-blockers Atypical antipsychotics

I-27.  Patients taking which of the following drugs should be advised to avoid drinking grapefruit juice? A. B. C. D. E.

Amoxicillin Aspirin Atorvastatin Prevacid Sildenafil

I-28.  Which of the following diseases is responsible for a greater percentage of deaths in women compared with men? A. B. C. D. E.

Alzheimer’s disease Cerebrovascular disease Chronic obstructive pulmonary disease Sepsis All of the above

I-29.  Which of the following statements regarding coronary heart disease (CHD) in women when compared with men is TRUE? A. Angina is a rare symptom in women with CHD. B. At the time of diagnosis of CHD, women typically have fewer comorbidities compared with men. C. Physicians are less likely to consider CHD in women and are also less likely to recommend both diagnostic and therapeutic procedures in women. D. Women and men present with CHD at similar ages. E. Women are more likely to present with ventricular tachycardia, but men more commonly have cardiac arrest or cardiogenic shock.

A. B. C. D. E.

Elevated total triglyceride levels Hypertension Low high-density lipoprotein cholesterol Obesity Smoking

A. B. C. D. E.

Depression Hypertension Obesity Rheumatoid arthritis Type 1 diabetes mellitus

I-32.  Which of the following statements regarding Alzheimer’s disease and gender are true? A. Alzheimer’s disease affects men and women at equal rates. B. Alzheimer’s disease affects men two times more commonly than women. C. In a recent placebo-controlled trial, postmenopausal hormone therapy did not show improvement in disease progression in women with Alzheimer’s disease. D. The difference in deaths from Alzheimer’s disease between men and women can be entirely accounted for by the difference in life expectancy between men and women. E. Women with Alzheimer’s disease have higher levels of circulating estrogen than women without Alzheimer’s disease. I-33.  All of the following are changes in the cardiovascular system seen in pregnancy EXCEPT: A. B. C. D. E.

Decreased blood pressure Increased cardiac output Increased heart rate Increased plasma volume Increased systemic vascular resistance

I-34.  A 36-year-old woman has a history of hypertension and is planning on starting a family. She is currently taking lisinopril 10 mg daily for control of her blood pressure. She wants to stop taking her oral contraceptive medications. Her current blood pressure is 128/83 mmHg. What do you advise her about ongoing treatment with antihypertensive medications?

QUESTIONS

I-31.  All of the following diseases are more common in women than men EXCEPT:

A. Because the cardiovascular changes that occur during pregnancy lead to a fall in blood pressure, she can safely discontinue her lisinopril when she stops her oral contraceptives. B. She should continue lisinopril and start hydrochlorothiazide. C. She should discontinue lisinopril and start irbesartan. D. She should discontinue lisinopril and start labetalol. E. She should not get pregnant because she is high risk of complications.

SECTION I

I-30.  Which of the following is an independent risk factor for coronary heart disease in women but not men?

I-35.  Which of the following cardiovascular conditions is a contraindication to pregnancy? A. B. C. D. E.

Atrial septal defect without Eisenmenger syndrome Idiopathic pulmonary arterial hypertension Marfan syndrome Mitral regurgitation Prior peripartum cardiomyopathy with a current ejection fraction of 65%

I-36.  A 33-year-old woman with diabetes mellitus and hypertension presents to the hospital with seizures during week 37 of her pregnancy. Her blood pressure is 156/92 mmHg. She has 4+ proteinuria. Management should include all of the following EXCEPT: A. B. C. D.

Emergent delivery Intravenous labetalol Intravenous magnesium sulfate Intravenous phenytoin

I-37.  A 27-year-old woman develops left leg swelling during week 20 of her pregnancy. Left lower extremity ultrasonogram reveals a left iliac vein deep venous thrombosis (DVT). Proper management includes: A. B. C. D. E.

Bedrest Catheter-directed thrombolysis Enoxaparin Inferior vena cava filter placement Warfarin

I-38.  In which of the following categories should women undergo routine screening for gestational diabetes? A. Age greater than 25 years B. Body mass index greater than 25 kg/m2 C. Family history of diabetes mellitus in a first-degree relative D. African American E. All of the above I-39.  Which of the following surgeries would be considered at the greatest risk for postsurgical complications? A. B. C. D. E.

Carotid endarterectomy Non-emergent repair of a thoracic aortic aneurysm Resection of a 5-cm lung cancer Total colectomy for colon cancer Total hip replacement

5

SECTION I Introduction to Clinical Medicine

I-40.  A 64-year-old man is contemplating undergoing elective cholecystectomy for biliary colic and cholelithiasis. He has a history of coronary artery disease with coronary artery bypass surgery performed at the age of 51 after an anterior wall myocardial infarction. His most recent ejection fraction 2 years previously was 35%. He also has a 45 pack-year history of tobacco, quitting after his surgery 13 years previously. Since his bypass surgery, he reports failure to return to full functional capacity. You ask him about his current exercise capacity. Which of the following would be considered poor exercise tolerance and increase his risk of perioperative complications?

of the following is NOT a component of the seven elements for communicating bad news (P-SPIKES approach)?

A. Inability to achieve 4 metabolic equivalents during an exercise test B. Inability to carry 15–20 lb C. Inability to climb two flights of stairs at a normal pace D. Inability to walk four blocks at a normal pace E. All of the above

I-44.  Which of the following is not a component of a living will?

I-41.  A 74-year-old man is scheduled to undergo total colectomy for recurrent life-threatening diverticular bleeding. He denies any chest pain with exertion but is limited in his physical activity because of degenerative arthritis of his knees. He has no history of coronary artery disease or congestive heart failure but does have diabetes mellitus and hypertension. His current medications include aspirin 81 mg daily, atorvastatin 10 mg daily, enalapril 20 mg daily, and insulin glargine 25 units daily in combination with insulin lispro on a sliding scale. His blood pressure is 128/86 mmHg. His physical examination findings are normal. His most recent hemoglobin A1C is 6.3%, and his creatinine is 1.5 mg/dL. You elect to perform an electrocardiogram preoperatively, and it demonstrates Q waves in leads II, III, and aVF. Based on this information, what is his expected his postoperative risk of a major cardiac event? A. B. C. D. E.

0.5% 1% 5% 10% 20%

I-42.  All of the following are risk factors for postoperative pulmonary complications EXCEPT: A. B. C. D. E.

Age greater than 60 years Asthma with a peak expiratory flow rate of 220 L/min Chronic obstructive pulmonary disease Congestive heart failure Forced expiratory volume in 1 second of 1.5 L

I-43.  You are caring for a 56-year-old woman who was admitted to the hospital with a change in mental status. She underwent a right-sided mastectomy and axillary lymph node dissection 3 years previously for stage IIIB ductal carcinoma. Serum calcium is elevated at 15.3 mg/dL. A chest radiograph demonstrates innumerable pulmonary nodules, and a head CT shows a brain mass in the right frontal lobe with surrounding edema. Despite correcting her calcium and treating cerebral edema, the patient remains confused. You approach the family to discuss the diagnosis of widely metastatic disease and the patient’s poor prognosis. Which 6

A. Assess the family’s perception of her current illness and the status of her underlying cancer diagnosis. B. Empathize with the family’s feelings and provide emotional support. C. Prepare mentally for the discussion. D. Provide an appropriate setting for discussion. E. Schedule a follow-up meeting in 1 day to reassess whether there are additional informational and emotional needs.

A. Delineation of specific interventions that would be acceptable to the patient under certain conditions B. Description of values that should guide discussions regarding terminal care C. Designation of a health care proxy D. General statements regarding whether the patient desires receipt of life-sustaining interventions such as mechanical ventilation I-45.  A 72-year-old woman has stage IV ovarian cancer with diffuse peritoneal studding. She is developing increasing pain in her abdomen and is admitted to the hospital for pain control. She previously was treated with oxycodone 10 mg orally every 6 hours as needed. Upon admission, she is initiated on morphine intravenously via patient-controlled analgesia. During the first 48 hours of her hospitalization, she received an average daily dose of morphine 90 mg and reports adequate pain control unless she is walking. What is the most appropriate opioid regimen for transitioning this patient to oral pain medication?

A. B. C. D. E.

Sustained-Release Morphine None 45 mg twice daily 45 mg twice daily 90 mg twice daily 90 mg three time daily

Immediate-Release Morphine 15 mg every 4 hours as needed 5 mg every 4 hours as needed 15 mg every 4 hours as needed 15 mg every 4 hours as needed 15 mg every 4 hours as needed

I-46.  You are asked to consult on 62-year-old man who was recently found to have newly metastatic disease. He was originally diagnosed with cancer of the prostate 5 years previously and presented to the hospital with back pain and weakness. Magnetic resonance imaging (MRI) demonstrated bony metastases to his L2 and L5 vertebrae with spinal cord compression at the L2 level only. On bone scan images, there was evidence of widespread bony metastases. He has been started on radiation and hormonal therapy, and his disease has shown some response. However, he has become quite depressed since the metastatic disease was found. His family reports that he is sleeping for 18 or more hours daily and has stopped eating. His weight is down 12 lb over 4 weeks. He expresses profound fatigue, hopelessness, and a feeling of sadness. He claims to have no interest in his usual activities and no longer interacts

I-47.  You are treating a 76-year-old woman with Alzheimer’s disease admitted to the intensive care unit for aspiration pneumonia. After 7 days of mechanical ventilation, her family requests that care be withdrawn. The patient is palliated with fentanyl intravenously at a rate of 25 μg/hr and midazolam intravenously at 2 mg/hr. You are urgently called to the bedside 15 minutes after the patient is extubated because the patient’s daughter is distraught. She states that you are “drowning” her mother and is upset because her mother appears to be struggling to breathe. When you enter the room, you hear a gurgling noise that is coming from accumulated secretions in the oropharynx. You suction the patient for liberal amounts of thin salivary secretions and reassure the daughter that you will make her mother as comfortable as possible. Which of the following interventions may help with the treatment of the patient’s oral secretions? A. B. C. D. E.

Increased infusion rate of fentanyl N-acetylcysteine nebulized Pilocarpine drops Placement of a nasal trumpet and oral airway to allow easier access for aggressive suctioning Scopolamine patches

I-48.  Which of the following is the most common type of preventable adverse event in hospitalized patients? A. B. C. D. E.

Adverse drug events Diagnostic failures Falls Technical complications of procedures Wound infections

I-49.  All of the following statements regarding the use of complementary and alternative medicine (CAM) in the US are true EXCEPT: A. Acupuncture is the most frequently used CAM approach in the US B. CAM approaches represent approximately 10% of out-of-pocket medical expenses in the US C. Control of back or musculoskeletal pain is a common reason for US patients to utilize CAM approaches D. Recent estimates suggest 30-40% of Americans use CAM approaches E. The most common reasons US patients seek CAM approaches is for management of symptoms poorly controlled by conventional approaches

A. B. C. D. E.

Prescription of analgesic for pain control Referral to renal transplantation Surgical treatment for lung cancer Utilization of cardiac diagnostic and therapeutic procedures All of the above

QUESTIONS

A. Do not initiate pharmacologic therapy because the patient is experiencing an appropriate reaction to his newly diagnosed metastatic disease. B. Initiate therapy with doxepin 75 mg nightly. C. Initiate therapy with fluoxetine 10 mg daily. D. Initiate therapy with fluoxetine 10 mg daily and methylphenidate 2.5 mg twice daily in the morning and at noon. E. Initiate therapy with methylphenidate 2.5 mg twice daily in the morning and at noon.

I-50.  Independent of insurance status, income, age, and comorbid conditions, African American patients are less likely to receive equivalent levels of care compared with white patients for the following scenarios:

SECTION I

with his grandchildren. What is the best approach to treating this patient’s depression?

I-51.  All of the following statements regarding the difference between breast cancer in pregnant versus nonpregnant women are true EXCEPT: A. Estrogen-positive tumors are more common in pregnant women. B. Her-2 positivity is more common in pregnant women. C. A higher stage is more common in pregnant women. D. Positive lymph nodes are more common in pregnant women. E. Tumor size at diagnosis is larger in pregnant women. I-52.  A 32-year-old woman seeks evaluation for cough that has been present for 4 months. She reports that the cough is present day and night. It does awaken her from sleep and is worse in the early morning hours. She also notes the cough to be worse in cold weather and after exercise. She describes the cough as dry and has no associated shortness of breath or wheezing. She gives no antecedent history of an upper respiratory tract infection that preceded the onset of cough. She has a medical history of pulmonary embolus occurring in the postpartum period 6 years previously. Her only medication is norgestimate/ethinyl estradiol. She works as an elementary school teacher. On review of systems, she reports intermittent itchy eyes and runny nose that is worse in the spring and fall. She denies postnasal drip and heartburn. Her physical examination findings are normal with the exception of coughing when breathing through an open mouth. A chest radiograph is also normal. Spirometry demonstrates a forced expiratory volume in 1 second (FEV1) of 3.0 L (85% predicted), forced vital capacity (FVC) of 3.75 L (88% predicted), and FEV1/FVC ratio of 80%. After administration of a bronchodilator, the FEV1 increases to 3.3 L (10% change). What would you recommend next in the evaluation and treatment of this patient? A. B. C. D. E.

Initiate a nasal corticosteroid. Initiate a proton pump inhibitor. Perform a methacholine challenge test. Perform a nasopharyngeal culture for Bordetella pertussis. Reassure the patient that there are no pulmonary abnormalities and continue supportive care.

7

SECTION I

I-53.  A 56-year-old man presents to his primary care physician complaining of coughing up blood. He has felt ill for the past 4 days with a low-grade fever and cough. The cough was initially productive of yellow-green sputum, but it now is sputum mixed with red blood. He estimates that he has produced about 1–2 tsp (5–10 mL) of blood in the past day. He smokes 1 pack of cigarettes daily and has done so since the 15 years of age. He is known to have moderate chronic obstructive pulmonary disease and coronary artery disease. He takes aspirin, metoprolol, lisinopril, tiotropium, and albuterol as needed. His physical examination is notable for a temperature of 37.8°C (100.0°F). Bilateral expiratory wheezing and coarse rhonchi are heard on examination. Chest radiograph is normal. What is the most likely cause of hemoptysis in this individual?

Introduction to Clinical Medicine

A. B. C. D. E.

Acute bronchitis Infection with tuberculosis Lung abscess Lung cancer Medications

I-54.  A 65-year-old man with a known squamous cell carcinoma near the right upper lobe bronchus is admitted to intensive care after coughing up more than 100 mL of bright red blood. He appears in significant respiratory distress with an oxygen saturation of 78% on room air. He continues to have violent coughing with ongoing hemoptysis. He had a prior pulmonary embolus and is being treated with warfarin. His last INR was therapeutic at 2.5 three days previously. All of the following would be useful in the immediate management of this patient EXCEPT: A. Consultation with anesthesia for placement of a duallumen endotracheal tube. B. Consultation with interventional radiology for embolization. C. Consultation with thoracic surgery for urgent surgical intervention if conservative management fails. D. Correction of the patient’s coagulopathy. E. Positioning of the patient in the left lateral decubitus position. I-55.  Microbial agents have been used as bioweapons since ancient times. All of the following are key features of microbial agents that are used as bioweapons EXCEPT: A. B. C. D. E.

Environmental stability High morbidity and mortality rates Lack of rapid diagnostic capability Lack of readily available antibiotic treatment Lack of universally available and effective vaccine

I-56.  Ten individuals in Arizona are hospitalized over a 4-week period with fever and rapidly enlarging and painful lymph nodes. Seven of these individuals experience severe sepsis, and three die. While reviewing the epidemiologic characteristics of these individuals, you note that they are all illegal immigrants and have recently stayed in the same immigrant camp. Blood cultures are growing gram-negative rods that are identified as 8

Yersinia pestis. You notify local public health officials and the Centers for Disease Control and Prevention. Which of the following factors indicate that this is NOT likely to be an act of bioterrorism? A. The area affected was limited to a small immigrant camp. B. The individuals presented with symptoms of bubonic plague rather than pneumonic plague. C. The individuals were in close contact with one another, suggesting possible person-to-person transmission. D. The mortality rate was less than 50%. E. Yersinia pestis is not environmentally stable for longer than 1 hour. I-57.  Which of the following routes of dispersal are likely for botulinum toxin used as a bioweapon? A. B. C. D. E.

Aerosol Contamination of the food supply Contamination of the water supply A and B All of the above

I-58.  Anthrax spores can remain dormant in the respiratory tract for how long? A. B. C. D. E.

1 week 6 weeks 6 months 1 year 3 years

I-59.  Twenty recent attendees at a National Football League game arrive at the emergency department complaining of shortness of breath, fever, and malaise. Chest radiographs show mediastinal widening on several of these patients, prompting a concern for inhalational anthrax as a result of a bioterror attack. Antibiotics are initiated, and the Centers for Disease Control and Prevention is notified. What form of isolation should be instituted for these patients in the hospital? A. B. C. D.

Airborne Contact Droplet None

I-60.  The Centers for Disease Control and Prevention (CDC) has designated several biologic agents as category A in their ability to be used as bioweapons. Category A agents include agents that can be easily disseminated or transmitted, result in high mortality, can cause public panic, and require special action for public health preparedness. All of the following agents are considered category A EXCEPT: A. B. C. D. E.

Bacillus anthracis Francisella tularensis Ricin toxin from Ricinus communis Smallpox Yersinia pestis

A. B. C. D. E.

Arsine—asphyxiant Chlorine gas—pulmonary damage Cyanogen chloride—nerve agent Mustard gas—vesicant Sarin—nerve agent

A. 2-Pralidoxime should be administered to all affected individuals. B. The associated mortality rate of this agent is more than 50%. C. The cause of respiratory distress in affected individuals is related to direct alveolar injury and adult respiratory distress syndrome. D. The erythema that occurs can be delayed as long as 2 days after exposure and depends on several factors, including ambient temperature and humidity. E. The fluid within the bullae should be treated as a hazardous substance that can lead to local reactions and blistering with exposure. I-63.  A 24-year-old man is evaluated immediately after exposure to chlorine gas as an act of chemical terrorism. He currently denies dyspnea. His respiratory rate is 16 breaths/ min and oxygen saturation is 97% on room air. All of the following should be included in the immediate treatment of this individual EXCEPT: A. B. C. D. E.

A. B. C. D. E.

QUESTIONS

I-62.  Over the course of 12 hours, 24 individuals present to a single emergency department complaining of a sunburn-like reaction with development of large blisters. Most of these individuals are also experiencing irritation of the eyes, nose, and pharynx. Two individuals developed progressive dyspnea, severe cough, and stridor requiring endotracheal intubation. On physical examination, all of the patients exhibited conjunctivitis and nasal congestion. Erythema of the skin was greatest in the axillae, neck, and antecubital fossae. Many of the affected had large, thin-walled bullae on the extremities that were filled with a clear or straw-colored fluid. On further questioning, all of the affected individuals had been shopping at a local mall within the past 24 hours and ate at the food court. Many commented on a strong odor of burning garlic in the food court at that time. You suspect a bioterrorism act. Which of the following is TRUE with regard to the likely agent causing the patients’ symptoms?

nausea, vomiting, diarrhea, and muscle twitching. On physical examination, the patient has a blood pressure of 156/92 mmHg, a heart rate of 92, a respiratory rate of 30 breaths/min, and a temperature of 37.4°C (99.3°F). She has pinpoint pupils with profuse rhinorrhea and salivation. She also is coughing profusely, with production of copious amounts of clear secretions. A lung examination reveals wheezing on expiration in bilateral lung fields. The patient has a regular rate and rhythm with normal heart sounds. Bowel sounds are hyperactive, but the abdomen is not tender. She is having diffuse fasciculations. At the end of your examination, the patient abruptly develops tonic-clonic seizures. Which of the following agents is most likely to cause this patient’s symptoms?

SECTION I

I-61.  All of the following chemical agents of bioterrorism are correctly identified by their mechanism of injury EXCEPT:

Arsine Cyanogen chloride Nitrogen mustard Sarin VX

I-65.  All the following should be used in the treatment of this patient EXCEPT: A. B. C. D. E.

Atropine Decontamination Diazepam Phenytoin 2-Pralidoxime chloride

I-66.  All of the following statements are true regarding the results of detonation of a low-yield nuclear device by a terror group EXCEPT: A. After recovery of initial exposure symptoms, the patient remains at risk of systemic illness for up to 6 weeks. B. Appropriate medical therapy can change the LD50 from approximately 4–8 gray (Gy). C. Initial mortality is mostly caused by shock blast and thermal damage. D. Most of the total mortality is related to release of alpha and beta particles. E. The hematopoietic, gastrointestinal, and neurologic systems are most likely involved in acute radiation syndrome.

Aggressive bathing of all exposed skin areas Flushing of the eyes with water or normal saline Forced rest and fresh air Immediate removal of clothing if no frostbite Maintenance of a semiupright position

I-64.  You are a physician working in an urban emergency department when several patients are brought in after the release of an unknown gas at the performance of a symphony. You are evaluating a 52-year-old woman who is not able to talk clearly because of excessive salivation and rhinorrhea, although she is able to tell you that she feels as if she lost her sight immediately upon exposure. At present, she also has 9

SECTION I Introduction to Clinical Medicine

I-67.  A “dirty” bomb is detonated in downtown Boston. The bomb was composed of cesium-137 with trinitrotoluene. In the immediate aftermath, an estimated 30 people were killed because of the power of the blast. The fallout area was about 0.5 mile, with radiation exposure of about 1.8 Gy. An estimated 5000 people have been potentially exposed to beta and gamma radiation. Most of these individuals show no sign of any injury, but about 60 people have evidence of thermal injury. What is the most appropriate approach to treating the injured victims? A. All individuals who have been exposed should be treated with potassium iodide. B. All individuals who have been exposed should be treated with Prussian blue. C. All individuals should be decontaminated before transportation to the nearest medical center for emergency care to prevent exposure of health care workers. D. Severely injured individuals should be transported to the hospital for emergency care after removing the victims’ clothes because the risk of exposure to health care workers is low. E. With this degree of radiation exposure, no further testing and treatment are needed. I-68.  A 37-year-old woman is brought to the ICU after her elective laparoscopic cholecystectomy is complicated by a temperature of 105°F, tachycardia, and systemic hypotension. Examination is notable for diffuse muscular rigidity. Which of the following drugs should be administered immediately? A. B. C. D. E.

Acetaminophen Dantrolene Haloperidol Hydrocortisone Ibuprofen

I-69.  Hyperthermia is defined as: A. A core temperature greater than 40.0°C B. A core temperature greater than 41.5°C C. An uncontrolled increase in body temperature despite a normal hypothalamic temperature setting D. An elevated temperature that normalizes with antipyretic therapy E. Temperature greater than 40.0°C, rigidity, and autonomic dysregulation I-70.  Which of the following conditions is associated with increased susceptibility to heat stroke in elderly adults? A. B. C. D. E.

A heat wave Antiparkinsonian therapy Bedridden status Diuretic therapy All of the above

I-71.  A recent 18-year-old immigrant from Kenya presents to a university clinic with fever, nasal congestion, severe fatigue, and a rash. The rash started with discrete lesions at the hairline that coalesced as the rash spread caudally. There is sparing of the palms and soles. Small white spots 10

with a surrounding red halo are noted on examination of the palate. The patient is at risk for developing which of the following in the future? A. B. C. D. E.

Encephalitis Epiglottitis Opportunistic infections Postherpetic neuralgia Splenic rupture

I-72.  A 23-year-old woman with a chronic lower extremity ulcer related to prior trauma presents with rash, hypotension, and fever. She has had no recent travel or outdoor exposure and is up to date on all of her vaccinations. She does not use IV drugs. On examination, the ulcer looks clean with a well-granulated base and no erythema, warmth, or pustular discharge. However, the patient does have diffuse erythema that is most prominent on her palms, conjunctiva, and oral mucosa. Other than profound hypotension and tachycardia, the remainder of the examination is nonfocal. Laboratory results are notable for a creatinine of 2.8 mg/dL, aspartate aminotransferase of 250 U/L, alanine aminotransferase of 328 U/L, total bilirubin of 3.2 mg/dL, direct bilirubin of 0.5 mg/dL, INR of 1.5, activated partial thromboplastin time of 1.6 × control, and platelets at 94,000/μL. Ferritin is 1300 μg/mL. The patient is started on broad-spectrum antibiotics after appropriate blood cultures are drawn and is resuscitated with IV fluid and vasopressors. Her blood cultures are negative at 72 hours; at this point, her fingertips start to desquamate. What is the most likely diagnosis? A. B. C. D. E.

Juvenile rheumatoid arthritis (JRA) Leptospirosis Staphylococcal toxic shock syndrome Streptococcal toxic shock syndrome Typhoid fever

I-73.  A 75-year-old man with chronic systolic heart failure requiring high-dose diuretics and lisinopril is seen by his primary care physician for acute onset of right great toe pain with redness and swelling. He is unable to bear weight on this foot. On examination, he is afebrile and has normal vital signs. His complaints in his right great toe are verified. No other joints are involved, and he appears otherwise to be in well-compensated heart failure. Prednisone and allopurinol are prescribed. Five days later, the patient is seen in the emergency department with a temperature of 101°F and a rash throughout his body and mouth. On examination, he has diffuse erythema, areas of skin exfoliation, and oral and orbital edema. Mucous membranes are not involved. Laboratory studies show mild transaminitis and peripheral eosinophilia. Which of the following syndromes describes this condition? A. Acute bacterial endocarditis B. Angioedema caused by lisinopril C. Drug-induced hypersensitivity syndrome caused by allopurinol D. MRSA cellulitis E. Staphylococcal toxic shock syndrome caused by septic arthritis

Empiric treatment with corticosteroids Empiric treatment for Mycobacterium tuberculosis Needle biopsy of enlarged lymph nodes PET-CT imaging Serum angiotensin-converting enzyme levels

I-75.  A 48-year-old man is brought to the emergency department (ED) in January after being found unresponsive in a city park. He has alcoholism and was last seen by his daughter about 12 hours before being brought to the ED. At that time, he left their home intoxicated and agitated. He left seeking additional alcohol because his daughter had poured out his last bottle of vodka hoping that he would seek treatment. On presentation, he has a core body temperature of 88.5°F (31.4°C), heart rate of 48 beats/min, respiratory rate of 28 breaths/min, and blood pressure of 88/44 mmHg; oxygen saturation is unable to be obtained. The arterial blood gas demonstrates a pH of 7.05, PaCO2 of 32 mmHg, and PaO2 of 56 mmHg. Initial blood chemistries demonstrate a sodium of 132 meq/L, potassium of 5.2 meq/L, chloride of 94 meq/L, bicarbonate of 10 meq/L, blood urea nitrogen of 56 mg/dL, and creatinine of 1.8 mg/ dL. Serum glucose is 63 mg/dL. The serum ethanol level is 65 mg/dL. The measured osmolality is 328 mOsm/kg. ECG

A. Endotracheal intubation with hyperventilation to a goal PaCO2 of less than 20 mmHg B. Intravenous hydration with a 1–2 L bolus of warmed lactated Ringer’s solution C. No other measures are necessary because interpretation of the acid–base status is unreliable with this degree of hypothermia. D. Measure levels of ethylene glycol and methanol E. Placement of a transvenous cardiac pacemaker

QUESTIONS

A. B. C. D. E.

demonstrates sinus bradycardia with a long first-degree atrioventricular block and J waves. In addition to initiating a rewarming protocol, what additional tests should be performed in this patient?

SECTION I

I-74.  A 50-year-old man is evaluated for fevers and weight loss of uncertain etiology. He first developed symptoms 3 months previously. He reports daily fevers to as high as 39.4°C (103°F) with night sweats and fatigue. Over this same period, his appetite has been decreased, and he has lost 50 lb compared with his weight at his last annual examination. Fevers have been documented in his primary care physician’s office to as high as 38.7°C (101.7°F). He has no exposures or ill contacts. His medical history is significant for diabetes mellitus, obesity, and obstructive sleep apnea. He is taking insulin glargine 50 U daily. He works in a warehouse driving a forklift. He has not traveled outside of his home area in a rural part of Virginia. He has never received a blood transfusion and is married with one female sexual partner for the past 25 years. On examination, no focal findings are identified. Multiple laboratory studies have been performed that have shown nonspecific findings only with exception of an elevated calcium at 11.2 g/dL. A complete blood count showed a white blood cell count of 15,700/μL with 80% polymorphonuclear cells, 15% lymphocytes, 3% eosinophils, and 2% monocytes. The peripheral smear is normal. The hematocrit is 34.7%. His erythrocyte sedimentation rate (ESR) is elevated at 57 mm/hr. A rheumatologic panel is normal, and the ferritin is 521 ng/mL. Liver and kidney function are normal. The serum protein electrophoresis demonstrated polyclonal gammopathy. HIV, Epstein-Barr virus (EBV), and cytomegalovirus (CMV) testing are negative. The urine Histoplasma antigen result is negative. Routine blood cultures for bacteria, chest radiograph, and purified protein derivative (PPD) testing results are negative. A CT scan of the chest, abdomen, and pelvis has borderline enlargement of lymph nodes in the abdomen and retroperitoneum to 1.2 cm. What would be the next best step in determining the etiology of fever in this patient?

I-76.  A homeless man is evaluated in the emergency department. He has noted that after he slept outside during a particularly cold night his left foot has become clumsy and feels “dead.” On examination, the foot has hemorrhagic vesicles distributed throughout the foot distal to the ankle. The foot is cool and has no sensation to pain or temperature. The right foot is hyperemic but does not have vesicles and has normal sensation. The remainder of the physical examination findings are normal. Which of the following statements regarding the management of this disorder is true? A. Active foot rewarming should not be attempted. B. During the period of rewarming, intense pain can be anticipated. C. Heparin has been shown to improve outcomes in this disorder. D. Immediate amputation is indicated. E. Normal sensation is likely to return with rewarming. I-77.  A 25-year-old woman becomes lightheaded and experiences a syncopal event while having her blood drawn during a cholesterol screening. She has no medical history and takes no medications. She experiences a brief loss of consciousness for about 20 seconds. She has no seizure-like activity and immediately returns to her usual level of functioning. She is diagnosed with vasovagal syncope, and no follow-up testing is recommended. Which of the following statements regarding neurally mediated syncope is TRUE? A. Neurally mediated syncope occurs when there are abnormalities of the autonomic nervous system. B. Proximal and distal myoclonus do not occur during neurally mediated syncope and should increase the likelihood of a seizure. C. The final pathway of neurally mediated syncope results in a surge of the sympathetic nervous system with inhibition of the parasympathetic nervous system. D. The primary therapy for neurally mediated syncope is reassurance, avoidance of triggers, and plasma volume expansion. E. The usual finding with cardiovascular monitoring is hypotension and tachycardia.

11

SECTION I Introduction to Clinical Medicine

I-78.  A 76-year-old woman is brought to the emergency department after a syncopal event that occurred while she was singing in her church choir. She has a history of hypertension, diabetes mellitus, and chronic kidney disease (stage III). She does recall at least two prior episodes of syncope similar to this one. Her medications include insulin glargine 40 units daily, lispro insulin sliding scale, lisinopril 20 mg daily, and hydrochlorothiazide 25 mg daily. By the time she arrived in the emergency department, she reports feeling back to her usual self. She does recall feeling somewhat lightheaded before the syncopal events but does not recall the event itself. Witnesses report some jerking of her upper extremities. She regained full consciousness in less than 2 minutes. Her current vital signs include blood pressure of 110/62 mmHg, heart rate of 84 beats/min, respiratory rate of 16 breaths/min, and oxygen saturation of 95% on room air. She is afebrile. Her physical examination is unremarkable and includes a normal neurologic examination. Which of the following would be least helpful in determining the etiology of the patient’s syncope? A. B. C. D. E.

CT scan of the head Electrocardiogram Fingerstick glucose measurement Orthostatic blood pressure measurement Tilt table testing

I-79.  A 48-year-old man presents to the emergency department complaining of dizziness. He describes it as a sensation that the room is spinning. All of the following would be consistent with a central cause of vertigo EXCEPT: A. B. C. D. E.

Absence of tinnitus Gaze-evoked nystagmus Hiccups Inhibition of nystagmus by visual fixation Purely vertical nystagmus

I-80.  A 62-year-old woman presents complaining of severe dizziness. She notes it especially when she turns over in bed and immediately upon standing. Her initial physical examination findings are normal. Upon further testing, you ask the patient to sit with her head turned 45 degrees to the right. You lower the patient to the supine position and extend the head backward 20 degrees. This maneuver immediately reproduces the patient’s symptoms, and you note torsional nystagmus. What is the most appropriate next step in evaluation and treatment of this patient? A. B. C. D. E.

MRI of the brainstem Methylprednisolone taper beginning at 100 mg daily Repositioning (Epley) maneuvers Rizatriptan 10 mg orally once Valacyclovir 1000 mg three times daily for 7 days

I-81.  A 42-year-old man presents complaining of progressive weakness over a period of several months. He reports tripping over his toes while walking and has dropped a cup of hot coffee on one occasion because he felt too weak to continue to hold it. A disorder affecting lower motor neurons is suspected. All of the following findings would be found in an individual with a disease primarily affecting lower motor neurons EXCEPT: 12

A. B. C. D. E.

Decreased muscle tone Distal greater than proximal weakness Fasciculations Hyperactive tendon reflexes Severe muscle atrophy

I-82.  A 78-year-old man is seen in clinic because of recent falls. He reports gait difficulties with a sensation of being off balance at times. One recent fall caused a shoulder injury requiring surgery to repair a torn rotation cuff. In epidemiologic case series, what is the most common cause of gait disorders? A. B. C. D. E.

Cerebellar degeneration Cerebrovascular disease with multiple infarcts Cervical myelopathy Parkinson’s disease Sensory deficits

I-83.  A 65-year-old man presents complaining of frequent falls and gait abnormalities. He first noticed the difficulty about 6 months ago. He has a history of hypertension and hypothyroidism and hyperlipidemia. His current medications include amlodipine 10 mg daily, simvastatin 20 mg daily, and levothyroxine 75 μg daily. On neurologic examination, you observe his gait to be wide based with short, shuffling steps. He has difficulty rising from his chair and initiating his gait. Upon turning, he takes multiple steps and appears unsteady. However, cerebellar testing results are normal, including heel-to-shin and Romberg testing. He has no evidence of sensory deficits in the lower extremities, and strength is 5/5 throughout all tested muscle groups. He shows no evidence of muscle spasticity on passive movement. His neurologic examination is consistent with which of the following causes? A. B. C. D. E.

Alcoholic cerebellar degeneration Communicating hydrocephalus Neurosyphilis Multiple system atrophy Lumbar myelopathy

I-84.  A 74-year-old woman is admitted to the medical intensive care unit with confusion and sepsis from a urinary origin. Her initial blood pressure was 70/40 mmHg with a heart rate of 130 beats/min. She is volume resuscitated but requires dopamine to maintain an adequate blood pressure. Her mental status improved initially, but now she is agitated and pulling at her IV catheters. She is screaming that she is trapped, and she is not oriented to place or year. All of the following statements regarding the patient’s condition are true EXCEPT: A. An episode of delirium is associated with an inhospital mortality rate of 25% to 33%. B. A patient who has an episode of delirium in the hospital is more likely to be discharged to a nursing home. C. Delirium is associated with an increased risk of all-cause mortality for at least 1 year after hospital discharge. D. Delirium is typically short-lived and does not persist longer than several days. E. Individuals who experience delirium have longer lengths of stay in the hospital.

E.

Arterial blood gas testing Brain imaging with MRI or head CT Fingerstick glucose testing More thorough review of the patient’s alcohol intake with his wife Review of the recent medications received by the patient

A. A 36-year-old man admitted to the medical ward with a deep venous thrombosis B. A 55-year-old man postoperative day 2 from a total colectomy C. A 68-year-old woman admitted to the intensive care unit (ICU) with esophageal rupture D. A 74-year-old woman in the preoperative clinic before hip surgery E. An 84-year-old man living in an assisted living facility

QUESTIONS

A. B. C. D.

I-86.  Delirium, an acute confusional state, is a common disorder that remains a major cause of morbidity and mortality in the United States. Which of the following patients is at the highest risk for developing delirium?

SECTION I

I-85.  You are covering the night shift at a local hospital and are called acutely to the bedside of a 62-year-old man to evaluate a change in his mental status. He was admitted 36 hours previously for treatment of community-­ acquired pneumonia. He received treatment with levofloxacin 500 mg daily and required oxygen 2 L/min. He has a medical history of tobacco abuse, diabetes mellitus, and hypertension. He reports alcohol intake of 2–4 beers daily. His vital signs at 10 pm were blood pressure of 138/85 mmHg, heart rate of 92 beats/min, respiratory rate of 20 breaths/min, temperature of 37.4°C (99.3°F), and SaO2 of 92% on oxygen 2 L/min. Currently, the patient is agitated and pacing his room. He is reporting that he needs to leave the “meeting” immediately and go home. He states that if he does not do this, someone is going to take his house and car away. He has removed his IV and oxygen tubing from his nose. His last vital signs taken 30 minutes previously were blood pressure of 156/92 mmHg, heart rate of 118 beats/min, respiratory rate of 26 breaths/min, temperature of 38.3°C (100.9°F), and oxygen saturation of 87% on room air. He is noted to be somewhat tremulous and diaphoretic. All of the following should be considered as part of the patient’s diagnostic workup EXCEPT:

I-87.  Which of the following is the most common finding in aphasic patients? A. B. C. D. E.

Alexia Anomia Comprehension Fluency Repetition

I-88.  A 65-year-old man experiences an ischemic cerebro­ vascular accident affecting the territory of the right anterior cerebral artery. After the stroke, an assessment reveals the findings shown in Figure I-88. What diagnosis does this figure suggest? A. B. C. D. E.

Construction apraxia Hemianopia Hemineglect Object agnosia Simultanagnosia

FIGURE I-88

13

SECTION I Introduction to Clinical Medicine

I-89.  A 42-year-old man is evaluated for excessive sleepiness that is interfering with his ability to work. He works at a glass factory that requires him to work rotating shifts. He typically cycles across day (7 am–3 pm), evening (3 pm–11 pm), and night (11 pm–7 am) shifts over the course of 4 weeks. He notes the problem to be most severe when he is on the night shift. Twice he has fallen asleep on the job. Although no accidents have occurred, he has been threatened with loss of his job if he falls asleep again. His preferred sleep schedule is 10 pm until 6 am, but even when he is working day shifts, he typically only sleeps from about 10:30 pm until 5:30 am. However, he feels fully functional at work on day and evening shifts. After his night shifts, he states that he finds it difficult to sleep when he first gets home, frequently not falling asleep until 10 am or later. He is up by about 3 pm when his children arrive home from school. He drinks about 2 cups of coffee daily but tries to avoid drinking more than this. He does not snore and has a body mass index of 21.3 kg/m2. All of the following are reasonable approaches to treatment in this man EXCEPT: A. Avoidance of bright light in the morning after his shifts B. Exercise in the early evening before going to work C. Melatonin 3 mg taken at bedtime on the morning after a night shift D. Modafinil 200 mg taken 30–60 minutes before starting a shift E. Strategic napping of no more than 20 minutes during breaks at work I-90.  A 45-year-old woman presents for evaluation of abnormal sensations in her legs that keep her from sleeping at night. She first notices the symptoms around 8 pm when she is sitting quietly watching television. She describes the symptoms as “ants crawling in my veins.” Although the symptoms are not painful, they are very uncomfortable and worsen when she lies down at night. They interfere with her ability to fall asleep about four times weekly. If she gets out of bed to walk or rubs her legs, the symptoms disappear almost immediately only to recur as soon as she is still. She also sometimes takes a very hot bath to alleviate the symptoms. During sleep, her husband complains that she kicks him throughout the night. She has no history of neurologic or renal disease. She currently is perimenopausal and has been experiencing very heavy and prolonged menstrual cycles over the past several months. The physical examination findings, including thorough neurologic examination, are normal. Her hemoglobin is 9.8 g/dL and hematocrit is 30.1%. The mean corpuscular volume is 68 fL. Serum ferritin is 12 ng/mL. Which is the most appropriate initial therapy for this patient? A. B. C. D. E.

Carbidopa/levodopa Hormone replacement therapy Iron supplementation Oxycodone Pramipexole

I-91.  A 20-year-old man presents for evaluation of excessive daytime somnolence. He is finding it increasingly difficult 14

to stay awake during his classes. Recently, his grades have fallen because whenever he tries to read, he finds himself drifting off. He finds that his alertness is best after exercising or brief naps of 10–30 minutes. Because of this, he states that he takes 5 or 10 “catnaps” daily. The sleepiness persists despite averaging 9 hours of sleep nightly. In addition to excessive somnolence, he reports occasional hallucinations that occur as he is falling asleep. He describes these occurrences as a voice calling his name as he drifts off. Perhaps once weekly, he awakens from sleep but is unable to move for a period of about 30 seconds. He has never had apparent loss of consciousness but states that whenever he is laughing, he feels heaviness in his neck and arms. Once he had to lean against a wall to keep from falling down. He undergoes an overnight sleep study and multiple sleep latency test. There is no sleep apnea. His mean sleep latency on five naps is 2.3 minutes. In three of the five naps, rapid eye movement sleep is present. Which of the following findings of this patient is most specific for the diagnosis of narcolepsy? A. B. C. D. E.

Cataplexy Excessive daytime somnolence Hypnagogic hallucinations Rapid eye movement sleep in more than two naps on a multiple sleep latency test Sleep paralysis

I-92.  Which of the following is the most common sleep disorder in the U.S. population? A. B. C. D. E.

Delayed sleep phase syndrome Insomnia Obstructive sleep apnea Narcolepsy Restless legs syndrome

I-93.  In which stage of sleep are the parasomnias somnambulism and night terrors most likely to occur? A. B. C. D.

Stage 1 Stage 2 Slow-wave sleep Rapid eye movement sleep

I-94.  A 44-year-old man is seen in the emergency department after a motor vehicle accident. The patient says, “I never saw that car coming from the right side.” On physical examination, his pupils are equal and reactive to light. His visual acuity is normal; however, there are visual field defects in both eyes laterally (bitemporal hemianopia). Which of the following is most likely to be found on further evaluation? A. B. C. D. E.

Retinal detachment Occipital lobe glioma Optic nerve injury Parietal lobe infarction Pituitary adenoma

I-95.  A 42-year-old construction worker complains of waking up with a red, painful left eye. She often works without goggles at her construction site. Her history is notable for hypertension, inflammatory bowel disease, diabetes, and

Acute angle-closure glaucoma Anterior uveitis Corneal abrasion Posterior uveitis Transient ischemic attack

I-96.  A 75-year-old triathlete complains of gradually worsening vision over the past year. It seems to be involving near and far vision. The patient has never required corrective lenses and has no significant medical history other than diet-controlled hypertension. He takes no regular medications. Physical examination is normal except for bilateral visual acuity of 20/100. There are no focal visual field defects and no redness of the eyes or eyelids. Which of the following is the most likely diagnosis? A. B. C. D. E.

Age-related macular degeneration Blepharitis Diabetic retinopathy Episcleritis Retinal detachment

I-97.  All of the following statements regarding olfaction are true EXCEPT: A. Decrements in olfaction may lead to nutritional deficiency. B. More than 40% of patients with traumatic anosmia will regain normal function over time. C. Significant decrements in olfaction are present in more than 50% of the population 80 years and older. D. The most common identifiable cause of long-lasting or permanent loss of olfaction in outpatients is severe respiratory infection. E. Women identify odorants better than men at all ages. I-98.  A 64-year-old man is evaluated for hearing loss that he thinks is worse in his left ear. His wife and children have told him for years that he does not listen to them. Recently, he has failed to hear the chime of the alarm on his digital watch, and he admits to focusing on the lips of individuals speaking to him because he sometimes has difficulties in word recognition. In addition, he reports a continuous buzzing that is louder in his left ear. He denies any sensation of vertigo, headaches, or balance difficulties. He has worked in a factory for many years that makes parts for airplanes, and the machinery that he works with sits to his left primarily. He has no family history of deafness, although his father had hearing loss as he aged. He has a medical history of hypertension, hyperlipidemia, and coronary artery disease. You suspect sensorineural hearing loss related to exposure to the intense noise in the factory for many decades. Which of the following findings would you expect on physical examination?

I-99.  A 32-year-old woman presents to her primary care physician complaining of nasal congestion and drainage and headache. Her symptoms originally began about 7 days ago with rhinorrhea and sore throat. For the past 5 days, she has been having increasing feelings of fullness and pressure in the maxillary area that is causing her headaches. The pressure is worse when she bends over, and she also notices it while lying in bed at night. She is otherwise healthy and has not had fevers. On physical examination, there is purulent nasal drainage and pain with palpation over bilateral maxillary sinuses. What is the best approach to ongoing management of this patient?

QUESTIONS

A. B. C. D. E.

A. A deep tympanic retraction pocket seen above the pars flaccida on the tympanic membrane. B. Cerumen impaction in the external auditory canal. C. Hearing loss that is greater at lower frequencies on pure tone audiometry. D. Increased intensity of sound when a tuning fork is placed on the mastoid process when compared with placement near the auditory canal. E. Increased intensity of sound in the right ear when a tuning fork is placed in the midline of the forehead.

SECTION I

prior IV drug use. Her only current medication is lisinopril. On examination, the left eye is diffusely red and sensitive to light. The eyelids are normal. In dim light, visual acuity is normal in both eyes. All of the following diagnoses will explain her findings EXCEPT:

A. Initiate therapy with amoxicillin 500 mg three times daily for 10 days. B. Initiate therapy with levofloxacin 500 mg daily for 10 days. C. Perform a sinus aspirate for culture and sensitivities. D. Perform a sinus CT. E. Treat with oral decongestants and nasal saline lavage. I-100.  A 28-year-old man seeks evaluation for sore throat for 2 days. He has not had a cough or rhinorrhea. He has no other medical conditions and works as a daycare provider. On examination, tonsillar hypertrophy with membranous exudate is present. What is the next step in the management of this patient? A. Empiric treatment with amoxicillin 500 mg twice daily for 10 days B. Rapid antigen detection test for Streptococcus pyogenes only C. Rapid antigen detection test for Streptococcus pyogenes plus throat culture if the rapid test result is negative D. Rapid antigen detection test for Streptococcus pyogenes plus a throat culture regardless of result E. Throat culture only I-101.  A 62-year-old man presents to his physician complaining of shortness of breath. All of the following findings are consistent with left ventricular dysfunction as a cause of the patient’s dyspnea EXCEPT: A. B. C. D. E.

Feeling of chest tightness Nocturnal dyspnea Orthopnea Pulsus paradoxus greater than 10 mmHg Sensation of air hunger

15

SECTION I Introduction to Clinical Medicine

I-102.  A 42-year-old woman seeks evaluation for a cough that has been present for almost 3 months. The cough is mostly dry and non-productive, but occasionally productive of yellow phlegm. She reports that the cough is worse at night and often wakes her from sleep. She denies any recent upper respiratory tract infection, allergic rhinitis, fever, chills or cough. She recalls her mother told her that she had asthma as a child but she has never felt symptomatic wheezing as an adult. She exercises regularly but continues to smoke 1 pack per day of cigarettes; she’d like to quit. The patient takes no medications. Her physical examination is unremarkable. Which of the following is indicated at this point? A. Chest PET-CT B. Chest radiograph C. Measurement of serum angiotensin-converting enzyme (ACE) D. Measurement of serum IgE E. Sinus CT I-103.  In the patient described above, her chest radiograph is normal and further history reveals a long history of symptoms suggestive of GERD. She also admits that her cough is worse on nights after a large or late meal. She often has a bad taste in her mouth as she starts coughing. Based on this information, which of the following would be a reasonable empiric therapeutic trial? A. B. C. D. E.

Inhaled corticosteroid Inhaled long acting beta agonist Nasal corticosteroid Oral proton pump inhibitor Oral triple antibiotic therapy for H. pylori

I-104.  A 48-year-old man is evaluated for hypoxia of unknown etiology. He recently has noticed shortness of breath that is worse with exertion and in the upright position. It is relieved with lying down. On physical examination, he is visibly dyspneic with minimal exertion. He is noted to have a resting oxygen saturation of 89% on room air. When lying down, his oxygen saturation increases to 93%. His pulmonary examination shows no wheezes or crackles. His cardiac examination findings are normal without murmur. His chest radiograph reports a possible 1-cm lung nodule in the right lower lobe. On 100% oxygen and in the upright position, the patient has an oxygen saturation of 90%. What is the most likely cause of the patient’s hypoxia? A. B. C. D. E.

Circulatory hypoxia Hypoventilation Intracardiac right-to-left shunting Intrapulmonary right-to-left shunting Ventilation–perfusion mismatch

I-105.  A patient is evaluated in the emergency department for peripheral cyanosis. All of the following are potential etiologies EXCEPT:

16

A. B. C. D. E.

Cold exposure Deep venous thrombosis Methemoglobinemia Peripheral vascular disease Raynaud’s phenomenon

I-106.  An 18-year-old college freshman is being evaluated for a heart murmur heard at health screening. She reports an active lifestyle, no past medical history, and no cardiac symptoms. She has a midsystolic murmur that follows a nonejection sound and crescendos with S2. The murmur duration is greater when going from supine to standing and decreases when squatting. The murmur is heard best along the lower left sternal border and apex. Her electrocardiogram is normal. Which of the following is the most likely condition causing the murmur? A. B. C. D. E.

Aortic stenosis Hypertrophic obstructive cardiomyopathy Mitral valve prolapse Pulmonic stenosis Tricuspid regurgitation

I-107.  Which of the following characteristics makes a heart murmur more likely to be caused by tricuspid regurgitation than mitral regurgitation? A. B. C. D. E.

Decreased intensity with amyl nitrate Inaudible A2 at the apex Prominent c-v wave in jugular pulse Onset signaled by a midsystolic click Wide splitting of S2

I-108.  You are examining a 25-year-old patient in clinic who came in for a routine examination. Cardiac auscultation reveals a second heart sound that is split and does not vary with respiration. There is also a grade 2–3 midsystolic murmur at the midsternal border. Which of the following is most likely? A. B. C. D. E.

Atrial septal defect Hypertrophic obstructive cardiomyopathy Left bundle branch block Normal physiology Pulmonary hypertension

I-109.  A 32-year-old woman presents to her physician complaining of hair loss. She is currently 10 weeks postpartum after delivery of a normal healthy baby girl. She admits to having increased stress and sleep loss because her child has colic. She also has not been able to nurse because of poor milk production. On examination, the patient’s hair does not appear to have decreased density. With a gentle tug, more than 10 hairs come out but are not broken and all appear normal. There are no scalp lesions. What do you recommend for this patient? A. Careful evaluation of the patient’s hair care products for a potential cause B. Reassurance only C. Referral for counseling for trichotillomania D. Treatment with minoxidil E. Treatment with topical steroids

I-111.  A 44-year-old woman is prescribed phenytoin for the development of complex partial seizures. One month after initiating the medication, she is evaluated for a diffuse erythematous eruption with associated fever to 101.3°F. She is noted to have facial edema with diffusely enlarged lymph nodes along the cervical, axillary, and inguinal areas. Her white cell count is 14,500/μL (75% neutrophils, 12% lymphocytes, 5% atypical lymphocytes, and 8% eosinophils). A basic metabolic panel is normal, but elevations in the liver functions tests are noted with an AST of 124 U/L, ALT of 148 U/L, alkaline phosphatase of 114 U/L, and total bilirubin of 2.2 mg/dL. All of the following are indicated in the management of this patient EXCEPT: A. B. C. D. E.

Administration of carbamazepine 200 mg twice daily Administration of prednisone 1.5–2 mg/kg daily Administration of topical glucocorticoids Discontinuation of phenytoin Evaluation for development of thyroiditis for up to 6 months

I-112.  Which of the following drugs is associated with development of both phototoxicity and photoallergy? A. B. C. D. E.

Amiodarone Diclofenac Doxycycline Hydrochlorothiazide Levofloxacin

I-113.  You are seeing a patient in follow-up in whom you have begun an evaluation for an elevated hematocrit. You suspect polycythemia vera based on a history of aquagenic pruritus and splenomegaly. Which set of laboratory tests is consistent with the diagnosis of polycythemia vera? A. Elevated red blood cell mass, high serum erythropoietin levels, and normal oxygen saturation B. Elevated red blood cell mass, low serum erythropoietin levels, and normal oxygen saturation C. Normal red blood cell mass, high serum erythropoietin levels, and low arterial oxygen saturation D. Normal red blood cell mass, low serum erythropoietin levels, and low arterial oxygen saturation

A. B. C. D. E.

Angiodysplasia of the small bowel Epistaxis Menorrhagia Postpartum hemorrhage Spontaneous hemarthrosis

I-115.  A 68-year-old man is admitted to the intensive care unit with spontaneous retroperitoneal bleeding and hypotension. He has a medical history of hypertension, diabetes mellitus, and chronic kidney disease stage III. His medications include lisinopril, amlodipine, sitagliptin, and glimepiride. On initial presentation, he is in pain and has a blood pressure of 70/40 mmHg with a heart rate of 132 beats/min. His hemoglobin on admission is 5.3 g/dL and hematocrit is 16.0%. His coagulation studies demonstrate an aPTT of 64 seconds and a PT of 12.1 seconds (INR 1.0). Mixing studies (1:1) are performed. Immediately, the aPTT decreases to 42 seconds. At 1 hour, the aPTT is 56 seconds, and at 2 hours, it is 68 seconds. Thrombin time and reptilase time are normal. Fibrinogen is also normal. What is the most likely cause of the patient’s coagulopathy? A. B. C. D. E.

QUESTIONS

A. Cross-linking of IgE molecules fixed to sensitized cells in the presence of a specific drug-protein conjugate B. Deposition of circulating immune complexes C. Development of drug-specific T-cell immunogenicity D. Direct mast cell degranulation E. Hepatic metabolism into toxic intermediate

I-114.  All of the following are common manifestations of bleeding caused by von Willebrand disease EXCEPT:

SECTION I

I-110.  A 26-year-old man develops diffuse itching, wheezing, and laryngeal edema within minutes of receiving intravenous radiocontrast media for an intravenous pyelogram. He has not previously received contrast dye per his recollection. He is treated with supportive care and recovers without further complications. Which of the following best describes the mechanism of the patient’s reaction to the contrast media?

Acquired factor VIII deficiency Acquired factor VIII inhibitor Heparin Lupus anticoagulant Vitamin K deficiency

I-116.  A 54-year-old man is seen in the clinic complaining of painless enlargement of lymph nodes in his neck. He has not otherwise been ill and denies fevers, chills, weight loss, and fatigue. His past medical history is remarkable for pulmonary tuberculosis that was treated 10 years previously under directly observed therapy. He currently takes no medications. He is a heterosexual man in a monogamous relationship for 25 years. He denies illicit drug use. He has smoked 1½ packs of cigarettes daily since 16 years of age. He works as a logger. On physical examination, the patient is thin, but not ill-appearing. He is not febrile and has normal vital signs. He has dental caries noted with gingivitis. In the right supraclavicular area, there is a hard and fixed lymph node measuring 2.5 × 2.0 cm in size. Lymph nodes less than 1 cm in size are noted in the anterior cervical chain. There is no axillary or inguinal lymphadenopathy. His liver and spleen are not enlarged. Which of the following factors in history or physical examination increases the likelihood that the lymph node enlargement is caused by malignancy? A. B. C. D. E.

Age greater than 50 years Location in the supraclavicular area Presence of a lymph node that is hard and fixed Size greater than 2.25 cm2 (1.5 × 1.5 cm) All of the above

17

SECTION I Introduction to Clinical Medicine

I-117.  A 24-year-old woman presents for a routine checkup and complains only of small masses in her groin. She states that they have been present for at least 3 years. She denies fever, malaise, weight loss, and anorexia. She works as a sailing instructor and competes in triathlons. On physical examination, she is noted to have several palpable 1-cm inguinal lymph nodes that are mobile, nontender, and discrete. There is no other lymphadenopathy or focal findings on examination. What should be the next step in management? A. B. C. D. E. F.

Bone marrow biopsy CT scan of the chest, abdomen, and pelvis Excisional biopsy Fine-needle aspiration for culture and cytopathology Pelvic ultrasonography Reassurance

I-118.  All of the following diseases are associated with massive splenomegaly (spleen extends 8 cm below the costal margin or weighs >1000 g) EXCEPT: A. B. C. D. E.

Autoimmune hemolytic anemia Chronic lymphocytic leukemia Cirrhosis with portal hypertension Marginal zone lymphoma Myelofibrosis with myeloid metaplasia

I-119.  The presence of Howell-Jolly bodies, Heinz bodies, basophilic stippling, and nucleated red blood cells in a patient with hairy cell leukemia before any treatment intervention implies which of the following? A. B. C. D. E.

Diffuse splenic infiltration by tumor Disseminated intravascular coagulation (DIC) Hemolytic anemia Pancytopenia Transformation to acute leukemia

I-120.  Which of the following is true regarding infection risk after elective splenectomy? A. Patients are at no increased risk of viral infection after splenectomy. B. Patients should be vaccinated 2 weeks after splenectomy. C. Splenectomy patients over the age of 50 are at greatest risk for postsplenectomy sepsis. D. Staphylococcus aureus is the most commonly implicated organism in postsplenectomy sepsis. E. The risk of infection after splenectomy increases with time. I-121.  An 18-year-old man is seen in consultation for a pulmonary abscess caused by infection with Staphylococcus aureus. He had been in his usual state of health until 1 week ago when he developed fevers and a cough. He has no ill contacts and presents in the summer. His medical history is significant for episodes of axillary and perianal abscesses requiring incision and drainage. He cannot specifically recall how often this has occurred, but he does know it has been more than five times that he can recall. In one 18

instance, he recalls a lymph node became enlarged to the point that it “popped” and drained spontaneously. He also reports frequent aphthous ulcers and is treated for eczema. On physical examination, his height is 5′3′′. He appears ill with a temperature of 39.6°C. Eczematous dermatitis is present in the scalp and periorbital area. There are crackles at the left lung base. Axillary lymphadenopathy is present bilaterally and is tender. The spleen in enlarged. His laboratory studies show a white blood cell count of 12,500/μL (94% neutrophils), hemoglobin of 11.3 g/dL, hematocrit of 34.2%, and platelets of 320,000/μL. Granulomatous inflammation is seen on lymph node biopsy. Which of the following tests are most likely found in this patient? A. B. C. D. E.

Elevated angiotensin-converting enzyme level Eosinophilia Giant primary granules in neutrophils Mutations of the tumor necrosis factor-alpha receptor Positive nitroblue tetrazolium dye test

I-122.  A 72-year-old man with chronic obstructive pulmonary disease and stable coronary disease presents to the emergency department with several days of worsening productive cough, fevers, malaise, and diffuse muscle aches. A chest radiograph demonstrates a new lobar infiltrate. Laboratory measurements reveal a total white blood cell count of 12,100 cells/μL with a neutrophilic predominance of 86% and 8% band forms. He is diagnosed with community-acquired pneumonia, and antibiotic treatment is initiated. Under normal, or “nonstress,” conditions, what percentage of the total body neutrophils are present in the circulation? A. B. C. D. E.

2% 10% 25% 40% 90%

I-123.  A patient with longstanding HIV infection, alcoholism, and asthma is seen in the emergency department for 1–2 days of severe wheezing. He has not been taking any medicines for months. He is admitted to the hospital and treated with nebulized therapy and systemic glucocorticoids. His CD4 count is 8 and viral load is greater than 750,000. His total white blood cell (WBC) count is 3200 cells/μL with 90% neutrophils. He is accepted into an inpatient substance abuse rehabilitation program and before discharge is started on opportunistic infection prophylaxis, bronchodilators, a prednisone taper over 2 weeks, ranitidine, and highly active antiretroviral therapy. The rehabilitation center pages you 2 weeks later; a routine laboratory check reveals a total WBC count of 900 cells/μL with 5% neutrophils. Which of the following new drugs would most likely explain this patient’s neutropenia? A. B. C. D. E.

Darunavir Efavirenz Ranitidine Prednisone Trimethoprim–sulfamethoxazole

A. Chronic mercury poisoning is best assessed using hair samples. B. Ethyl mercury preservative in multiuse vaccines has not been implicated in causing autism. C. Exposure to as little as a few drops of dimethylmercury may be lethal. D. Offspring of mothers who ingested mercurycontaminated fish are at higher risk of neurobehavioral abnormalities. E. Pregnant women should avoid consumption of sardines and mackerel.

A. Drug effects begin earlier, peak earlier, and last longer. B. Drug effects begin earlier, peak later, and last longer. C. Drug effects begin earlier, peak later, and last shorter. D. Drug effects begin later, peak earlier, and last shorter. E. Drug effects begin later, peak later, and last longer.

I-125.  A 39-year-old man comes to clinic reporting a 4-day illness that began while he was in the Caribbean on vacation. A few hours after attending a large seafood buffet, he developed abdominal pain, chills, nausea, and diarrhea. Soon thereafter, he noticed diffused paresthesias, throat numbness, and fatigue. The symptoms slowly improved over 2 days, and he returned home yesterday. Today he noticed while washing that cold water felt hot and warm water felt cold. He is concerned about this new symptom. All of the following are true regarding his illness EXCEPT: A. His symptoms should improve over weeks to months. B. It is likely caused by ingestion of contaminated snapper or grouper. C. It is likely caused by ingestion of undercooked oysters or clams. D. Subsequent episodes may be more severe. E. No diagnostic laboratory test is available. I-126.  Which of the following is the most common cause of death from poisoning? A. B. C. D. E.

Acetaminophen Carbon monoxide Chlorine gas Insecticide Tricyclic antidepressants

I-127.  Which of the following is a distinguishing feature of amphetamine overdose versus other causes of sympathetic overstimulation caused by drug overdose or withdrawal? A. B. C. D. E.

Hallucination Hot, dry, flushed skin and urinary retention History of benzodiazepine abuse Markedly increased blood pressure, heart rate, and end-organ damage in the absence of hallucination Nystagmus

I-128.  A patient with metabolic acidosis, reduced anion gap, and increased osmolal gap is most likely to have which of the following toxic ingestions? A. B. C. D. E.

I-130.  Which of the following statements regarding gastric decontamination for toxin ingestion is true? A. Activated charcoal’s most common side effect is aspiration. B. Gastric lavage via nasogastric tube is preferred over the use of activated charcoal when therapeutic endoscopy may also be warranted. C. Syrup of ipecac has no role in the hospital setting. D. There are insufficient data to support or exclude a benefit when gastric decontamination is used more than 1 hour after a toxic ingestion. E. All of the above are true.

QUESTIONS

I-129.  Which of the following is true regarding drug effects after an overdose compared with a reference dose?

SECTION I

I-124.  All of the following statements regarding mercury exposure or poisoning are true EXCEPT:

I-131.  One of your patients is contemplating a trekking trip to Nepal at elevations between 2500 and 3000 m. Five years ago, while skiing at Telluride (altitude, 2650 m), she recalls having headache, nausea, and fatigue within 1 day of arriving that lasted about 2–3 days. All of the following are true regarding the development of acute mountain sickness in this patient EXCEPT: A. Acetazolamide starting 1 day before ascent is effective in decreasing the risk. B. Gingko biloba is not effective in decreasing the risk. C. Gradual ascent is protective. D. Her prior episode increases her risk for this trip. E. Improved physical conditioning before the trip decreases the risk. I-132.  A 36-year-old man develops shortness of breath, dyspnea, and dry cough 3 days after arriving for helicopter snowboarding in the Bugaboo mountain range in British Columbia (elevation, 3000 m). Over the next 12 hours, he becomes more short of breath and produces pink, frothy sputum. An EMT-trained guide hears crackles on chest examination. All of the following are true regarding his illness EXCEPT: A. B. C. D. E.

Descent and oxygen are most therapeutic. Exercise increased his risk. Fever and leukocytosis may occur. He should never risk return to high altitude after recovery. Pretreatment with nifedipine or tadalafil would have lowered his risk.

Lithium Methanol Oxycodone Propylene glycol Salicylate 19

SECTION I

I-133.  Which of the following is considered an absolute contraindication to hyperbaric oxygen therapy? A. B. C. D. E.

Carbon monoxide poisoning History of COPD History of high altitude pulmonary edema Radiation proctitis Untreated pneumothorax

Introduction to Clinical Medicine

I-134.  A 35-year-old woman is scuba diving while vacationing in Malaysia. During her last dive of the day, her regulator malfunctions, requiring her to ascend from 20 m to the surface rapidly. Upon returning to the boat, she feels well. However, about 6 hours after returning to shore, she develops diffuse itching and muscle aches, leg pain, blurred vision, slurred speech, and nausea. Which of the following statements regarding her condition is true? A. Decompression illness is unlikely at 20-m water depth. B. Inhalation of 100% oxygen is contraindicated. C. She can never again scuba dive to a depth greater than 6 m. D. She should receive recompression and hyperbaric oxygen therapy. E. She should remain upright as much as possible. I-135.  Which of the following statements regarding the distinction between acute lung injury (ALI) and acute respiratory distress syndrome (ARDS) is true? A. ALI and ARDS can be distinguished by radiographic testing. B. ALI and ARDS can be distinguished by the magnitude of the PaO2/FIO2 ratio. C. ALI can be diagnosed in the presence of elevated left atrial pressure, but ARDS can not. D. ALI is caused by direct lung injury, but ARDS is the result of secondary lung injury. E. The risk of ALI but not ARDS increases with multiple predisposing conditions. I-136.  Which of the following has been demonstrated to reduce mortality in patients with ARDS? A. High-dose glucocorticoids within 48 hours of presentation B. High-frequency mechanical ventilation C. Inhaled nitric oxide D. Low tidal volume mechanical ventilation E. Surfactant replacement I-137.  A 38-year-old man is hospitalized in the ICU with ARDS after a motor vehicle accident with multiple long bone fractures, substantial blood loss, and hypotension. By day 2 of hospitalization, he is off vasopressors but is requiring a high FIO2 and positive end-expiratory pressure (PEEP) to maintain adequate oxygenation. His family is asking about the short- and long-term prognosis for recovery. All of the following statements about his prognosis are true EXCEPT:

20

A. He has a greater chance of survival than a patient with similar physiology who is older than 70 years old. B. His overall mortality from ARDS is approximately 25–45%. C. If he survives, he is likely to have some degree of depression or posttraumatic stress disorder. D. If he survives, he likely will have normal or near normal lung function. E. The most likely cause of mortality is hypoxemic respiratory failure. I-138.  Clinical trials support the use of noninvasive ventilation in which of the following patients? A. A 33-year-old man who was rescued from a motor vehicle accident. He is unarousable with possible internal injuries. Room air blood gas is 7.30 (pH), PCO2 50 mmHg, PO2 60 mmHg. B. A 49-year-old woman with end-stage renal disease admitted with presumed staphylococcal sepsis from her hemodialysis catheter. She is somnolent, blood pressure is 80/50 mmHg, heart rate is 105 beats/min, and room air oxygen saturation is 95%. C. A 58-year-old woman with a history of cirrhotic liver disease admitted with a presumed esophageal variceal bleed. Her blood pressure is 75/55 mmHg, and she has a heart rate of 110 beats/min. She is awake and alert. D. A 62-year-old man with a long history of COPD admitted with an exacerbation related to an upper respiratory tract infection. He is in marked respiratory distress but is awake and alert. Chest radiograph only shows hyperinflation. His room air arterial blood gas is pH, 7.28; PCO2, 75 mmHg; and PO2, 46 mmHg. E. A 74-year-old man with cardiogenic shock and an acute ST-segment elevation myocardial infarction. His blood pressure is 84/65 mmHg, heart rate is 110 beats/min, respiratory rate is 24 breaths/min, and room air oxygen saturation is 85%. I-139.  You are caring for a patient on mechanical ventilation in the intensive care unit. Whenever the patient initiates a breath, no matter her spontaneous respiratory rate, she gets a fixed volume breath from the machine that does not change from breath to breath. After receiving a dose of sedation, she does not initiate any breaths, but the machine delivers the same volume breath at periodic fixed intervals during this time. Which of the following modes of mechanical ventilation is this patient receiving? A. B. C. D. E.

Assist control Continuous positive airway pressure Pressure control Pressure support Synchronized intermittent mandatory ventilation (SIMV)

I-140.  A 68-year-old woman has been receiving mechanical ventilation for 10 days for community-acquired pneumonia. You are attempting to decide whether the patient is

A. B. C. D.

I-141.  A 45-year-old woman with HIV is admitted to the intensive care unit with pneumonia and pneumothorax secondary to infection with Pneumocystis jiroveci. She requires mechanical ventilatory support, chest tube placement, and central venous access. The ventilator settings are PC mode; inspiratory pressure, 30 cmH2O, 1.0; and PEEP, 10 cmH2O. An arterial blood gas measured on these settings shows: pH 7.32, 46 mmHg, and 62 mmHg. All of the following are important supportive measures for this patient EXCEPT: A. B. C. D. E.

Analgesia to maintain patient comfort Daily change of ventilator circuit Gastric acid suppression Nutritional support Prophylaxis against deep venous thrombosis

I-142.  All of the following statements about the physiology of mechanical ventilation are true EXCEPT: A. Application of PEEP decreases left ventricular preload and afterload. B. High inspired tidal volumes contribute to the development of acute lung injury caused by overdistention of alveoli with resultant alveolar damage. C. Increasing the inspiratory flow rate will decrease the ratio of inspiration to expiration (I:E) and allow more time for expiration. D. Mechanical ventilation provides assistance with inspiration and expiration. E. PEEP helps prevent alveolar collapse at end-expiration. I-143.  A 64-year-old man requires endotracheal intubation and mechanical ventilation for chronic obstructive pulmonary disease. He was paralyzed with rocuronium for intubation. His initial ventilator settings were AC mode; respiratory rate 10 breaths/minute; FIO2 1.0; Vt (tidal volume) 550 mL; and positive end-expiratory pressure 0 cm H2O. On admission to the intensive care unit the patient remains paralzyed; arterial blood gas is pH 7.22, PCO2 78 mmHg, PO2 394 mmHg. The FIO2 is decreased to 0.6. Thirty minutes later you are called to the bedside to evaluate the patient for hypotension. Current vital signs are blood pressure 80/40 mmHg, heart rate, 133 beats/min; respiratory rate, 24/minute; and oxygen saturation 92%. Physical examination shows the patient is agitated and moving all extremities, a prolonged expiration with wheezing continuing until the initiation of the next breath. Breath sounds are heard in both lung fields. The high-pressure alarm on the ventilator is triggering. What should be done first in treating this patient’s hypotension?

Administer a fluid bolus of 500 mL. Disconnect the patient from the ventilator. Initiate a continuous IV infusion of midazolam. Initiate a continuous IV infusion of norepinephrine. Perform tube thoracostomy on the right side.

I-144.  All of the following are relative contraindications for the use of succinylcholine as a paralytic for endotracheal intubation EXCEPT: A. B. C. D. E.

Acetaminophen overdose Acute renal failure Crush injuries Muscular dystrophy Tumor lysis syndrome

QUESTIONS

E.

Alert mental status PEEP of 5 cmH2O pH greater than 7.35 Rapid shallow breathing index (respiratory rate/tidal volume) greater than 105 SaO2 greater than 90% and FIO2 less than 0.5

A. B. C. D. E.

SECTION I

a­ ppropriate for a spontaneous breathing trial. All of the following factors would indicate that the patient is likely to be successfully extubated EXCEPT:

I-145.  Match the following vasopressors with the statement that best describes their action on the cardiovascular system. 1. 2. 3. 4. A.

Dobutamine Low-dose dopamine (2–4 μg/kg/min) Norepinephrine Phenylephrine Acts solely at α-adrenergic receptors to cause vasoconstriction B. Acts at β1-adrenergic receptors and dopaminergic receptors to increase cardiac contractility and heart rate; also causes vasodilatation and increased splanchnic and renal blood flow C. Acts at β1- and, to a lesser extent, β2-adrenergic receptors to increase cardiac contractility, heart rate, and vasodilatation D. Acts at α- and β1-adrenergic receptors to increase heart rate, cardiac contractility, and vasoconstriction I-146.  An 86-year-old nursing home resident is brought by ambulance to the local emergency department. He was found unresponsive in his bed immersed in black stool. Apparently, he had not been feeling well for 1–2 days, had complained of vague abdominal pain, and had decreased oral intake; no further history is available from the nursing home staff. His past medical history is remarkable for Alzheimer’s dementia and treated prostate cancer. The emergency responders were able to appreciate a faint pulse and obtained a blood pressure of 91/49 mmHg and a heart rate of 120 beats/min. In the emergency department, his pressure is 88/51 mmHg and heart rate is 131 beats/min. He is moaning and obtunded, localizes to pain, and has flat neck veins. Skin tenting is noted. A central venous catheter is placed that reveals CVP less than 5 mmHg, specimens for initial laboratory testing are sent off, and electrocardiogram and chest x-ray are obtained. Catheterization of the bladder yields no urine. Anesthesiology has been called to the bedside and is assessing the patient’s airway. What is the best immediate step in management? A. Infuse hypertonic saline to increase the rate of vascular filling. B. Infuse isotonic crystalloid solution via IV wide open. C. Infuse a colloidal solution rapidly. D. Initiate inotropic support with dobutamine. E. Initiate IV pressors starting with Levophed. 21

SECTION I

I-147.  In the patient described above, which of the following is true regarding his clinical condition?

Introduction to Clinical Medicine

A. Loss of 20–40% of the blood volume leads to shock physiology. B. Loss of less than 20% of the blood volume will manifest as orthostasis. C. Oliguria is a crucial prognostic sign of impending vascular collapse. D. Symptoms of hypovolemic shock differ from those of hemorrhagic shock. E. The first sign of hypovolemic shock is mental obtundation. I-148.  A 52-year-old man presents with crushing substernal chest pain. He has a history of coronary artery disease and has had two non–ST-segment elevation myocardial infarctions in the past 5 years, both requiring percutaneous intervention and intracoronary stent placement. His electrocardiogram shows ST elevations across the precordial leads, and he is taken emergently to the catheterization laboratory. After angioplasty and stent placement, he is transferred to the coronary care unit. His vital signs are stable on transfer; however, 20 minutes after arrival, he is found to be unresponsive. His radial pulse is thready, extremities are cool, and blood pressure is difficult to obtain; with a manual cuff, it is 65/40 mmHg. The nurse turns to you and asks what you would like to do next. Which of the following accurately represents the physiologic characteristics of this patient’s condition?

A. B. C. D. E.

22

Central Venous Pressure Decreased Decreased Increased Increased Decreased

Cardiac Output Decreased Increased Increased Decreased Decreased

Systemic Vascular Resistance Decreased Decreased Decreased Increased Increased

mmHg, heart rate of 122 beats/min, temperature of 39.1°C, respiratory rate of 24 breaths/min, and oxygen saturation of 97% on room air. Physical examination shows clear lung fields and a regular tachycardia without murmur. There is no abdominal tenderness or masses. Stool is negative for occult blood. There are no rashes. Hematologic studies show a white blood cell count of 24,200/μL with a differential of 82% PMNs, 8% band forms, 6% lymphocytes, and 3% monocytes. Hemoglobin is 8.2 g/dL. A urinalysis has numerous white blood cells with gram-negative bacteria on Gram stain. Chemistries reveal the following: bicarbonate of 16 meq/L, BUN of 60 mg/dL, and creatinine of 2.4 mg/dL. After fluid administration of 2 L, the patient has a blood pressure of 88/54 mmHg and a heart rate of 112 beats/min with a central venous pressure of 18 cmH2O. There is 25 mL of urine output in the first hour. The patient has been initiated on antibiotics with cefepime. What should be done next for the treatment of this patient’s hypotension? A. B. C. D. E.

Dopamine, 3 μg/kg/min IV Hydrocortisone, 50 mg IV every 6 hours Norepinephrine, 2 μg/min IV Ongoing colloid administration at 500–1000 mL/h Transfusion of 2 units of packed red blood cells

I-151.  All of the following statements about the pathogenesis of sepsis and septic shock are true EXCEPT: A. Blood cultures are positive in only 20–40% of cases of severe sepsis. B. Microbial invasion of the bloodstream is not necessary for the development of severe sepsis. C. Serum levels of TNF-alpha are typically reduced in patients with severe sepsis or septic shock. D. The hallmark of septic shock is a marked decrease in peripheral vascular resistance that occurs despite increased plasma levels of catecholamines. E. Widespread vascular endothelial injury is present in severe sepsis and is mediated by cytokines and procoagulant factors that stimulate intravascular thrombosis.

I-149.  All of the following are factors that are related to the increased incidence of sepsis in the United States EXCEPT:

I-152.  Which of the following treatments is recommended to improve mortality in septic shock?

A. Aging of the population B. Increased longevity of individuals with chronic disease C. Increased risk of sepsis in individuals without comorbidities D. Increased risk of sepsis in individuals with AIDS E. Increased use of immunosuppressive drugs

A. Activated protein C (drotrecogin alpha) B. Administration of antibiotics within 1 hour of presentation C. Bicarbonate therapy for severe acidosis D. Erythropoietin E. Vasopressin infusion

I-150.  A 68-year-old woman is brought to the emergency department for fever and lethargy. She first felt ill yesterday and experienced generalized body aches. Overnight, she developed a fever of 39.6°C and had shaking chills. By this morning, she was feeling very fatigued. Her son feels that she has had periods of waxing and waning mental status. She denies cough, nausea, vomiting, diarrhea, and abdominal pain. She has a medical history of rheumatoid arthritis. She takes prednisone, 10 mg daily, and methotrexate, 15 mg weekly. On examination, she is lethargic but appropriate. Her vital signs are blood pressure of 85/50

I-153.  All of the following statements regarding cardiogenic shock are true EXCEPT:

A. B. C. D. E.

Increased heart rate Increased left ventricular afterload Lower diastolic blood pressure Not contraindicated in acute aortic regurgitation Reduced myocardial oxygen consumption

I-155.  Which of the following is the most common electrical mechanism to explain sudden cardiac death? A. B. C. D. E.

Asystole Bradycardia Pulseless electrical activity (PEA) Pulseless ventricular tachycardia (PVT) Ventricular fibrillation

I-156.  All of the following statements regarding successful resuscitation from sudden cardiac death are true EXCEPT: A. Advanced age does not affect the likelihood of immediate resuscitation, only the probability of hospital discharge. B. After cardiac out of hospital cardiac arrest, survival rates are approximately 25% if defibrillation is administered after 5 minutes. C. If the initial rhythm in an out-of-hospital cardiac arrest is pulseless ventricular tachycardia, the patient has a higher probability of survival than asystole. D. Prompt CPR followed by prompt defibrillation improves outcomes in all settings. E. The probability of survival from cardiac arrest is higher if the event takes place in a public setting than at home. I-157.  A 28-year-old woman has severe head trauma after a motor vehicle accident. One year after the accident, she is noted to have spontaneous eye opening and is able to track an object visually at times. She does not speak or follow any commands. She breathes independently but is fed through a gastrostomy tube. She can move all extremities spontaneously but without purposeful movement. What term best describes this patient’s condition?

Coma Locked-in Minimally conscious state Persistent vegetative state Vegetative state

I-158.  A 52-year-old man is evaluated after a large subarachnoid hemorrhage (SAH) from a ruptured cerebral aneurysm. There is concern that the patient has brain death. What test is most commonly used to diagnose brain death in this situation? A. B. C. D. E.

Apnea testing Cerebral angiography Demonstration of absent cranial nerve reflexes Demonstration of fixed and dilated pupils Performance of transcranial Doppler ultrasonography

QUESTIONS

I-154.  Aortic counterpulsation with an intra-aortic balloon pump has which of the following as an advantage over therapy with infused vasopressors or inotropes in a patient with acute ST-segment elevation myocardial infarction and cardiogenic shock?

A. B. C. D. E.

SECTION I

A. Approximately 80% of cases of cardiogenic shock complicating acute myocardial infarction are attributable to acute severe mitral regurgitation. B. Cardiogenic shock is more common in ST-segment elevation than non–ST-segment elevation myocardial infarction. C. Cardiogenic shock is uncommon in inferior wall myocardial infarction. D. Cardiogenic shock may occur in the absence of significant coronary stenosis. E. Pulmonary capillary wedge pressure is elevated in cardiogenic shock.

I-159.  Which of the following neurologic phenomena is classically associated with herniation of the brain through the foramen magnum? A. Third-nerve compression and ipsilateral papillary dilation B. Catatonia C. “Locked-in” state D. Miotic pupils E. Respiratory arrest I-160.  A 72-year-old woman is admitted to the intensive care unit after a cardiac arrest at home. She had a witnessed collapse, and her family immediately began to perform cardiopulmonary resuscitation. Emergency medical service arrived within 10 minutes, and the initial cardiac rhythm demonstrated ventricular fibrillation. Spontaneous circulation returned after defibrillation, and the estimated time the patient was without a pulse was 15–20 minutes. The patient is brought to hospital and remains intubated, paralyzed, and sedated in the coronary care unit. She is being treated with medically induced hypothermia and is completely unresponsive to all stimuli 12 hours after the initial event. Her pupils are 3 mm and respond sluggishly to light. She has no cough or gag reflex. Intermittent myoclonic jerks are seen. The family has concerns about her neurologic prognosis after her prolonged cardiac arrest. What advice do you give the family regarding prognosis in this situation? A. An MRI scan of the brain should be performed before determining neurologic outcome. B. Apnea testing will be performed at the first opportunity to determine if the patient has suffered brain death. C. Given the immediate actions of the family to initiate cardiopulmonary resuscitation, the patient has a greater than 50% chance to have good neurologic outcomes. D. It is impossible to predict the patient’s likelihood of neurologic recovery as her examination is unreliable in the face of sedation and hypothermia. E. No information regarding prognosis can be determined until 72 hours have passed.

23

SECTION I Introduction to Clinical Medicine

I-161.  A 52-year-old man presents to the emergency department complaining of the worst headache of his life that is unresolving. It began abruptly 3 days before presentation and is worse with bending over. It rapidly increased in intensity over 30 minutes, but he did not seek medical care at that time. Over the ensuing 72 hours, the headache has persisted although lessened in intensity. He has not lost consciousness and has no other neurologic symptoms. His vision is normal, but he does report that light is painful to his eyes. His past medical history is notable for hypertension, but he takes his medications irregularly. Upon arrival to the emergency department, his initial blood pressure is 232/128 mmHg with a heart rate of 112 beats/min. No nuchal rigidity is present. A head CT shows no acute bleeding and no mass effect. What is the next best step in the management of this patient? A. B. C. D. E.

Cerebral angiography CT angiography Lumbar puncture Magnetic resonance angiography Treat with sumatriptan

I-162.  A 56-year-old man is admitted to intensive care with a subarachnoid hemorrhage. Upon admission, he is unresponsive, and his head CT shows evidence of blood in the third ventricle with midline shift. He undergoes successful coiling of an aneurysm of the anterior cerebral artery. All of the following would be indicated in the management of this patient EXCEPT: A. B. C. D. E.

Glucocorticoids Hypernatremia Nimodipine Ventriculostomy Volume expansion

I-163.  A 56-year-old man is admitted to the intensive care unit with a hypertensive crisis after cocaine use. Initial blood pressure is 245/132 mmHg. On physical examination, the patient is unresponsive except to painful stimuli. He has been intubated for airway protection and is being mechanically ventilated, with a respiratory rate of 14 breaths/ min. His pupils are reactive to light, and he has normal corneal, cough, and gag reflexes. The patient has a dense left hemiparesis. When presented with painful stimuli, the patient responds with flexure posturing on the right side. Computed tomography (CT) reveals a large area of intracranial bleeding in the right frontoparietal area. Over the next several hours, the patient deteriorates. The most recent examination reveals a blood pressure of 189/100 mmHg. The patient now has a dilated pupil on the right side. The patient continues to have corneal reflexes. You suspect rising intracranial pressure related to the intracranial bleed. All but which of the following can be done to decrease the patient’s intracranial pressure?

24

A. Administer intravenous mannitol at a dose of 1 g/kg body weight. B. Administer hypertonic fluids to achieve a goal sodium level of 155–160 meq/L. C. Consult neurosurgery for an urgent ventriculostomy. D. Initiate intravenous nitroprusside to decrease the mean arterial pressure (MAP) to a goal of 100 mmHg. E. Increase the respiratory rate to 30 breaths/min. I-164.  A 64-year-old man presents to the emergency department complaining of shortness of breath and facial swelling. He smokes 1 pack of cigarettes daily and has done so since the age of 16 years. On physical examination, he has dyspnea at an angle of 45 degrees or less. His vital signs are heart rate of 124 beats/min, blood pressure of 164/98 mmHg, respiratory rate of 28 breaths/min, temperature of 37.6°C (99.6°F), and oxygen saturation of 89% on room air. Pulsus paradoxus is not present. His neck veins are dilated and do not collapse with inspiration. Collateral venous dilation is noted on the upper chest wall. There is facial edema and 1+ edema of the upper extremities bilaterally. Cyanosis is present. There is dullness to percussion and decreased breath sounds over the lower half of the right lung field. Given this clinical scenario, what would be the most likely finding on CT examination of the chest? A. A central mass lesion obstructing the right mainstem bronchus B. A large apical mass invading the chest wall and brachial plexus C. A large pericardial effusion D. A massive pleural effusion leading to opacification of the right hemithorax E. Enlarged mediastinal lymph nodes causing obstruction of the superior vena cava I-165.  In the scenario in question I-165, the initial therapy of this patient includes all of the following EXCEPT: A. Administration of furosemide as needed to achieve diuresis B. Elevation of the head of the bed to 45 degrees C. Emergent radiation D. Low-sodium diet E. Oxygen I-166.  A 58-year-old woman with known stage IV breast cancer presents to the emergency department with an inability to move her legs. She has had lower back pain for the past 4 days and has found it difficult to lie down. There is no radiating pain. Earlier today, the patient lost the ability to move either of her legs. In addition, she has been incontinent of urine recently. She has been diagnosed previously with metastatic disease to the lung and pleura from her breast cancer but was not known to have spinal or brain metastases. Her physical examination confirms absence of movement in the bilateral lower extremities associated with decreased to absent sensation below the umbilicus. There is increased tone and 3+ deep tendon reflexes in the

I-167.  A 21-year-old man is treated with induction chemotherapy for acute lymphoblastic leukemia. His initial white blood cell count before treatment was 156,000/μL. All of the following are expected complications during his treatment EXCEPT:

Acute kidney injury Hypercalcemia Hyperkalemia Hyperphosphatemia Hyperuricemia

I-168.  All of the following would be important for prevention of these complications EXCEPT: A. Administration of allopurinol 300 mg/m2 daily B. Administration of intravenous fluids at a minimum of 3000 mL/m2 daily C. Alkalinization of the urine to a pH of greater than 7.0 by administration of sodium bicarbonate D. Frequent monitoring of serum chemistries every 4 hours E. Prophylactic hemodialysis before initiating chemotherapy

ANSWERS

A. Administer dexamethasone 10 mg intravenously. B. Consult neurosurgery for emergent spinal decompression. C. Consult radiation oncology for emergent spinal radiation. D. Perform MRI of the brain. E. Perform MRI of the entire spinal cord.

A. B. C. D. E.

SECTION I

lower extremities with crossed adduction. Anal sphincter tone is decreased, and the anal wink reflex is absent. What is the most important first step to take in the management of this patient?

ANSWERS I-1.

The answer is D. (Chap. 1) Evidence-based medicine (EBM) is an important cornerstone to the effective and efficient practice of internal medicine. EBM refers to the concept that clinical decisions should be supported by data with the strongest evidence gleaned from randomized controlled clinical trials. Clearly, in some situations, it is impossible or unethical to perform randomized controlled trials, and data from observational studies such as cohort or case-control studies supply important information regarding disease associations. Professional organizations and government agencies use EBM to develop clinical practice guidelines. These guidelines combine the best available evidence from clinical and observational studies with expert opinion to develop clinical-decision support tools (option C). The purpose of clinical guidelines is to provide a framework for diagnosis and treatment of a specific clinical problem in a cost-effective and efficient manner. When multiple clinical trials have been published, accumulated data can be summarized in a systematic review (option A). In a systematic review, the researchers carefully scrutinize the methods of published trials for inclusion into the review and use statistical analysis to attempt to provide additional strength to clinical findings. A new branch of research called comparative effectiveness research (option B) attempts to compare different approaches to treating disease to determine effectiveness from both a clinical and cost-effectiveness standpoint. A variety of methods can be used, and systematic reviews are an important tool in comparative effectiveness research. The weakest type of evidence is anecdotal evidence (option E), which is one individual’s clinical experience in treating a disease and can be biased by prior experiences.

I-2.

The answer is D. (Chap. 1) Before performing any procedure, a physician has the ethical duty to discuss the details of the procedure with the patient and ensuring that he or she understands before proceeding. This process includes ensuring that the patient has the mental capacity to provide consent, outlining the risks and benefits of the procedure, and discussing alternatives and potential consequences of these alternatives. Informed consent does not require that a patient outline his or her wishes if he or she becomes incompetent to make decisions. This is accomplished in an advanced directive, which can outline the goals of care and also appoint someone to make medical decisions.

I-3.

The answer is A. (Chap. 2) Disability-adjusted life years (DALYs) is the standard measure for determining global burden of disease by the World Health Organization. This measure 25

SECTION I

takes into account both absolute years of life lost because of disease (premature death) as well as productive years lost because of disability. DALYs is believed to more accurately reflect the true effects of disease within a population because individuals who become disabled cannot contribute fully to society. Life expectancy, years of life lost because of disease, standardized mortality ratios, and infant mortality do provide important information about the general health of a population but do not capture the true burden of disease. The answer is E. (Chap. 2) The causes of morbidity and burden of disease in a population differ from the absolute causes of mortality in a population. Unipolar depressive disorder accounts for 10.0 million disability-adjusted life years lost (DALYs) in high-income countries. Depression is quite common in the general population of developed countries. However, the death rate from depression is low and is mainly reflected in suicides. Thus, depression creates disability and lost productivity without a significant impact on years of life lost. Depression often presents at young ages and persists or recurs throughout a lifetime, leading to significant morbidity over time. After unipolar depressive disorder, the leading causes of DALYs lost in high-income countries are ischemic heart disease, cerebrovascular disease, Alzheimer’s disease and other dementia, and alcohol use disorders. However, worldwide, the leading cause of DALYs is lower respiratory infections caused by the high burden of disease in low-income countries with an estimated 76.9 million DALYs lost because of lower respiratory infections in low-income countries. Additionally, in lowincome countries, the top five causes of DALYs are related to infectious diseases (diarrheal diseases, HIV, malaria) and prematurity.

I-5.

The answer is D. (Chap. 2) Although ischemic heart disease is the leading cause of death worldwide, low-income countries have a disproportionate number of deaths caused by lower respiratory tract infections. This primarily reflects the large numbers of individuals in low-income countries who die of tuberculosis and other infectious pneumonias. Ischemic heart disease is the second leading cause of death in low-income countries.

I-6.

The answer is B. (Chap. 2) Global health experts have developed priorities for improving global health in conjunction with the World Health Organization (WHO). Many of these efforts are focused on the prevention, early recognition, and treatment of infectious diseases in developing countries and the developing world. Among infectious causes of disease, malaria ranks as the third most deadly. In 2001, the WHO Roll Back Malaria campaign was endorsed by heads of state in Africa in an effort to develop a coordinated plan for malaria prevention and treatment. A major goal of the Roll Back Malaria campaign was to prevent gains in disease prevention in one country from being lost because of lack of a coordinated effort in neighboring countries. This effort involves a multifaceted approach that includes vector control, prevention of transmission, and early recognition and treatment. Insecticide-treated bed nets are a simple and cost-effective method of reducing malaria transmission with a 50% decreased incidence of malaria in individuals who sleep under these bed nets. Indoor residual spraying is also an important factor in decreasing malaria transmission as outdoor vector control alone is ineffective in controlling transmission. It has been found that 80% of structures in a community must be treated to decrease disease transmission. Another important part of decreasing disease transmission is to give at least two doses of effective antimalarial drugs during pregnancy to decrease placental transmission of disease. If disease is unable to be prevented, it is important to recognize and treat the disease early. Chloroquine resistance has emerged in many areas around the world, particularly in sub-Saharan Africa, the Middle East, India, Southeast Asia, and parts of South America. Given the widespread chloroquine resistance, the WHO now recommends only artemisinin-based combination therapy for falciparum malaria infection.

I-7.

The answer is A. (Chap. 3) Bayes’ theorem is a statistical model based on conditional probabilities that is useful in medical decision making. The three components of Bayes’ theorem as it relates to medical decision making are the pretest probability of disease, the sensitivity of the test, and the specificity of the test. These factors are combined into the following formula:

Introduction to Clinical Medicine

I-4.

26

Posttest probability =

Pretest probability × Test sensitivity (Pretest probability × Test sensitivity) + [(1 - Pretest probability) × False-positive rate]

Posttest probability = (0.10)(0.66)/[(0.10)(0.66) + (0.90)(0.16)] = 0.31

ANSWERS

In most occasions, the pretest probability is an estimate based on the prevalence of disease in the population and the clinical situation. The false-positive rate is 1 - specificity. In this clinical scenario, the pretest probability of disease was estimated at 10%, and the treadmill ECG stress test has an average sensitivity of 66% and a specificity of 84%. Based on the formula above, the posttest probability would be low at only 31%.

SECTION I



I-8 and I-9.  The answers are C and C, respectively. (Chap. 3) In evaluating the usefulness of a test, it is imperative to understand the clinical implications of the sensitivity and specificity of that test. Simply, the sensitivity is the proportion of people with disease that are correctly identified by the test—the true-positive rate. Alternatively, the specificity can be viewed as the true-negative rate and is the proportion of individuals without disease who would have a negative test result. The perfect test would have a sensitivity of 100% and a specificity of 100%, but this is unachievable in clinical practice. Sensitivity and specificity are inherent properties of the test and are not affected by the disease prevalence. However, by obtaining information about the prevalence of the disease in the population, one can generate a two-by-two table, as shown below. This table is used to generate the total number of patients in each group of the population. The sensitivity of the test is TP/(TP + FN). The specificity is TN/(TN + FP). In this case, the disease prevalence is 10%. In a population of 1000 individuals, 100 would truly have latent tuberculosis, and the table is filled in as follows:  

Latent Tuberculosis Test

+ −

+ 90 10

− 180 720

I-10.

The answer is D. (Chap. 3) A receiver operating characteristic (ROC) curve plots sensitivity (or true-positive rate) on the y-axis and 1 − specificity (or false-positive rate) on the x-axis. Each point on the curve represents a cutoff point of sensitivity and 1 − specificity, and these cutoff points are used to select the threshold value for a diagnostic test that yields the best trade-off between true-positive and false-positive tests. The area under the curve can be used as a quantitative measure of the information content of a test. Values range from 0.5 (a 45-degree line) representing no diagnostic information to 1.0 for an ideal test. In the medical literature, ROC curves are often used to compare alternative diagnostic tests, but the interpretation of a specific test and ROC curve is not as simple in clinical practice. One criticism of the ROC curve is that it only evaluates only one test parameter with exclusion of other potentially relevant clinical data. Also, one must consider the underlying population in which the ROC curve was validated and how generalizable this is the entire population with disease.

I-11.

The answer is C. (Chap. 3) The positive and negative predictive values of a test are strongly influenced by the prevalence of disease in a population. The positive predictive value is calculated as the number of true-positive test results divided by the number of all positive test values. Alternatively, the negative predictive value is calculated as the number of true-negative test results divided by the number of all negative test results. For example, in a population of 1000 with a disease prevalence of 5%, a specific test has a sensitivity of 95% and a specificity of 80%. In this setting, the two-by-two table would be completed as follows:

27

Disease

SECTION I

Test

+ −

+ 48 2

− 190 760

Introduction to Clinical Medicine

Thus, the positive predictive value of the test would be [48/(48 + 190)] × 100 = 20.2%, and the negative predictive value would be [760/(760 + 2)] × 100 = 99.7%. The sensitivity and specificity of the test, however, are properties of the test and are not affected by disease prevalence. Positive and negative likelihood ratios are calculated from the sensitivity and specificity and are defined as the ratio of the probability of a given test result (positive or negative) in an individual with disease to the probability of that test result in a patient without disease. For a positive likelihood ratio, a higher ratio indicates that a test performs better at identifying a patient with disease. For the negative likelihood ratio, a smaller ratio performs better at ruling out disease. The number needed to treat is a measure of the effectiveness of an intervention. It is simply calculated as 1 divided by the absolute reduction in risk related to the intervention. I-12 and I-13.  The answers are B and C, respectively. (Chap. 3) The goal of a meta-analysis is to summarize the treatment benefit conferred by an intervention by combining and summarizing data available from multiple clinical trials. Meta-analyses often focus on summary measures of relative risk reductions expressed by the relative risk or odds ratios; however, clinicians should also understand the absolute risk reduction (ARR) related to an intervention. This is the difference in mortality (or another endpoint) between the treatment and the placebo arms. In this case, the absolute risk reduction is 10% − 2% = 8%. From this number, one can calculate the number needed to treat (NNT), which is 1/ ARR. The NNT is the number of patients who must receive the intervention to prevent one death (or another outcome assessed in the study). In this case, the NNT is 1/8% = 12.5 patients.

28

I-14.

The answer is E. (Chap. 4) Within a population, it is certainly impractical to perform all possible screening procedures for the variety of diseases that exist in that population. This approach would be overwhelming to the medical community and would not be cost effective. Indeed, the amount of monetary and psychological stress that would occur from pursuing false-positive test results would add an additional burden on the population. When determining which procedures should be considered as screening tests, a variety of endpoints can be used. One of these is to determine how many individuals would need to be screened in the population to prevent or alter the outcome in one individual with disease. Although this can be statistically determined, there are no recommendations for what the threshold value should be and may change based on the invasiveness or cost of the test and the potential outcome avoided. Additionally, one should consider both the absolute and relative impact of screening on disease outcome. Another measure used in considering the utility of screening tests is the cost per life year saved. Most measures are considered cost effective if they cost less than $30,000 to $50,000 per year of life saved. This measure is also sometimes adjusted for the quality of life as well and presented as quality-adjusted life years saved. A final measure that is used in determining the effectiveness of a screening test is the effect of the screening test on life expectancy of the entire population. When applying the test across the entire population, this number is surprisingly small, and a goal of about 1 month is desirable for a population-based screening strategy.

I-15.

The answer is C. (Chap. 4) Evaluating the utility of screening tests requires also understanding the potential biases that can exist when interpreting data from screening trials. One of the most difficult to ascertain but potentially the most confounding is lead time bias. Simply, lead time bias refers to the bias that occurs when one finds a tumor at an earlier clinical stage than would be expected from usual care but ultimately does not lead to an overall change in the outcome. In this case, the apparent difference in time to diagnosis and death likely represents lead time bias. To fully determine this, one would

The answer is E. (Chap. 4) The U.S. Preventive Services Task Force (USPSTF) is an independent panel of experts selected by the federal government to provide evidence-based guidelines for prevention and screening for disease. The panel typically consists of primary care providers from internal medicine, family medicine, pediatrics, and obstetrics and gynecology. The USPSTF provides guidelines on a variety of measures, including blood pressure, height, weight, cholesterol, Pap smears, mammography, colorectal cancer screening, and adult immunizations. However, the most recent review of the evidence by the USPSTF concluded that there was insufficient evidence to recommend screening for thyroid disease in adults. Notably, the USPSTF also recommends against screening for prostate cancer in men older than 75 years and states that there is insufficient evidence for screening among younger men.

I-17.

The answer is B. (Chap. 4) Predicted increases in life expectancy are average numbers that apply to populations, not individuals. Because we often do not understand the true nature of risk of disease, screening and lifestyle interventions usually benefit a small proportion of the total population. For screening tests, false-positive test results may also increase the risk of diagnostic tests. Although Pap smears increase life expectancy overall by only 2–3 months, for an individual at risk of cervical cancer, Pap smear screening may add many years to life. The average life expectancy increases resulting from mammography (1 month), PSA (2 weeks), and exercise (1–2 years) are less than from quitting smoking (3–5 years).

I-18.

The answer is B. (Chaps. 4 and 235) Current guidelines from the National Cholesterol Education Project Adult Treatment Panel III recommend screening in all adults older than 20 years old. The testing should include fasting total cholesterol, triglycerides, lowdensity lipoprotein cholesterol, and high-density lipoprotein cholesterol. The screening should be repeated every 5 years. All patients with type 1 diabetes should have lipids followed closely to decrease cardiovascular risk by combining the results of lipid screening with other risk factors to determine risk category and intensity of recommended treatment.

I-19.

The answer is B. (Chap. 5) Bioavailability is the amount of the drug that is available to the systemic circulation when administered by routes other than the intravenous route. In this setting, bioavailability may be much less than 100%. The primary factors affecting bioavailability are the amount of drug that is absorbed and metabolism of the drug before entering the systemic circulation (the first-pass effect). Oral itraconazole is the recommended treatment for mild blastomycosis, but a problem with use of this drug is its bioavailability, which is estimated at about 55%. Although oral itraconazole does not experience a significant first-pass effect, its absorption from the stomach can be quite variable under different conditions. A first important consideration is the drug preparation. Whereas the liquid formulation should be taken on an empty stomach, the capsule should be taken after a meal. Furthermore, having an acid pH improves bioavailability, and use of gastric acid suppressors such as H2 blockers or proton pump inhibitors should be avoided with itraconazole use. When acid suppressors cannot be withheld, it is recommended to coadminister itraconazole with a cola beverage, which has been shown to enhance absorption in some clinical trials. Oral contraceptive pills will not affect the bioavailability of itraconazole; however, azole antifungals (including itraconazole) inhibit CYP450 3A4 and may increase the serum levels of estrogens and progestins.

ANSWERS

I-16.

SECTION I

need to know outcome data for the entire trial. In the case of lead time bias, one would find that although the number of tumors diagnosed at early stages was increased, the overall mortality would be the same. The recently published Lung Cancer Screening Trial (N Engl J Med, August 4, 2011) showed that low-dose helical CT scan in high-risk patients was associated 20% reduction in risk of dying from lung cancer compared with chest x-ray. Although this was the first clinical trial to show a radiologic intervention reducing mortality from lung cancer, how these results will translate into clinical practice and cost effectiveness is still uncertain.

29

The answer is D. (Chap. 5) Aminoglycoside antibiotics (tobramycin, gentamicin, amikacin) are active against Pseudomonas aeruginosa and are recommended for treatment of exacerbations of cystic fibrosis in combination with a beta-lactam antibiotic. The volume of distribution is increased in cystic fibrosis, altering drug metabolism and often necessitating higher doses than are normally given. To ensure therapeutic concentrations of tobramycin, a peak level should be check about 30 minutes after completion of an infusion. To reduce the risk of nephrotoxicity, a trough level should be checked immediately before the administration of a dose to ensure that the drug has been adequately metabolized. To ensure that steady-state concentration has been achieved, it is recommended that these levels be checked after three to five doses.

I-21.

The answer is A. (Chap. 5) Digitalis is a cardiac glycoside that exerts its effect via reversible inhibition of the sodium–potassium–ATPase pump. The cellular effect of this inhibition is to increase intracellular sodium and decrease extracellular potassium. The increase in intracellular sodium leads to a change in the membrane potential of the cell and an influx of calcium. This influx of calcium improves inotropy of the heart and leads to increased vagal tone with resultant decrease in heart rate through action at the sinoatrial and atrioventricular nodes. Digoxin is a drug with a narrow therapeutic window, meaning that the effective dose and the toxic dose are close to one another. Digoxin is a substrate for P-glycoprotein, which is an efflux pump that excretes drugs into the proximal tubule of the kidney. Caution must be taken when introducing a new medication that is an inhibitor of P-glycoprotein because these drugs can increase the serum concentration of digoxin. Examples of inhibitors of P-glycoprotein include amiodarone, clarithromycin, verapamil, and diltiazem. In this patient, initiation of an oral amiodarone load in the face of the patient’s known renal insufficiency was sufficient to cause digoxin toxicity. The typical manifestations of digoxin toxicity in this patient with a subacute onset include lethargy, generalized weakness, and delirium. Gastrointestinal manifestations may be seen but are less pronounced that in acute overdoses. The cardiac manifestations of digoxin toxicity are of the greatest concern, and the electrocardiogram can demonstrate a wide range of abnormalities, including bradycardia, atrial tachyarrhythmias, atrioventricular block, and ventricular tachycardia or fibrillation. The ECG can evolve over time, so continuous cardiac monitoring is warranted. Electrolyte abnormalities are common, especially hyperkalemia caused by the effects on the sodium–potassium–ATPase pump. However, in chronic toxicity, hypokalemia can also be seen. Worsening renal function is also a frequent manifestation and is often the cause for the rise in digoxin levels. The therapeutic range of digoxin is between 0.8 and 2 ng/mL. However, the level may not correlate well with the development of toxicity. Levels greater than 10 ng/mL often require treatment with digoxin-specific antibody fragments (Fab). This patient has other indications for use of Fab fragments as well given the complete heart block on ECG. Thus, observation alone is not an appropriate choice in this patient. Fab fragments are highly effective in the management of cardiac arrhythmias and are given as a single intravenous dose. Given the large molecular weight of digoxin and large volume of distribution, neither hemodialysis nor hemoperfusion is effective in elimination of digoxin. There are case reports of combined use of Fab fragments and plasmapheresis in individuals with profound renal failure, but this is not a standard option.

I-22.

The answer is C. (Chap. 5) Some medications circulate in the plasma partially bound to plasma proteins. In this setting, only unbound (or free) drug can distribute to the sites of action to exert pharmacologic effects. Examples of medications that are bound to plasma proteins include phenytoin, warfarin, valproic acid, and amiodarone. Hypoalbuminemia can lead to increased free levels of drugs that are more highly protein bound and can lead to drug toxicity at total drug levels that are not typically considered toxic. In this case, the patient has evidence of worsening liver disease with a low albumin level that has lead to signs and symptoms of phenytoin toxicity. A free drug level should be checked to confirm this. Although phenytoin can be used safely in those with mild liver disease, it should be discontinued in individuals with evidence of cirrhosis, which this patient clearly exhibits. Signs and symptoms of phenytoin toxicity include slurred speech, horizontal nystagmus, and altered mental status that can progress to obtundation and coma. Typically, severe phenytoin toxicity is not encountered unless the total

SECTION I

I-20.

Introduction to Clinical Medicine 30

The answer is D. (Chap. 5; VD Cataldo et al: N Engl J Med 364:947, 2011.) In the past decades, increasing interest and research has occurred in the area of genetic variability with respect to drug effects, particularly in the area of cancer chemotherapy. Each individual tumor contains multiple mutations that exhibit different biologic advantages that promote proliferation of tumor cells and escape from immune attack by the host. As investigators have learned more about the function of these mutations, drug development has concurrently allowed specific therapies directed against a particular mutation. Some specific examples of chemotherapeutic successes with targeted chemotherapy include use of imatinib in chronic myelogenous leukemia and gastrointestinal stroma tumors. In non–small cell lung cancer (NSCLC; adenocarcinoma and squamous cell carcinoma), targeted chemotherapeutic agents have included small molecule epidermal growth factor receptor (EGFR) tyrosine kinase inhibitors, a monoclonal antibody against vascular endothelial growth factor, and a monoclonal antibody that binds to EGFR. Current research is continuing to define the most appropriate role of these agents in the treatment of NSCLC. Recently, the National Comprehensive Cancer Network recognized the EGFR tyrosine kinase inhibitor erlotinib as second- and third-line therapy in individuals who have advanced stage NSCLC with good performance status. However, in individuals with activating mutations of the EGFR, erlotinib monotherapy is recommended. The two most common mutations are deletions of exon 19 and an arginine for leucine substitution at position 858 in exon 21. In clinical trials, individuals with these mutations, treatment with erlotinib or gefitinib is associated with an initial response rate of 55–90%. Moreover, those with activating mutations of EGFR have improved progression-free survival when treated with erlotinib or gefitinib. On the other hand, those without these mutations have been shown to do worse with these medications. Therefore, it is important to perform testing for mutations of EGFR before using either of these medications. Other clinical predictors of response to the EGFR tyrosine kinase inhibitors are female sex, lack of tobacco use, adenocarcinoma by pathology, and individuals of East Asian descent.

I-24.

The answer is D. (Chap. 5) Calcineurin inhibitors such as tacrolimus and cyclosporine are immunosuppressive agents that are used after solid organ transplants as well as for treatment of graft-versus-host disease (GVHD) in bone marrow transplant patients. These drugs are primarily metabolized via the cytochrome P450 pathway and excreted into bile. Many drugs and foods can be inhibitors or inducers of this pathway, and thoughtful consideration of possible drug interactions must be considered when starting any patient on a new medication while on tacrolimus or cyclosporine. In this case, voriconazole inhibits metabolism of tacrolimus, leading to increased serum concentrations of the drug. The clinical signs and symptoms of tacrolimus toxicity include hypertension, edema, headaches, insomnia, and tremor. In addition, elevated levels of tacrolimus can lead to worsening renal function and electrolyte abnormalities, including hyperkalemia, hypomagnesemia, hypophosphatemia, and hyperglycemia. It is recommended that the tacrolimus dose be decreased to one-third of the original dose when it is necessary to co-administer tacrolimus and voriconazole. Aspergillus meningitis is a rare infection that typically results from direct invasion from a rhinosinusitis. Congestive heart failure is unlikely in the clinical scenario because this is a young woman with no known heart disease and the neurologic symptoms are not consistent with that diagnosis. GVHD occurs when transplanted immune cells recognizes the host cells as foreign and initiates an immune response. GVHD occurs after allogeneic

ANSWERS

I-23.

SECTION I

phenytoin level is greater than 30 μg/mL. However, in the case of hypoalbuminemia, the total level can substantially misrepresent the free level of drug. When the free phenytoin level was checked in this case, it was elevated at 5 μg/mL (therapeutic range, 1.0–2.5 μg/ mL). Other less likely possibilities in the differential diagnosis of this patient include nonconvulsive status epilepticus and hepatic encephalopathy, but the presence of horizontal nystagmus is more suggestive of phenytoin toxicity. Infection is also a common cause of altered mental status in individuals with cirrhosis. However, the ascitic fluid does not support a diagnosis of spontaneous bacterial peritonitis that may be associated with a positive Gram stain of the ascites fluid. The history and physical examination are not consistent with a diagnosis of bacterial meningitis, and a head CT is not likely to provide additional information in this clinical setting.

31

SECTION I

hematopoietic stem cell transplants, and there is increased risk of GVHD in those with a greater disparity of human leukocyte antigens between the graft and the host. GVHD presents acutely with a diffuse maculopapular rash, fever, elevations in bilirubin and alkaline phosphatase, and diarrhea with abdominal cramping. There are case reports of nephritic syndrome related to GVHD, but renal involvement is not common. Also unlikely are neurologic symptoms, headache, hypertension, and tremor. Thrombotic thrombocytopenic purpura (TTP) could be considered in an individual with renal disease, altered mental status, and hypertension if there was concurrent evidence of an intravascular hemolytic process. However, TTP has not been associated with administration of voriconazole.

Introduction to Clinical Medicine 32

I-25.

The answer is E. (Chap. 5) Adverse drug reactions create significant morbidity in the treatment of disease. The most common classes of drugs that cause adverse events are antimicrobials, nonsteroidal anti-inflammatory drugs and aspirin, analgesics, anticoagulants, glucocorticoids, antineoplastics, diuretics, digoxin, and hypoglycemic agents. These drugs account for about 90% of all adverse drug events. Adverse drug events can be broadly classified as related or unrelated to the intended pharmacologic action. In this case, the patient has developed serum sickness (option E) after administration of benzathine penicillin. Serum sickness is an immunologic reaction to penicillin that is not a part of the intended pharmacologic action of the drug. Serum sickness is a type III immune complex mediated reaction that occurs when complex of drug and the appropriate antibody are deposited on endothelial cells. After the first exposure to the drug, it takes about 1–2 weeks for the immune reaction to occur, although with subsequent exposures, this would occur more quickly. Deposition of the immune complexes leads to complement activation with neutrophilic inflammation. Clinically, serum sickness presents as fever, urticarial rash, lymphadenopathy, inflammatory arthritis, and glomerulonephritis. Clinical recovery typically occurs in 7–28 days. Common pharmacologic causes of serum sickness are antibiotics and foreign proteins, including streptokinase, vaccines, and therapeutic antibodies. Secondary syphilis typically does not present until 4–10 weeks after primary infection. The rash typically is an erythematous maculopapular eruption that affects the palms and soles. Secondary syphilis should be adequately treated by the patient’s single dose of benzathine penicillin as long as the primary infection occurred with the past year. A Jarisch-Herxheimer reaction occurs when there is a systemic reaction to the killing of syphilis organisms. It begins in the first 24 hours after treatment and is associated with fevers, myalgias, headaches, and tachycardia. Disseminated gonococcal infection presents as an asymmetric migratory polyarthritis with fever and a papular or pustular rash. Septic arthritis may occur. A negative urethral swab result does not rule out this possibility, but the clinical presentation is not consistent with disseminated gonococcal infection. Approximately 10–20% of patients with rheumatoid arthritis have a negative rheumatoid factor. Although the disease most often presents with a symmetric inflammatory arthritis of the larger joints, the acute presentation of this patient makes this diagnosis less likely.

I-26.

The answer is A. (Chap. 5) In population surveys of noninstitutionalized elderly adults, up to 10% had at least one adverse drug reaction in the prior year. Adverse drug reactions are common in elderly adults and are related to altered drug sensitivity, impaired renal or hepatic clearance, impaired homeostatic mechanisms, and drug interactions. Long halflife benzodiazepines are linked to the increased occurrence of hip fractures in elderly adults. The association may be caused by the increased risk of falling (related to sedation) in a population with a high prevalence of osteoporosis. This association may also be true for other drugs with sedative properties such as opioids or antipsychotics. Exaggerated responses to cardiovascular drugs such as angiotensin-converting enzyme inhibitors may occur because of a blunted vasoconstrictor or chronotropic response to reduced blood pressure. Conversely, elderly patients often display decreased sensitivity to beta-blockers.

I-27.

The answer is C. (Chap. 5) Grapefruit juice inhibits CYP3A4 in the liver, particularly at high doses. This can cause decreased drug elimination via hepatic metabolism and increase potential drug toxicities. Atorvastatin is metabolized via this pathway. Drugs that may

The answer is E. (Chap. 6) The top two causes of death for men and women are the same—heart disease and cancer. These two broad categories of disease account for more than 50% of all deaths in men and 47% of deaths in women. Likewise, the number one cause of cancer death (lung cancer) is the same in men and women. After this, there are significant differences in the major causes of death between the sexes. Cerebrovascular disease is the third most common cause of death in women responsible for 6.7% of death, but in men, it is only the fifth most common cause of death with only 4.5% of all deaths. Although chronic lower respiratory disease is the fourth most common cause of death in both men and women, the percentage of deaths from chronic lower respiratory disease in women is 5.3% compared with 4.9% in men. Other diseases that are responsible for a greater percentage of deaths in women are Alzheimer’s disease, sepsis, pneumonia, and hypertension.

I-29.

The answer is C. (Chap. 6) Coronary heart disease (CHD) is the most common cause of death in men and women, but important sex differences exist in the presentation and treatment of CHD. At the time of presentation of CHD, women are about 10–15 years older than men with CHD. In addition, women have a greater number of medical comorbidities at the time of diagnosis, including hypertension, heart failure, and diabetes mellitus. Angina is the most common presenting symptom of CHD in women and may have atypical features, including nausea, indigestion, and upper back pain. Women who present with a myocardial infarction (MI) more often present with cardiogenic shock or cardiac arrest, but men have a greater risk of ventricular tachycardia on presentation with MI. In the past, women had a greater risk of death from MI when presenting at younger ages, but this gap has decreased in recent years. However, women are still referred less often by physicians for diagnostic and therapeutic cardiovascular procedures, and there are more false-positive and false-negative diagnostic test results in women. Women are also less likely to receive angioplasty, thrombolysis, coronary artery bypass grafting, aspirin, and beta-blockers. Despite this, the 5- and 10-year survival rates after coronary artery bypass grafting are the same for men and women.

I-30.

The answer is A. (Chap. 6) In general, the risk factors for coronary heart disease (CHD) are similar in men and women. However, an elevated total triglyceride level has been demonstrated to be an independent risk factor in women but not men. Low high-density lipoprotein and diabetes mellitus are also stronger risk factors in women, but they also influence CHD in men. Other shared risk factors include elevated total cholesterol, hypertension, obesity, smoking, and lack of physical activity.

I-31.

The answer is E. (Chap. 6) Sex differences exist in the prevalence of many common diseases. Hypertension is more common in women, particularly in those older than 60 years. In addition, most autoimmune diseases are more common in women, including rheumatoid arthritis, systemic lupus erythematosus, and autoimmune thyroid disease. Major depression is twice as common in women than men, and this is true even in developing countries. Other psychological disorders that are more common in women are eating disorders and anxiety. Endocrine disorders, including obesity and osteoporosis, are more common in women, and 80% of patients referred for bariatric surgery are women. However, the prevalence of both type 1 and type 2 diabetes mellitus is the same between men and women.

I-32.

The answer is C. (Chap. 6) Alzheimer’s disease (AD) affects women twice as commonly as men. This sex difference cannot fully be explained by the difference in life expectancy between men and women. The brains of women differ from men in terms of size, structure, and functional organization. In addition, it is thought that estrogen may play a role in the development of AD. Women with AD have lower levels of circulating estrogen

ANSWERS

I-28.

SECTION I

enhance atorvastatin toxicity via this mechanism include phenytoin, ritonavir, clarithromycin, and azole antifungals. Aspirin is cleared via renal mechanisms. Prevacid can cause impaired absorption of other drugs via its effect on gastric pH. Sildenafil is a phosphodiesterase inhibitor that may enhance the effect of nitrate medications and cause hypotension.

33

SECTION I Introduction to Clinical Medicine 34

than age-controlled women without disease. Although observational studies suggested a protective effect of estrogen replacement therapy on the development of AD, this was not borne out in randomized and blinded placebo-controlled trials. Indeed, the largest trial to date demonstrated an increase in dementia and mild cognitive impairment in individuals receiving either estrogen or combined hormone replacement therapy. I-33.

The answer is E. (Chap. 7) The cardiovascular system undergoes many changes in a pregnant woman to accommodate the needs of the developing fetus. Plasma volume begins to expand early in pregnancy and ultimately is increased by about 40–50% at term. Coincident with the increased plasma volume, cardiac output increases as well by about 40%. Although this is primarily attributable to increases in stroke volume, heart rate also increases in pregnancy by about 10 beats/min. In the second trimester, systemic vascular resistance falls, and subsequently blood pressure decreases as well. Thus, a blood pressure greater than 140/90 mmHg is considered abnormal and is associated with increased maternal and fetal morbidity and mortality.

I-34.

The answer is D. (Chap. 7) Chronic essential hypertension occurs in up to 5% of all pregnancies. Although hypertension is not a contraindication to pregnancy, the condition is associated with an increased risk of intrauterine growth restriction, preeclampsia, placental abruption, and perinatal mortality. The cardiovascular changes of pregnancy typically do lead to a fall in systemic vascular resistance and a fall in blood pressure in the second trimester, but it is not safe to discontinue medications in those with a prior diagnosis of hypertension if the blood pressure in the first trimester is greater than 120/80 mmHg. When choosing an antihypertensive medication in pregnancy, angiotensin-converting enzyme inhibitors and angiotensin receptor blockers should be strictly avoided because they are known to cause birth defects, including congenital malformations and intrauterine death, particularly in the second and third trimesters. The most common medications used in pregnancy are α-methyldopa, labetalol, and nifedipine. Although these medications have limited data from randomized controlled trials, there is a long history of safety with use of these medications. Diuretics such as hydrochlorothiazide appear also to be safe in pregnancy, although there are concerns that use of diuretics would impair the volume expansion that occurs during pregnancy.

I-35.

The answer is B. (Chaps. 7 and 150) Most cardiovascular conditions can be managed safely in pregnancy, although these pregnancies are often considered high risk. The conditions that are considered to be contraindications to pregnancy are idiopathic pulmonary arterial hypertension and Eisenmenger syndrome (congenital heart disease resulting in pulmonary hypertension with right-to-left shunting). In these cases, it is typically recommended to terminate the pregnancy because there is a high risk of maternal and fetal death. Peripartum cardiomyopathy can recur in subsequent pregnancies, and it is recommended that individuals with an abnormal ejection fraction avoid further pregnancies. Approximately 15% of individuals with Marfan syndrome will have a major cardiovascular complication in pregnancy, although the condition is not considered a contraindication to pregnancy. An aortic root diameter of less than 40 mm is generally associated with the best outcomes in pregnancy. The valvular heart disease with the greatest risk in pregnancy is mitral stenosis. There is an increased risk of pulmonary edema, and pulmonary hypertension is a common long-term consequence of mitral stenosis. However, aortic stenosis, aortic regurgitation, and mitral regurgitation are typically well tolerated. Congenital heart disease in the mother is associated with an increased risk of congenital heart disease in the offspring, but atrial and ventricular septal defects are usually well tolerated in pregnancy as long as there is no evidence of Eisenmenger syndrome.

I-36.

The answer is D. (Chap. 7) This patient has severe eclampsia, and delivery should be performed as rapidly as possible. Mild eclampsia is the presence of new-onset hypertension and proteinuria in a pregnant woman after 20 weeks’ gestation. Severe eclampsia is eclampsia complicated by central nervous system symptoms (including seizure), marked hypertension, severe proteinuria, renal failure, pulmonary edema, thrombocytopenia, or disseminated intravascular coagulation. Delivery in a mother

The answer is C. (Chap. 7) Pregnancy causes a hypercoagulable state, and deep venous thrombosis (DVT) occurs in about 1 in 2000 pregnancies. DVT occurs more commonly in the left leg than the right leg during pregnancy because of compression of the left iliac vein by the gravid uterus. In addition, pregnancy represents a procoagulant states with increases in factors V and VII and decreases in proteins C and S. Approximately 25% of pregnant women with DVT have a factor V Leiden mutation, which also predisposes to preeclampsia. Warfarin is strictly contraindicated because of a risk of fetal abnormality. Low-molecular-weight heparin (LMWH) is appropriate therapy at this point in pregnancy but is typically switched to unfractionated heparin 4 weeks before anticipated delivery because LMWH may be associated with an increased risk of epidural hematoma. Ambulation, rather than bedrest, should be encouraged as with all DVTs. There is no proven role for local thrombolytics or an inferior vena cava filter in pregnancy. The latter would be considered only when anticoagulation is not possible.

I-38.

The answer is E. (Chap. 7) Pregnancy complicated by diabetes is associated with greater maternal and perinatal morbidity and mortality rates and occurs in 4% of all pregnancies. Women with gestational diabetes are at increased risk of preeclampsia, delivering infants large for gestational age, and birth lacerations. Their infants are at risk of hypoglycemia and birth injury. Appropriate therapy can reduce these risks. All women should be screened for gestational diabetes unless they fall into a low-risk group. Low-risk groups include age younger than 25 years, body mass index less than 25 kg/m2, no maternal history of macrosomia or gestational diabetes, no diabetes in any first-degree relative, and those who are not members of a high-risk ethnic group (African American, Hispanic, or Native American).

I-39.

The answer is B. (Chap. 8) Medical providers are often asked to provide guidance regarding the postoperative risk of complications after a variety of noncardiac surgical procedures. When evaluating risk of complications, it is useful to categorize the surgical procedures into a low, intermediate, or higher risk category. Individuals who are at the highest risk of complications include those undergoing an emergent major operation, especially in elderly adults. Other higher risk procedures include aortic and other noncarotid major vascular surgery and surgeries with a prolonged operative time and large anticipated blood loss or fluid shifts (e.g., pancreaticoduodenectomy or Whipple procedure). Surgeries that are believed to be an intermediate risk include major thoracic surgery, major abdominal surgery, carotid endarterectomy, head and neck surgery, orthopedic surgery, and prostate surgery. Lower risk procedures include eye, skin, and superficial surgery as well as endoscopy.

I-40.

The answer is E. (Chap. 8) Poor exercise tolerance is an important predictor of postoperative complications. To standardize the determination of functional status, general guidelines are available that attempt to categorize the risk of complications according to functional status (Table I-40). The risk of postoperative complications increases when an individual cannot meet a metabolic equivalent (MET) level of 4. General activities that require a MET level of 4 include carrying 15–20 lb, playing golf, and playing doubles tennis. In addition, individuals experience increased risk of postoperative complications if they are unable to walk four blocks or climb two flights of stairs when walking at a normal pace. Individuals at highest risk of postoperative complications in relation to exercise capacity are those who have difficulty performing activities of daily living because of dyspnea, angina, or excessive fatigue.

ANSWERS

I-37.

SECTION I

with severe eclampsia before 37 weeks’ gestation decreases maternal morbidity but increases fetal risks of complications of prematurity. Aggressive management of blood pressure, usually with labetalol or hydralazine intravenously, decreases the maternal risk of stroke. However, similar to any hypertensive crisis, the decrease in blood pressure should be achieved slowly to avoid hypotension and risk of decreased blood flow to the fetus. Eclamptic seizures should be controlled with magnesium sulfate; it has been shown to be superior to phenytoin and diazepam in large randomized clinical trials.

35

TABLE I-40  Functional Status ↔

SECTION I

Higher Risk

Lower

•  Difficulty with adult activities of daily living •  Cannot walk four blocks or up two flights of stairs or unable to meet a MET level of 4 •  Inactive but no limitation •  Active: easily does vigorous tasks •  Performs regular vigorous exercises

Introduction to Clinical Medicine

Abbreviation: MET, metabolic equivalents. Source: From Fleisher LA et al: ACC/AHA guideline for perioperative cardiovascular evaluation for noncardiac surgery: A report of the American College of Cardiology/American Heart Association Task Force on Practice Guidelines (Writing Committee to Revise the 2002 Guidelines on Perioperative Cardiovascular Evaluation for Noncardiac Surgery). Circulation 116:1971, 2007.

I-41.

The answer is D. (Chap. 8) Cardiovascular events continue to be a major source of morbidity and mortality after surgical interventions, and preoperative prediction of those individuals at the highest risk of cardiovascular events has been an area of much research. Over the years, a variety of risk stratification tools have been developed to assist clinicians in determining which patients may benefit from preoperative noninvasive cardiac testing or initiation of preoperative preventive medical management. One of the simplest and most widely used is the revised cardiac risk index (RCRI), which scores patients on a scale from 0–6. The six factors that comprise the RCRI (Table I-41) are high-risk surgical procedures, known ischemic heart disease, congestive heart failure, cerebrovascular disease, diabetes mellitus requiring insulin, and chronic kidney disease with a creatinine greater than 2 mg/dL. An individual would be considered as

TABLE I-41  Revised Cardiac Risk Index Clinical Markers High-Risk Surgical Procedures Vascular surgery Major intraperitoneal or intrathoracic procedures Ischemic Heart Disease History of myocardial infarction Current angina considered to be ischemic Requiring sublingual nitroglycerin Positive exercise test Pathological Q waves on ECG History of PTCA or CABG with current angina considered to be ischemic Congestive Heart Failure Left ventricular failure by physical examination History of paroxysmal nocturnal dyspnea History of pulmonary edema S3 gallop on cardiac auscultation Bilateral rales on pulmonary auscultation Pulmonary edema on chest radiograph Cerebrovascular Disease History of transient ischemic attack History of cerebrovascular accident Diabetes Mellitus Treatment with insulin Chronic Renal Insufficiency Serum creatinine >2 mg/dL Abbreviations: CABG, coronary artery bypass grafting; ECG, electrocardiogram; PTCA, percutaneous transluminal coronary angioplasty. Source: Adapted from Lee TH et al: Derivation and prospective validation of a simple index for prediction of cardiac risk of major noncardiac surgery. Circulation 100:1043, 1999, with permission.

36

ANSWERS

I-42.

SECTION I

having ischemic heart disease if he or she had a history of angina, myocardial infarction, or prior angioplasty or bypass surgery. In addition, ischemic heart disease would be considered present if a patient was requiring sublingual nitroglycerin, had a positive exercise test result, or had pathologic Q waves on the electrocardiogram (ECG). Therefore, the patient has a RCRI score of 3 (high-risk surgery, ischemic heart disease by ECG, and diabetes mellitus requiring insulin). Individuals with scores of 3 or higher fall in the highest risk category of the RCRI. These individuals have an estimated postoperative risk of major cardiovascular events between 9 and 11%. If an individual has a risk score of 2, the risk of postoperative cardiovascular events is 4–6.6%. There is a 1% risk of postoperative events with a risk score of 1 and a 0.5% risk with a risk score of 0. No group has a risk score as high as 20% using this methodology. The answer is B. (Chap. 8) Pulmonary and cardiovascular complications are a major source of morbidity and mortality after surgery. Primary care physicians are often asked to determine a patient’s postoperative risk of pulmonary complications. Factors identified by the American College of Physicians as conferring an increased risk of pulmonary complications are shown in Table I-42. Although many of these factors are directly related to pulmonary function, some of these are not. Notably, the presence of congestive heart failure and a serum albumin level of less than 3.5 g/dL predict postoperative pulmonary complications. Asthma is not a predictor of pulmonary complications as long as the disease is under sufficient control. Factors listed in the table that are useful determinants of asthma control include peak expiratory flow rate greater than 100 L or 50% predicted and forced expiratory volume in 1 second of less than 2 L.

TABLE I-42  Predisposing Risk Factors for Pulmonary Complications 1. Upper respiratory tract infection: cough, dyspnea 2. Age greater than 60 years 3. COPD 4. American Society of Anesthesiologists Class ≥2 5. Functionally dependent 6. Congestive heart failure 7. Serum albumin 200%), but this test does not distinguish G-cell hyperfunction from ZES. The best test in this setting is the secretin stimulation test. An increase in gastrin levels greater than 200 pg within 15 minutes of administering 2 μg/kg of secretin by IV bolus has a sensitivity and specificity of greater than 90% for ZES. Endoscopic ultrasonography is useful in locating the gastrin-secreting tumor once the positive secretin test is obtained. Genetic testing for mutations in the gene that encodes the menin protein can detect the fraction of patients with gastrinomas that are a mani festation of multiple endocrine neoplasia type I (Wermer’s syndrome). Gastrinoma is the second most common tumor in this syndrome following parathyroid adenoma, but its peak incidence is generally in the third decade. VIII-9.  The answer is B. (Chap. 294) The patient presents with nonspecific gastrointestinal symptoms, but the presence of weight loss suggests malabsorption syndrome. Patients with lactose intolerance are usually able to relate symptoms to consumption of milkbased products and also report a strong history of crampy pain and flatulence. Therefore, a lactose-free diet is unlikely to be helpful. The patient does not have nocturnal diarrhea, which is commonly a feature of steatorrhea along with floating stools. In the absence of symptoms suggesting fat malabsorption, the first test should not be fecal fat measurement. As the patient has weight loss, irritable bowel syndrome is less likely, and an increase dietary fiber is unlikely to be useful. Finally, her symptoms may be consistent with celiac disease. The widespread availability of antibodies to gliadin, endomysial, and tTG can be easily measured in peripheral blood. Antiendomysial antibody has a 90–95% sensitivity and equal specificity, making it a reasonable first test in symptomatic individuals. The presence of the antibody is not diagnostic, however, and duodenal biopsy is recommended. Duodenal biopsy will show villous atrophy, absence or reduced height of villi, cuboidal appearance of surface epithelial cells, and increased lymphocytes and 357

SECTION VIII

plasma cells in the lamina propria. These changes regress with complete removal of gluten from the diet.

Disorders of the Gastrointestinal System

VIII-10.  The answer is B. (Chap. 294) Short bowel syndrome is a descriptive term referring to the many clinical complications that may occur after resection of varying lengths of the small bowel. Rarely, these complications may be due to congenital abnormalities of the small bowel. Most commonly in adults, short bowel syndrome occurs in mesenteric vascular disease, primary mucosal or submucosal disease (Crohn’s disease), and operations without preexisting small bowel disease such as trauma. Multiple factors contribute to diarrhea and steatorrhea including gastric acid hypersecretion, increased bile acids in the colon due to absent or decreased reabsorption in the small bowel, and lactose intolerance due to increased gastric acid secretion. Nonintestinal symptoms may include renal calcium oxalate calculi due to an increase in oxalate absorption by the large intestine with subsequent hyperoxaluria. This may be due to increased fatty acids in the colon that bind calcium, and thus calcium in the gut is not free to bind oxalate and free oxalate is absorbed in the large intestine. Increased bile acid pool size results in the generation of cholesterol gallstones from supersaturating in gallbladder bile. Gastric hypersecretion of acid is well described and thought to be due to loss of inhibition of gastric acid secretion because of absent short bowel to secrete inhibitory hormones. Coronary artery disease is not described as a complication of short bowel syndrome. VIII-11.  The answer is E. (Chap. 294) The patient presents with symptoms suggestive of Whipple’s disease with a chronic multisystem disease often including diarrhea/steatorrhea, migratory arthralgias, weight loss, and CNS or cardiac problems. Generally the presentation is of insidious onset, and dementia is a late finding and poor prognostic sign. The disease primarily occurs in middle-aged white males. The diagnosis requires small bowel biopsy and demonstration of PAS-positive macrophages within the small bowel. Small bacilli are often present and suggest the diagnosis of Whipple’s disease. Similar macrophages may be found in other affected organs, e.g., the CNS. Dilated lymphatics are present in patients with intestinal lymphangiectasia. Mononuclear cell infiltrate in the lamina propria is often demonstrated in patients with tropical sprue, and flat villi with crypt hyperplasia are the hallmark of celiac disease. VIII-12.  The answer is D. (Chap. 294) This patient has a stool osmolality gap (measured stool osmolality - calculated stool osmolality) of less than 50 mosmol/L, suggesting a secretory rather than an osmotic cause for diarrhea. Secretory causes of diarrhea include toxin-mediated diarrhea (cholera, enterotoxigenic Escherichia coli) and intestinal peptide–mediated diarrhea in which the major pathophysiology is a luminal or circulating secretagogue. The distinction between secretory diarrhea and osmotic diarrhea aids in forming a differential diagnosis. Secretory diarrhea will not decrease substantially during a fast and has a low osmolality gap. Osmotic diarrhea will generally decrease during a fast and has a high (>50 mosmol/L) osmolality gap. Celiac sprue, chronic pancreatitis, lactase deficiency, and Whipple’s disease all cause an osmotic diarrhea. VIII-13.  The answer is E. (Chaps. 294 and e37) Cobalamin malabsorption may occur due to disease at multiple anatomic sites extending from the stomach to the ileum. In the past, the Schilling test was utilized to assess cobalamin absorption, but this test is not currently commercially available. Cobalamin is primarily present in meat, but dietary deficiency is rare except in strict vegans. Dietary cobalamin is bound in the stomach to R-binder protein that is synthesized in salivary glands and stomach. The cobalamin–R binder complex requires an acid medium. Therefore, achlorhydria of any cause may result in the inability for splitting of cobalamin from food and binding to R-binder protein. Cobalamin absorption has an absolute requirement for intrinsic factor, which allows uptake by specific receptors in the ileum. Intrinsic factor is produced and released by gastric parietal cells. Thus pernicious anemia, the autoimmune atrophy of parietal cells, is a cause of cobalamin malabsorption. Pancreatic protease enzymes lyse the cobalamin–R binder protein complex to release cobalamin in the proximal intestine where it is bound to intrinsic factor for ileal absorption. Thus a deficiency of pancreatic enzymes, such as in chronic pancreatitis,

358

ANSWERS

VIII-14.  The answer is D. (Chap. 295) The incidence of inflammatory bowel disease is highly influenced by ethnicity, location, and environmental factors. Both conditions have their highest incidence in the United Kingdom and North America, and the peak incidence has a bimodal distribution of age of presentation: 15–30 years and 60–80 years. The incidence of both ulcerative colitis and Crohn’s disease is highest among persons of the Ashkenazi Jewish population. Prevalence decreases progressively in non-Jewish white, AfricanAmerican, Hispanic, and Asian populations. Cigarette smoking is associated with a decreased incidence of ulcerative colitis, but may cause Crohn’s disease. Oral contraceptive use is associated with a slightly higher incidence of Crohn’s disease, but not ulcerative colitis. Monozygotic twins are highly concordant for Crohn’s disease, but not ulcerative colitis.

SECTION VIII

can lead to cobalamin malabsorption. Finally, cobalamin-intrinsic factor is absorbed via an intact epithelium in the ileum. Inflammation (Crohn’s disease) or absence (surgical removal) of ileum will cause cobalamin malabsorption. The large intestine is not involved in cobalamin absorption; thus ulcerative colitis confined to the large intestine will not cause malabsorption.

VIII-15.  The answer is C. (Chap. 295) Chronic bloody diarrhea associated with weight loss and systemic symptoms in a young person is highly suggestive of inflammatory bowel disease. Her surgical findings suggest discontinuous lesions, which is typical of Crohn’s disease. Ulcerative colitis, in contrast, typically affects the rectum and proceeds caudally from there without normal mucosa until the area of inflammation terminates. The presence of strictures and fissures further supports the diagnosis of Crohn’s disease, as these are not features of ulcerative colitis. Microscopically, both ulcerative colitis and Crohn’s disease may have crypt abscess and, although Crohn’s disease is more often transmural, full thickness disease may be present in ulcerative colitis. The hallmark of Crohn’s disease is granulomas that may be present throughout the bowel wall and involve the lymph nodes, mesentery, peritoneum, liver, and pancreas. Although pathognomonic for Crohn’s disease, granulomas are only found in about half of surgical resections. Flat villi are not always present in either disease and are more commonly found in isolation with celiac disease. VIII-16.  The answer is D. (Chap. 295) There are a number of dermatologic manifestations of inflammatory bowel disease (IBD), and each type of IBD has a particular predilection for different dermatologic conditions. This patient has pyoderma gangrenosum. Pyoderma gangrenosum can occur in up to 12% of patients with ulcerative colitis and is characterized by a lesion that begins as a pustule and progresses concentrically to surrounding normal skin. The lesions ulcerate with violaceous, heaped margins and surrounding erythema. They are typically found on the lower extremities. Often the lesions are difficult to treat and respond poorly to colectomy; similarly, pyoderma gangrenosum is not prevented by colectomy. Treatment commonly includes IV antibiotics, glucocorticoids, dapsone, infliximab, and other immunomodulatory agents. Erythema nodosum is more common in Crohn’s disease, and attacks correlate with bowel symptoms. The lesions are typically multiple red hot, tender nodules measuring 1–5 cm and are found on the lower legs and arms. Psoriasis is more common in ulcerative colitis. Finally, pyoderma vegetans is a rare disorder in intertriginous areas reported to be a manifestation of inflammatory bowel disease in the skin. VIII-17.  The answer is D. [Chap. 295, Cochrane Database Syst Rev 2007 Oct 17; (4)] Despite being described as a clinical entity for over a century, the etiology of IBD remains cryptic. Current theory is related to an interplay between inflammatory stimuli in genetically predisposed individuals. Recent studies have identified a group of genes or polymorphisms that confer risk of IBD. Multiple microbiologic agents, including some that reside as “normal” flora, may initiate IBD by triggering an inflammatory response. Anaerobic organisms (e.g., Bacteroides and Clostridia spp.) may be responsible for the induction of inflammation. Other organisms, for unclear reasons, may have the opposite effect. These “probiotic” organisms include Lactobacillus spp., Bifidobacterium spp., Taenia suis, and Saccharomyces boulardii. Shigella, Escherichia, and Campylobacter spp. are known to promote inflammation. Studies of probiotic therapy in adults and children with IBD have shown potential benefit for reducing disease activity. 359

SECTION VIII

VIII-18.  The answer is D. (Chap. 295) Methotrexate, azathioprine, cyclosporine, tacrolimus, and anti-tumor necrosis factor (TNF) antibody are reasonable options for patients with CD, depending on the extent of macroscopic disease. Pneumonitis is a rare but serious complication of methotrexate therapy. Primary sclerosing cholangitis is an extraintestinal manifestation of inflammatory bowel disease (IBD). Pancreatitis is an uncommon complication of azathioprine, and IBD patients treated with azathioprine are at a fourfold increased risk of developing a lymphoma. Anti-TNF antibody therapy is associated with an increased risk of tuberculosis, disseminated histoplasmosis, and a number of other infections.

Disorders of the Gastrointestinal System

VIII-19.  The answer is C (Chap. 296) Irritable bowel syndrome is characterized by the following: recurrence of lower abdominal pain with altered bowel habits over a period of time without progressive deterioration, onset of symptoms during periods of stress or emotional upset, absence of other systemic symptoms such as fever and weight loss, and smallvolume stool without evidence of blood. Warning signs that the symptoms may be due to something other than irritable bowel syndrome include presentation for the first time in old age, progressive course from the time of onset, persistent diarrhea after a 48-hour fast, and presence of nocturnal diarrhea or steatorrheal stools. Each patient, except for patient C, has “warning” symptoms that should prompt further evaluation. VIII-20.  The answer is C. (Chap. 296) Although this patient has signs and symptoms consistent with IBS, the differential diagnosis is large. Few tests are required for patients who have typical IBS symptoms and no alarm features. In this patient, alarm features include anemia, an elevated ESR, and evidence of WBCs in the stool. Alarm features warrant further investigation to rule out other gastrointestinal disorders such as colonic pathology including diverticular disease or inflammatory bowel disease. In this case, colonoscopy to evaluate for luminal lesions and mucosal characteristics would be the logical first step. At this point, with the warning signs, empiric therapy for IBS is premature. Reassurance, stool bulking agents, and antidepressants are all therapies to consider if a patient does indeed have IBS. VIII-21.  The answer is D. (Chap. 296) Up to 80% of patients with irritable bowel syndrome (IBS) also have abnormal psychiatric features; however, no single psychiatric diagnosis predominates. The mechanism is not well understood but may involve altered pain thresholds. Although these patients are hypersensitive to colonic stimuli, this does not carry over to the peripheral nervous system. Functional brain imaging shows disparate activation in, for example, the mid-cingulate cortex, but brain anatomy does not discriminate IBS patients from those without IBS. An association between a history of sexual abuse and IBS has been reported. There is no reported association with sexually transmitted diseases. Patients with IBS do not have an increased risk of autoimmunity. VIII-22.  The answer is B. (Chap. 297) The patient presents with classic signs of diverticulitis with fever, abdominal pain that is usually left lower quadrant, anorexia or obstipation, and leukocytosis. This most commonly occurs in older individuals. Patients may present with acute abdomen due to perforation, though this occurs in less than 25% of cases. Plain radiographs of the abdomen are seldom helpful, but may rarely show the presence of an air–fluid level in the left lower quadrant indicating a giant diverticulum with impending perforation. CT with oral contrast is the diagnostic modality of choice with the following findings: sigmoid diverticula, thickened colonic wall greater than 4 mm, and inflammation within the pericolic space with or without the collection of contrast material or fluid. Abscesses, if present, will also be demonstrated on CT. Barium enema and colonoscopy should be avoided in acute diverticulitis because the insufflation of air or contrast material may lead to perforation. Although diverticular disease may result in hematochezia, these are generally not temporally linked to diverticulitis. VIII-23.  The answer is B. (Chap. 297) Medical management is appropriate for many patients with uncomplicated diverticular disease. Uncomplicated disease has fever, abdominal pain, leukocytosis, and anorexia/obstipation, while complicated disease includes that with abscess formation, perforation, strictures, or fistulae. Uncomplicated disease

360

ANSWERS

VIII-24.  The answer is B. (Chap. 297) Hemorrhoids can be internal or external; however, they are normally internal and may prolapse to the external position. Hemorrhoids are staged in the following manner: stage I, enlargement with bleeding; stage II, protrusion with spontaneous reduction; stage III, protrusion requiring manual reduction; stage IV, irreducible protrusion. Stage I, which this patient has, is treated with fiber supplementation, cortisone suppositories, and/or sclerotherapy. Stage II is treated with fiber and cortisone suppositories. Stage III patients are offered the prior three therapies and banding or operative hemorrhoidectomy. Stage IV patients benefit from fiber and cortisone therapy as well as operative hemorrhoidectomy. While substantial upper GI bleeding may result in hematochezia, the absence of suggestive signs/symptoms and the consistent findings of hemorrhoids do not indicate the need for upper endoscopy.

SECTION VIII

accounts for at least 75% of cases. Medical therapy generally involves bowel rest and antibiotics, usually trimethoprim/sulfamethoxazole or ciprofloxacin and metronidazole targeting aerobic gram-negative rods and anaerobic bacteria. Patients with more than two attacks of diverticulitis were previously thought to require surgical therapy, but newer data suggest that these patients do not have an increased risk of perforation and can continue medical management. Patients with immunosuppressive therapy, chronic renal failure, or collagen vascular disease have a fivefold higher risk of perforation during recurrent attacks. Surgical therapy is indicated for surgical low-risk patients with complicated disease.

VIII-25.  The answer is A. (Chap. 297) An anorectal abscess is an abnormal fluid-containing cavity in the anorectal region. Anorectal abscess results from an infection involving the glands surrounding the anorectal canal. The disease is more common in males with a peak incidence in the third to fifth decades. Patients with diabetes, inflammatory bowel disease, or who are immunocompromised are at increased risk for this condition. Perianal pain with defecation and fever are common presenting symptoms. VIII-26.  The answer is C. (Chap. 297) This patient has symptoms (social isolation), signs (foul odor), and risk factors (multiparity) for procidentia (rectal prolapse) and fecal incontinence. Procidentia is far more common in women than men and is often associated with pelvic floor disorders. It is not uncommon for these patients to become socially withdrawn and suffer from depression because of the associated fecal incontinence. The foul odor is a result of poor perianal hygiene due to the prolapsed rectum. Although depression in the elderly is an important medical problem, it is too premature in the evaluation of this patient to initiate medical therapy for depression. Occult malignancy and thyroid abnormalities may cause fecal incontinence and depression, but a physical examination would be diagnostic and avoid costly tests. Often patients are concerned that they have a rectal mass or carcinoma. Examination after an enema often makes the prolapse apparent. Medical therapy is limited to stool bulking agents or fiber. Surgical correction is the mainstay of therapy. VIII-27.  The answer is E. (Chap. 297) Surgical therapy is indicated in all low-risk surgical patients with complicated diverticular disease. Patients with at least two episodes of diverticulitis requiring hospitalization, with disease that does not respond to medical therapy, or who develop intra-abdominal complications are considered to have complicated disease. Complicating this patient’s relapse of diverticulitis is probably an enterovesicular fistula causing pneumaturia. Studies indicate that younger patients (3 cm) would also need to be considered carefully for cholecystectomy. Ursodeoxycholic acid can be used in some instances to dissolve gallstones. It acts to decrease the cholesterol saturation of bile and also allows the dispersion of cholesterol from stones by

372

ANSWERS

VIII-63.  The answer is D. (Chap. 311) A practitioner needs to have a high index of suspicion for acalculous cholecystitis in critically ill patients who develop decompensation during the course of treatment for the underlying disease and have no other apparent source of infection. Some predisposing conditions for the development of acalculous cholecystitis include serious trauma or burns, postpartum following prolonged labor, prolonged parenteral hyperalimentation, and the postoperative period following orthopedic and other major surgical procedures. The clinical manifestations of acalculous cholecystitis are identical to calculous disease, but the disease is more difficult to diagnose. Ultrasono­ graphy and CT scanning typically only show biliary sludge, but they may demonstrate large and tense gallbladders. Hepatobiliary scintigraphy often shows delayed or absent gallbladder emptying. Successful management relies on accurate and early diagnosis. In critically ill patients, a percutaneous cholecystostomy may be the safest immediate procedure to decompress an infected gallbladder. Once the patient is stabilized, early elective cholecystectomy should be considered. Metronidazole to provide anaerobic coverage should be added, but this would not elucidate or adequately treat the underlying condition.

SECTION VIII

producing a lamellar crystalline phase. It is only effective, however, in individuals with radiolucent stones measuring less than 10 mm.

VIII-64.  The answer is C. (Chap. 311) Gallstones are very common, particularly in Western countries. Cholesterol stones are responsible for 80% of cases of cholelithiasis; pigment stones account for the remaining 20%. Cholesterol is essentially water insoluble. Stone formation occurs in the setting of factors that upset cholesterol balance. Obesity, cholesterol-rich diets, high-calorie diets, and certain medications affect the biliary secretion of cholesterol. Intrinsic genetic mutations in certain populations may affect the processing and secretion of cholesterol in the liver. Pregnancy results in both an increase in cholesterol saturation during the third trimester and changes in gallbladder contractility. Pigment stones are increased in patients with chronic hemolysis, cirrhosis, Gilbert’s syndrome, and disruptions in the enterohepatic circulation. Although rapid weight loss and low-calorie diets are associated with gallstones, there is no evidence that a high-protein diet confers an added risk of cholelithiasis. VIII-65.  The answer is B. (Chaps. 42 and 311) The clinical presentation is consistent with a cholestatic picture. Painless jaundice always requires an extensive workup, as many of the underlying pathologies are ominous and early detection and intervention often offers the only hope for a good outcome. The gallbladder showed no evidence of stones and the patient shows no evidence of clinical cholecystitis, and so a hepatobiliary iminodiacetic acid (HIDA) scan is not indicated. Similarly, antibiotics are not necessary at this point. The cholestatic picture without significant elevation of the transaminases on the liver function tests makes acute hepatitis unlikely. Antimitochondrial antibodies are elevated in cases of primary biliary cirrhosis (PBC), which may present in a similar fashion. However, PBC is far more common in women than in men, and the average age of onset is the fifth or sixth decade. The lack of an obvious lesion on CT scan does not rule out a source of the cholestasis in the biliary tree. Malignant causes such as cholangiocarcinoma and tumor of the ampulla of Vater, and nonmalignant causes such as sclerosing cholangitis and Caroli’s disease may be detected only by direct visualization with endoscopic retrograde cholangiopancreatography (ERCP). ERCP is useful both diagnostically and therapeutically, as stenting procedures may be done to alleviate the obstruction. VIII-66.  The answer is A. (Chap. 313) The most common cause of acute pancreatitis in the United States is gallstones causing common bile duct obstruction. Although bile duct obstruction may be demonstrated on technetium HIDA scan, right upper quadrant ultrasound is preferred for ease, demonstration of gallstones in the gallbladder, and demonstration of obstructed bile duct. Alcohol is the second most common cause, followed by complications of endoscopic retrograde cholangiopancreatography (ERCP). Hypertriglyceridemia accounts for 1–4% of cases with triglyceride levels usually greater than 1000 mg/dL. Other potential causes of pancreatitis include trauma, postoperative states, drugs such as 373

SECTION VIII

v­ alproic acid, ­anti-HIV medications, estrogens, and sphincter of Oddi dysfunction. Additionally, there are a number of rare causes that have been described. The most judicious first step in evaluation is to test for gallstones and pursue more rare causes after the most common cause has been ruled out.

Disorders of the Gastrointestinal System

VIII-67.  The answer is A. (Chap. 313) Physical examination in acute pancreatitis commonly shows an uncomfortable patient often with low-grade fever, tachycardia, and hypotension. Abdominal tenderness and muscle rigidity are often present to varying degrees. Cullen’s sign is a faint blue discoloration around the umbilicus that may occur as the result of hemoperitoneum. Turner’s sign is blue-red-purple or green-brown discoloration of the flanks from tissue catabolism of hemoglobin. Both of these signs indicate the presence of severe necrotizing pancreatitis. VIII-68.  The answer is E. (Chap. 313) The BISAP (Bedside Index of Severity in Acute Pancreatitis) score has recently replaced Ranson’s criteria and APACHE II severity scores as the recommended modality to assess the severity of pancreatitis due to the cumbersome nature of the prior scores and the requirement of prior scores to collect large amounts of clinical and laboratory data over time. Furthermore, the APACHE II and Ranson’s scoring mechanisms did not have acceptable positive and negative predictive values in predicting severe acute pancreatitis. The BISAP score incorporates five variables in determining severity: BUN greater than 35 mg/dL, impaired mental status, presence of SIRS, age above 60 years, and pleural effusion on radiography. The presence of three or more of these factors is associated with substantially increased risk for in-hospital mortality. Additional risk factors initially predicting severity include BMI of 30 or above and comorbid disease. VIII-69.  The answer is E. (Chap. 313) Several trials over the last several decades have demonstrated that there is no role for prophylactic antibiotics in the management of either interstitial or necrotizing pancreatitis. Antibiotics are recommended for only patients who appear septic at presentation while awaiting the results of culture data. If cultures are negative, antibiotics should be discontinued to decrease the risk of the development of fungal superinfection. Similarly, several drugs have been evaluated in the treatment of acute pancreatitis and found to be of no benefit. These drugs include H2 blockers, ­glucagon, protease inhibitors such as aprotinin, glucocorticoids, calcitonin, nonsteroidal anti-inflammatory drugs, and lexipafant, a platelet-activating factor inhibitor. A recent meta-analysis of somatostatin, octreotide, and the antiprotease gabexate mesylate in the therapy of acute pancreatitis suggested a reduced mortality rate but no change in complications with octreotide, and no effect on mortality but reduced pancreatic damage with gabexate. VIII-70.  The answer is C. (Chap. 313) Persistent inflammatory changes in the pancreas may remain for weeks to months after an episode of acute pancreatitis. Similarly, there may be prolonged elevation of amylase and lipase. In this regard, persistent changes on CT or persistent pancreatic enzyme elevation should not discourage clinicians from feeding hungry patients with acute pancreatitis. Although there had been prior concern that feeding patients with pancreatitis may exacerbate pancreatic inflammation, this has not been borne out. Similarly, enteral feeding with a nasojejunal tube in patients with acute pancreatitis has been demonstrated to have fewer infectious complications than feeding with total parenteral nutrition. Because of this, nasogastric feeding is the preferred method of nutritional support in acute pancreatitis. Enteral feeding also helps to maintain the integrity of the intestinal tract in acute pancreatitis. VIII-71.  The answer is D. (Chap. 313) The pathophysiology of acute pancreatitis evolves in three phases. During the initial phase, pancreatic injury leads to intrapancreatic activation of digestive enzymes with subsequent autodigestion and acinar cell injury. Acinar injury is primarily attributed to activation of zymogens (proenzymes), particularly trypsinogen, by lysosomal hydrolases. Once trypsinogen is converted to trypsin, the activated trypsin further perpetuates the process by activating other zymogens to further autodigestion.

374

ANSWERS

VIII-72.  The answer is D. (Chap. 313) Chronic pancreatitis is a common disorder in any patient population with relapsing acute pancreatitis, especially patients with alcohol dependence, pancreas divisum, and cystic fibrosis. The disorder is notable for both endocrine and exocrine dysfunction of the pancreas. Often diabetes ensues as a result of loss of islet cell function; though insulin-dependent, it is generally not as prone to diabetic ketoacidosis or coma as are other forms of diabetes mellitus. As pancreatic enzymes are essential to fat digestion, their absence leads to fat malabsorption and steatorrhea. In addition, the fat-soluble vitamins, A, D, E, and K, are not absorbed. Vitamin A deficiency can lead to neuropathy. Vitamin B12, or cobalamin, is often deficient. This deficiency is hypothesized to be due to excessive binding of cobalamin by cobalamin-binding proteins other than intrinsic factor that are normally digested by pancreatic enzymes. Replacement of pancreatic enzymes orally with meals will correct the vitamin deficiencies and steatorrhea. The incidence of pancreatic adenocarcinoma is increased in patients with chronic pancreatitis, with a 20-year cumulative incidence of 4%. Chronic abdominal pain is nearly ubiquitous in this disorder, and narcotic dependence is common. Niacin is a water-soluble vitamin, and absorption is not affected by pancreatic exocrine dysfunction.

SECTION VIII

The inflammation initiated by intrapancreatic activation of zymogens leads to the second phase of acute pancreatitis, with local production of chemokines that causes activation and sequestration of neutrophils in the pancreas. Experimental evidence suggests that neutrophilic inflammation can also cause further activation of trypsinogen, leading to a cascade of increasing acinar injury. The third phase of acute pancreatitis reflects the systemic processes that are caused by release of inflammatory cytokines and activated proenzymes into the systemic circulation. This process can lead to the systemic inflammatory response syndrome with acute respiratory distress syndrome, extensive third-spacing of fluids, and multiorgan failure.

VIII-73.  The answer is A. (Chap. 313) This patient likely has chronic pancreatitis related to longstanding alcohol use, which is the most common cause of chronic pancreatitis in adults in the United States. Chronic pancreatitis can develop in individuals who consume as little as 50 g of alcohol daily (equivalent to ~30–40 ounces of beer). The patient’s description of his loose stools is consistent with steatorrhea, and the recurrent bouts of abdominal pain are likely related to his pancreatitis. In most patients, abdominal pain is the most prominent symptom. However, up to 20% of individuals with chronic pancreatitis present with symptoms of maldigestion alone. The evaluation for chronic pancreatitis should allow one to characterize the pancreatitis as large- vs. small-duct disease. Large-duct disease is more common in men and is more likely to be associated with steatorrhea. In addition, large-duct disease is associated with the appearance of pancreatic calcifications and abnormal tests of pancreatic exocrine function. Women are more likely to have smallduct disease, with normal tests of pancreatic exocrine function and normal abdominal radiography. In small-duct disease, the progression to steatorrhea is rare, and the pain is responsive to treatment with pancreatic enzymes. The characteristic findings on CT and abdominal radiograph of this patient are characteristic of chronic pancreatitis, and no further workup should delay treatment with pancreatic enzymes. Treatment with pancreatic enzymes orally will improve maldigestion and lead to weight gain, but they are unlikely to fully resolve maldigestive symptoms. Narcotic dependence can frequently develop in individuals with chronic pancreatitis due to recurrent and severe bouts of pain. However, as this individual’s pain is mild, it is not necessary to prescribe narcotics at this point in time. An ERCP or magnetic resonance cholangiopancreatography (MRCP) may be considered to evaluate for a possible stricture that is amenable to therapy. However, sphincterotomy is a procedure performed via ERCP that may be useful in treating pain related to chronic pancreatitis and is not indicated in the patient. Angiography to assess for ischemic bowel disease is not indicated as the patient’s symptoms are not consistent with intestinal angina. Certainly, weight loss can occur in this setting, but the patient usually presents with complaints of abdominal pain after eating and pain that is out of proportion with the clinical examination. Prokinetic agents would likely only worsen the patient’s malabsorptive symptoms and are not indicated.

375

This page intentionally left blank

SECTION IX Rheumatology and Immunology

QUESTIONS DIRECTIONS: Choose the one best response to each question.

IX-1.  All of the following are key features of the innate immune system EXCEPT: A. Exclusively a feature of vertebrate animals. B. Important cells include macrophages and natural killer lymphocytes. C. Nonrecognition of benign foreign molecules or microbes. D. Recognition by germ line–encoded host molecules. E. Recognition of key microbe virulence factors but not recognition of self molecules. IX-2.  A 29-year-old male with episodic abdominal pain and stress-induced edema of the lips, the tongue, and occasionally the larynx is likely to have low functional or absolute levels of which of the following proteins? A. B. C. D. E.

C1 esterase inhibitor C5A (complement cascade) Cyclooxygenase IgE T-cell receptor, α chain

IX-3.  Which of the following statements best describes the function of proteins encoded by the human major histocompatibility complex (MHC) I and II genes? A. Activation of the complement system B. Binding to cell surface receptors on granulocytes and macrophages to initiate phagocytosis C. Nonspecific binding of antigen for presentation to T cells D. Specific antigen binding in response to B-cell activation to promote neutralization and precipitation

IX-4.  A 37-year-old man has recently been diagnosed with systemic hypertension. He is prescribed lisinopril as initial monotherapy. He takes this medication as prescribed for 3 days and on the third day notes that his right hand is swollen, mildly itchy, and tingling. Later that evening his lips become swollen and he has difficulty breathing. Which of the following statements accurately describes this condition? A. His symptoms are due to direct activation of mast cells by lisinopril. B. His symptoms are due to impaired bradykinin degradation by lisinopril. C. His symptoms are unlikely to recur if he is switched to enalapril. D. Peripheral blood analysis will show deficiency of C1 inhibitor. E. Plasma IgE levels are likely to be elevated. IX-5.  A 35-year-old female comes to the local health clinic for recurrent urticarial lesions that occasionally leave a residual discoloration for the last 6 months. She also has had arthralgias. The sedimentation rate now is 85 mm/h. The procedure most likely to yield the correct diagnosis in this case would be: A. B. C. D. E.

A battery of wheal-and-flare allergy skin tests Measurement of total serum IgE concentration Measurement of C1 esterase inhibitor activity Skin biopsy Patch testing

377

SECTION IX Rheumatology and Immunology

IX-6.  A 28-year-old woman seeks evaluation from her primary care doctor for recurrent episodes of hives and states that she is “allergic to cold weather.” She reports that for more than 10 years she has developed areas of hives when exposed to cold temperatures, usually on her arms and legs. She has not sought evaluation previously and states that over the past several years the occurrence of the hives has become more frequent. Other than cold exposure, she can identify no other triggers for the development of hives. She has no history of asthma or atopy. She denies food intolerance. Her only medication is oral contraceptive pills, which she has taken for 5 years. She lives in a singlefamily home that was built 2 years ago. On examination, she develops a linear wheal after being stroked along her forearm with a tongue depressor. Upon placing her hand in cold water, her hand becomes red and swollen. In addition, there are several areas with a wheal and flare reaction on the arm above the area of cold exposure. What is the next step in the management of this patient? A. Assess for the presence of antithyroglobulin and antimicrosomal antibodies. B. Check C1 inhibitor levels. C. Discontinue the oral contraceptive pills. D. Treat with cetirizine, 10 mg daily. E. Treat with cyproheptadine, 8 mg daily. IX-7.  A 23-year-old woman seeks evaluation for seasonal rhinitis. She reports that she develops symptoms yearly in the spring and fall. During this time, she develops rhinitis with postnasal drip and cough that disrupts her sleep. In addition, she will also have itchy and watery eyes. When the symptoms occur, she takes nonprescription loratadine, 10 mg daily, with significant improvement in her symptoms. What is the most likely allergen(s) that is/are causing this patient’s symptoms? A. B. C. D. E. F.

Grass Ragweed Trees A and B B and C All of the above

IX-8.  Which of the following autoantibodies is most likely to be present in a patient with systemic lupus erythematosus? A. B. C. D. E.

Anti-dsDNA Anti-RNP Anti-Ro Antiphospholipid Antiribosomal P

IX-9.  A 23-year-old woman is evaluated by her primary care physician because she is concerned that she may have systemic lupus erythematosus after hearing a public health announcement on the radio. She has no significant past medical history, and her only medication is occasional ibuprofen. She is not sexually active and works in a grocery store. She reports that she has had intermittent oral ulcers and right knee pain. Physical examination shows 378

no evidence of alopecia, skin rash, or joint swelling/ inflammation. Her blood work shows that she has a positive antinuclear antibody (ANA) at a titer of 1:40, but no other abnormalities. Which of the following statements is true? A. Four diagnostic criteria are required to be diagnosed with systemic lupus erythematosus; this patient has three. B. Four diagnostic criteria are required to be diagnosed with systemic lupus erythematosus; this patient has two. C. If a urinalysis shows proteinuria, she will meet the criteria for systemic lupus erythematosus. D. She meets the criteria for systemic lupus erythematosus because she has three criteria for the disease. E. The demonstration of a positive ANA alone is adequate to diagnose systemic lupus erythematosus. IX-10.  A 32-year-old woman with a long-standing diagnosis of systemic lupus erythematosus is evaluated by her rheumatologist as routine follow-up. A new cardiac murmur is heard and an echocardiogram is ordered. She is feeling well, and has no fevers, weight loss, or preexisting cardiac disease. A vegetation on the mitral valve is demonstrated. Which of the following statements is true? A. Blood cultures are unlikely to be positive. B. Glucocorticoid therapy has been proven to lead to improvement in this condition. C. Pericarditis is frequently present concomitantly. D. The lesion has a low risk of embolization. E. The patient has been surreptitiously using injection drugs. IX-11.  A 24-year-old woman is newly diagnosed with systemic lupus erythematosus. Which of the following organ system complications is she most likely to have over the course of her lifetime? A. B. C. D. E.

Cardiopulmonary Cutaneous Hematologic Musculoskeletal Renal

IX-12.  A 27-year-old female with systemic lupus erythematosus (SLE) is in remission; current treatment consists of azathioprine 75 mg/d and prednisone 5 mg/d. Last year she had a life-threatening exacerbation of her disease. She now strongly desires to become pregnant. Which of the following is the least appropriate action to take? A. Advise her that the risk of spontaneous abortion is high. B. Warn her that exacerbations can occur in the first trimester and in the postpartum period. C. Tell her it is unlikely that a newborn will have lupus. D. Advise her that fetal loss rates are higher if anticardiolipin antibodies are detected in her serum. E. Stop the prednisone just before she attempts to become pregnant.

IX-14.  A 25-year-old African-American woman was has been followed in SLE clinic since her diagnosis 6 months ago. At that time she had evidence of mild joint disease, photosensitivity, malar rash, positive ANA, and anti-dsDNA. Her renal function and urinalysis were normal. She has been maintained on acetaminophen and hydroxychloroquine. She comes to the emergency department after a recent outing to the beach with friends. Over the past 2 days she’s noticed a marked increase in her fatigue and morning stiffness. She also has red-tinged urine. Physical examination is notable for a skin rash in sun-exposed areas, and diffuse wrist, knee, and ankle synovial thickening. Her platelet count has fallen from normal values to 45,000 and she has new leukopenia. In addition, her serum creatinine is 2.5 and there are RBC casts on urine analysis. An emergent renal biopsy is consistent with active diffuse lupus nephritis. After receiving methylprednisolone 1 g IV for 3 days, all of the following are appropriate treatment regimens EXCEPT: A. B. C. D. E.

Prednisone 60 mg/d Prednisone 60 mg/d plus azathioprine Prednisone 60 mg/d plus cyclophosphamide Prednisone 60 mg/d plus mycophenolate mofetil Rituximab

IX-15.  A 27-year-old woman is admitted to the intensive care unit after delivery of a full-term infant 3 days prior. The patient was found to have right hemiparesis and a blue left hand. Physical examination is also notable for livedo

A. B. C. D. E.

Anticardiolipin antibody panel Antinuclear antibody Doppler examination of her left arm arterial tree Echocardiography MRI of her brain

QUESTIONS

A. Initiate cyclophosphamide, 500 mg/m2 body surface area IV, and plan to repeat monthly for 3–6 months. B. Initiate hemodialysis. C. Initiate high-dose steroid therapy (IV methyl­ prednisolone, 1000 mg daily for 3 doses, followed by oral prednisone, 1 mg/kg daily) and mycophenolate mofetil, 2 g daily. D. Initiate plasmapheresis. E. Withhold all therapy until renal biopsy is performed.

reticularis. Her laboratories were notable for a white blood cell count of 10.2/mL, hematocrit 35%, and platelet count of 13,000/mL. Her BUN is 36 mg/dL and her creatinine is 2.3 mg/dL. Although this pregnancy was uneventful, the three prior pregnancies resulted in early losses. A peripheral smear shows no evidence of schistocytes. Which of the following laboratory studies will best confirm the underlying etiology of her presentation?

SECTION IX

IX-13.  A 45-year-old African-American woman with systemic lupus erythematosus (SLE) presents to the emergency department with complaints of headache and fatigue. Her prior manifestations of SLE have been arthralgias, hemolytic anemia, malar rash, and mouth ulcers, and she is known to have high titers of antibodies to double-stranded DNA. She currently is taking prednisone, 5 mg daily, and hydroxychloroquine, 200 mg daily. On presentation, she is found to have a blood pressure of 190/110 mmHg with a heart rate of 98 beats/min. A urinalysis shows 25 red blood cells (RBCs) per high-power field with 2+ proteinuria. No RBC casts are identified. Her blood urea nitrogen is 88 mg/dL, and creatinine is 2.6 mg/dL (baseline 0.8 mg/dL). She has not previously had renal disease related to SLE and is not taking nonsteroidal anti-inflammatory drugs. She denies any recent illness, decreased oral intake, or diarrhea. What is the most appropriate next step in the management of this patient?

IX-16.  A 28-year-old woman comes to the emergency department complaining of 1 day of worsening right leg pain and swelling. She drove in a car for 8 hours returning from a hiking trip 2 days ago then noticed some pain in the leg. At first she thought it was due to exertion but it has worsened over the day. Her only past medical history is related to difficulty getting pregnant with 2 prior spontaneous abortions. Her physical examination is notable for normal vital signs and heart and lung examination. Her right leg is swollen from the mid-thigh down and is tender. Doppler studies demonstrate a large deep venous thrombosis in the femoral and ileac veins extending into the pelvis. Laboratory studies on admission prior to therapy show normal electrolytes, normal white blood cell (WBC) and platelet counts, normal prothrombin time, and an activated partial thromboplastin time 3× normal. Her pregnancy test is negative. Lowmolecular-weight heparin therapy is initiated in the emergency department. Subsequent therapy should include: A. B. C. D. E.

Rituximab 375 mg/m2 per week for 4 weeks Warfarin with INR goal of 2.0–3.0 for 3 months Warfarin with INR goal of 2.0–3.0 for 12 months Warfarin with INR goal of 2.5–3.5 for life Warfarin with an INR goal of 2.5–3.5 for 12 months followed by daily aspirin for life

IX-17.  Which of the following is the most frequent site of joint involvement in established rheumatoid arthritis (RA)? A. B. C. D. E.

Distal interphalangeal joint Hip Knee Spine Wrist

IX-18.  In patients with established rheumatoid arthritis, all of the following pulmonary radiographic findings may be explained by their rheumatologic condition EXCEPT: A. B. C. D. E.

Bilateral interstitial infiltrates Bronchiectasis Lobar infiltrate Solitary pulmonary nodule Unilateral pleural effusion

379

SECTION IX

IX-19.  Which of the following is the earliest plain radiographic finding of rheumatoid arthritis? A. B. C. D. E.

Juxtaarticular osteopenia No abnormality Soft-tissue swelling Subchondral erosions Symmetric joint space loss

Rheumatology and Immunology

IX-20.  Which of the following statements regarding rheumatoid arthritis is true? A. Africans and African Americans most commonly have the class II major histocompatibility complex allele HLA-DR4. B. Females are affected three times more often than are males, and this difference is maintained throughout life. C. The earliest lesion in rheumatoid arthritis is an increase in the number of synovial lining cells with microvascular injury. D. There is an association with the class II major histocompatibility complex allele HLA-B27. E. Titers of rheumatoid factor are not predictive of the severity of rheumatoid arthritis or its extraarticular manifestations. IX-21.  A 46-year-old woman presents to your clinic with multiple complaints. She describes fatigue and general malaise over 2–3 months. Her appetite has decreased. She thinks she has unintentionally lost approximately 5.5 kg. Lately, she notes pain and stiffness in her fingers on both hands that is worse in the morning and with repetitive movement. She has a grandmother and a sister who have rheumatoid arthritis, and she is very concerned that she now has it as well. Which of her complaints represents the most common manifestation of established rheumatoid arthritis? A. Fatigue and anorexia for more than 2 months with concomitant joint pain B. Morning joint stiffness lasting for more than 1 hour C. Pain in symmetric joints that is worsened with movement D. Positive family history with two relatives with RA E. Weight loss of more than 4.5 kg during period of active disease IX-22.  All of the following are characteristic extraarticular manifestations of rheumatoid arthritis EXCEPT: A. B. C. D. E.

Anemia Cutaneous vasculitis Pericarditis Secondary Sjögren’s syndrome Thrombocytopenia

IX-23.  All of the following agents have been shown to have disease-modifying antirheumatic drug (DMARD) efficacy in patients with rheumatoid arthritis EXCEPT:

380

A. B. C. D. E.

Infliximab Leflunomide Methotrexate Naproxen Rituximab

IX-24.  Which of the following is the most common clinical presentation of acute rheumatic fever (ARF)? A. B. C. D. E.

Carditis Chorea Erythema marginatum Polyarthritis Subcutaneous nodules

IX-25.  A 19-year-old recent immigrant from Ethiopia comes to your clinic to establish primary care. She currently feels well. Her past medical history is notable for a recent admission to the hospital for new-onset atrial fibrillation. As a child in Ethiopia, she developed an illness that caused uncontrolled flailing of her limbs and tongue lasting approximately 1 month. She also has had three episodes of migratory large-joint arthritis during her adolescence that resolved with pills that she received from the pharmacy. She is currently taking metoprolol and warfarin and has no known drug allergies. Physical examination reveals an irregularly irregular heart beat with normal blood pressure. Her point of maximal impulse (PMI) is most prominent at the midclavicular line and is normal in size. An early diastolic rumble and a 3/6 holosystolic murmur are heard at the apex. A soft early diastolic murmur is also heard at the left third intercostal space. You refer her to a cardiologist for evaluation of valve replacement and echocardiography. What other intervention might you consider at this time? A. B. C. D. E.

Glucocorticoids Daily aspirin Daily doxycycline Monthly penicillin G injections Penicillin G injections as needed for all sore throats

IX-26.  A patient with a diagnosis of scleroderma who has diffuse cutaneous involvement presents with malignant hypertension, oliguria, edema, hemolytic anemia, and renal failure. You make a diagnosis of scleroderma renal crisis (SRC). What is the recommended treatment? A. B. C. D. E.

Captopril Carvedilol Clonidine Diltiazem Nitroprusside

IX-27.  A 57-year-old woman with depression and chronic migraine headaches reports several years of dry mouth and dry eyes. Her primary complaint is that she can no longer eat her favorite crackers, though she does report photosensitivity and eye burning on further questioning. She has no other associated symptoms. Examination shows dry, erythematous, sticky oral mucosa. All of the following tests are likely to be positive in this patient EXCEPT:

La/SS-B antibody Ro/SS-A antibody Schirmer’s I test Scl-70 antibody Sialometry

IX-28.  Which of the following is the most common extra­ glandular manifestation of primary Sjögren’s syndrome? Arthralgias/arthritis Lymphoma Peripheral neuropathy Raynaud’s phenomenon Vasculitis

IX-29.  A 44-year-old woman presents for evaluation of dry eyes and mouth. She first noticed these symptoms more than 5 years ago and the symptoms have worsened over time. She describes her eyes as gritty feeling, as if there were sand in her eyes. Sometimes her eyes burn, and she states that it is difficult to be outside in bright sunlight. In addition, her mouth is quite dry. In her job, she is frequently asked to give business presentations and finds it increasingly difficult to complete a 30- to 60-minute presentation. She has water with her at all times. Although she reports good dental hygiene without any recent changes, her dentist has had to place fillings twice in the past 3 years for dental caries. Her only other past medical history is treated tuberculosis that she contracted while in the Peace Corps in Southeast Asia when in her 20s. She takes no medication regularly and does not smoke. Ocular examination reveals punctuate corneal ulcerations on Rose Bengal stain, and the Schirmer’s test shows greater than 5 mm of wetness after 5 minutes. Her oral mucosa is dry with thick mucous secretions, and the parotid glands are enlarged bilaterally. Laboratory examination reveals positive antibodies to Ro and La (SS-A and SS-B). In addition, her chemistries reveal a sodium of 142 meq/L, potassium 2.6 meq/L, chloride 115 meq/L, and bicarbonate of 15 meq/L. What is the most likely cause of the hypokalemia and acidemia in this patient? A. B. C. D. E.

Diarrhea Distal (type I) renal tubular acidosis Hypoaldosteronism Purging with underlying anorexia nervosa Renal compensation for chronic respiratory alkalosis

IX-30.  A patient with primary Sjögren’s syndrome that was diagnosed 6 years ago and treated with tear replacement for symptomatic relief notes continued parotid swelling for the last 3 months. She has also noted enlarging posterior cervical lymph nodes. Evaluation shows leukopenia and low C4 complement levels. What is the most likely diagnosis? A. B. C. D. E.

A. B. C. D. E.

10% 30% 50% 90% 100%

IX-32.  Which of the following is the most common extra­ articular manifestation of ankylosing spondylitis? A. B. C. D. E.

Anterior uveitis Aortic insufficiency Inflammatory bowel disease Pulmonary fibrosis Third-degree heart block

QUESTIONS

A. B. C. D. E.

IX-31.  The histocompatibility antigen HLA-B27 is present in what percentage of patients with ankylosing spondylitis?

SECTION IX

A. B. C. D. E.

IX-33.  A 25-year-old man sees his primary care physician for evaluation of low back pain. The pain is severe, is worse in the morning, and is relieved with exercise and is worse with rest; in particular, nighttime sleeping is difficult. He does feel quite stiff in the morning for at least 30 minutes. An MRI of his lower back is obtained and shows active inflammation in the sacroiliac joint. On further questioning, he reports a history of unilateral eye redness treated with corticosteroids about 2 years ago. A test for HLA-B27 is positive. Which of the following is first-line therapy for his condition? A. B. C. D. E.

Infliximab Naproxen Prednisone Rituximab Tramadol

IX-34.  A 27-year-old man is seen at his primary care physician’s office for evaluation of painful arthritis involving the right knee that is associated with finger welling diffusely. He is otherwise healthy, but does recall a severe bout of diarrheal illness about 3–4 weeks prior that spontaneously resolved. He takes no medications and reports rare marijuana use. On review of systems, he reports painful urination. Examination shows inflammatory arthritis of the right knee, dactylitis, and normal genitourinary examination. He is diagnosed with reactive arthritis. Which of the following is the most likely etiologic agent of his diarrhea? A. B. C. D. E.

Campylobacter jejuni Clostridium difficile Escherichia coli Helicobacter pylori Shigella flexneri

Amyloidosis Chronic pancreatitis HIV infection Lymphoma Secondary Sjögren’s syndrome

381

SECTION IX

IX-35.  A 28-year-old woman undergoes evaluation for weight loss and bloody diarrhea that is ultimately diagnosed as Crohn’s disease. She has been diagnosed with dactylitis and bilateral sacroiliitis within the past 6 months. She is scheduled to begin treatment with infliximab in 2 weeks for her Crohn’s disease. Which of the following statements is true regarding the effect of infliximab on her arthritis?

Rheumatology and Immunology

A. Although infliximab is likely to improve her arthritic symptoms, NSAIDs should be tried first. B. Although infliximab is very effective therapy for Crohn’s disease, it will have no effect on her arthritis. C. Her arthritis is unrelated to Crohn’s disease, and because of this she should undergo a thorough evaluation for infectious causes before undergoing immunosuppressive therapy. D. Infliximab is very effective therapy for this type of arthritis. E. None of the above. IX-36.  Which of the following statements regarding the arthritis of Whipple’s disease is true? A. Arthritis is a rare finding in Whipple’s disease. B. Joint manifestations are usually concurrent with gastrointestinal symptoms and malabsorption. C. Radiography frequently shows joint erosions. D. Synovial fluid examination is unlikely to show polymorphonuclear cells. E. None of the above. IX-37.  A 35-year-old man has severe ankylosing spondylitis that is unresponsive to NSAID therapy. Therapy with infliximab has been recommended and he is wondering about potential side effects. All of the following are common potential side effects from this medication EXCEPT: A. B. C. D. E.

Demyelinating disorders Disseminated tuberculosis Exacerbation of congestive heart failure Hypersensitivity pneumonitis Pancytopenia

has been appreciated. Occasionally she notes numbness in the fingers and toes. The patient complains of chronic pain and poor sleep quality that she feels is due to her pain. She previously was seen in the clinic for chronic headaches that were felt to be tension related. She has tried taking over-thecounter ibuprofen twice daily without relief of pain. She has no other medical problems. On physical examination, the patient appears comfortable. Her joints exhibit full range of motion without evidence of inflammatory arthritis. She does have pain with palpation at bilateral suboccipital muscle insertions, at C5, at the lateral epicondyle, in the upper outer quadrant of the buttock, at the medial fat pad of the knee proximal to the joint line, and unilaterally on the second right rib. The erythrocyte sedimentation rate is 12 seconds. Antinuclear antibodies are positive at a titer of 1:40 in a speckled pattern. The patient is HLA-B27 positive. Rheumatoid factor is negative. Radiograms of the cervical spine, hips, and elbows are normal. What is the most likely diagnosis? A. B. C. D. E.

Ankylosing spondylitis Disseminated gonococcal infection Fibromyalgia Rheumatoid arthritis Systemic lupus erythematosus

IX-40.  A 42-year-old male presents with complaints of a rash and joint pain. He first noticed the rash 6 months ago. It is primarily on the hands (Figure IX-40), the extensor surfaces of the elbows, and the knees, low back, and scalp. Although he complains of the appearance of these lesions, they do not itch or hurt. He has not been previously evaluated for them and has recently noticed changes in the nail beds. For the last 2 weeks, the patient has had increasingly severe pain in the distal joints of the hands and feet. His hands are so painful that he is having trouble writing and holding utensils. He denies fevers, weight loss, fatigue, cough, shortness of breath, or changes in bowel or bladder habits. Which of the following is the most likely diagnosis?

IX-38.  Which of the following definitions best fits the term enthesitis? A. Alteration of joint alignment so that articulating surfaces incompletely approximate each other B. Inflammation at the site of tendinous or ligamentous insertion into bone C. Inflammation of the periarticular membrane lining the joint capsule D. Inflammation of a saclike cavity near a joint that decreases friction E. A palpable vibratory or crackling sensation elicited with joint motion IX-39.  A 35-year-old female presents to her primary care doctor complaining of diffuse body and joint pain. When asked to describe which of her joints are most affected, she answers, “All of them.” There is no associated stiffness, redness, or swelling of the joints. No Raynaud’s phenomenon 382

FIGURE IX-40  (See color atlas)

A. B. C. D. E.

Arthritis associated with inflammatory bowel disease Gout Osteoarthritis Psoriatic arthritis Rheumatoid arthritis

A. B. C. D. E.

Cryoglobulinemic vasculitis Henoch-Schönlein purpura Polyarteritis nodosa associated with hepatitis B Serum sickness Granulomatosis with polyangiitis (Wegener’s)

A. B. C. D. E.

Churg-Strauss syndrome Henoch-Schönlein purpura Microscopic polyangiitis Ulcerative colitis Granulomatosis with polyangiitis (Wegener’s)

IX-43.  A 40-year-old male presents to the emergency department with 2 days of low-volume hemoptysis. He reports that he has been coughing up 2–5 tablespoons of blood each day. He reports mild chest pain, low-grade fevers, and weight loss. In addition, he has had about 1 year of severe upper respiratory symptoms including frequent epistaxis and purulent discharge treated with several courses of antibiotics. Aside from mild hyperlipidemia, he is otherwise healthy. His only medications are daily aspirin and lovastatin. On physical examination he has normal vital signs, and the upper airway is notable for saddle nose deformity and clear lungs. A CT of the chest shows multiple cavitating nodules, and urinalysis shows red blood cells. Which of the following tests offers the highest diagnostic yield to make the appropriate diagnosis? A. B. C. D. E.

Deep skin biopsy Percutaneous kidney biopsy Pulmonary angiogram Surgical lung biopsy Upper airway biopsy

IX-44.  An 84-year-old woman sees her primary care physician for evaluation of severe headaches. She noted these several weeks ago and they have been getting worse. Although she has not had any visual aura, she is concerned that she has been intermittently losing vision in her left eye for the last few days. She denies new weakness or numbness, but she does report jaw pain with eating. Her past medical history includes coronary artery disease requiring a bypass grafting 10 years prior, diabetes mellitus, hyperlipidemia, and mild depression. Full review of symptoms is notable for night sweats and mild low back pain that is particularly prominent in the morning. Which of the following is the next most appropriate step? A. B. C. D. E.

Aspirin 975 mg po daily Measurement of erythrocyte sedimentation rate Prednisone 60 mg daily Referral for temporal artery biopsy Referral for ultrasound of temporal artery

A. B. C. D. E.

Hepatitis B surface antigen Cytoplasmic ANCA Hepatitis C polymerase chain reaction (PCR) HIV antibody Rheumatoid factor

IX-46.  A 54-year-old man is admitted for persistent lower abdominal and groin pain that began 7 months previously. Two months before his present admission, he required exploratory laparoscopy for acute abdominal pain and presumed cholecystitis. This revealed necrotic omental tissue and pericholecystitis necessitating omentectomy and cholecystectomy. However, the pain continued unchanged. He currently describes it as periumbilical and radiating into his groin and legs. It becomes worse with eating. The patient has also had episodic severe testicular pain, bowel urgency, nausea, vomiting, and diuresis. He has lost approximately 22.7 kg over the preceding 6 months. His past medical history is significant for hypertension that has recently become difficult to control. Medications on admission include aspirin, hydrochlo­ rothiazide, hydromorphone, lansoprazole, metoprolol, and quinapril. On physical examination, the patient appears comfortable. His blood pressure is 170/100 mmHg, his heart rate is 88 beats/min, and he is afebrile. He has normal first and second heart sounds without murmurs, and an S4 is present. There are no carotid, renal, abdominal, or femoral bruits. His lungs are clear to auscultation. Bowel sounds are normal. Abdominal palpation demonstrates minimal diffuse tenderness without rebound or guarding. No masses are present, and the stool is negative for occult blood. During the examination, the patient develops Raynaud’s phenomenon in his right hand that persists for several minutes. His neurologic examination is intact. Admission laboratory studies reveal an erythrocyte sedimentation rate of 72 mm/h, a BUN of 17 mg/dL, and a creatinine of 0.8 mg/dL. The patient has no proteinuria or hematuria. Tests for antinuclear antibodies, anti–double-strandedDNA antibodies, and antineutrophil cytoplasmic antibodies are negative. Liver function tests are abnormal with an AST of 89 IU/L and an ALT of 112 IU/L. Hepatitis B surface antigen and e antigen are positive. Mesenteric angiography demonstrates small, beaded aneurysms of the superior and inferior mesenteric veins. What is the most likely diagnosis? A. B. C. D. E.

QUESTIONS

IX-42.  A 53-year-old man presents with a vasculitis syndrome. His cytoplasmic antineutrophil cytoplasmic antibodies (c-ANCA) is positive. Which of the following syndromes is he most likely to have?

IX-45.  A 54-year-old man is evaluated for cutaneous vasculitis and peripheral nephropathy. Because of concomitant renal dysfunction he undergoes kidney biopsy that shows glomerulonephritis. Cryoglobulins are demonstrated in the peripheral blood. Which of the following laboratory studies should be sent to determine the etiology?

SECTION IX

IX-41.  All of the following vasculitic syndromes are thought to be due to immune complex deposition EXCEPT:

Hepatocellular carcinoma Ischemic colitis Microscopic polyangiitis Mixed cryoglobulinemia Polyarteritis nodosa

383

SECTION IX Rheumatology and Immunology

IX-47.  An 18-year-old man is admitted to the hospital with acute onset of crushing substernal chest pain that began abruptly 30 minutes ago. He reports the pain radiating to his neck and right arm. He has otherwise been in good health. He currently plays trumpet in his high school marching band but does not participate regularly in aerobic activities. On physical examination, he is diaphoretic and tachypneic. His blood pressure is 100/48 mmHg and heart rate is 110 beats/min. His cardiovascular examination shows a regular rhythm but is tachycardic. A II/VI holosystolic murmur is heard best at the apex and radiates to the axilla. His lungs have bilateral rales at the bases. The electrocardiogram demonstrates 4 mm of ST elevation in the anterior leads. On further questioning regarding his past medical history, he recalls having been told that he was hospitalized for some problem with his heart when he was 2 years old. His mother, who accompanies him, reports that he received aspirin and γ-globulin as treatment. Since that time, he has required intermittent followup with echocardiography. What is the most likely cause of this patient’s acute coronary syndrome? A. Dissection of the aortic root and left coronary ostia B. Presence of a myocardial bridge overlying the left anterior descending artery C. Stenosis of a coronary artery aneurysm D. Vasospasm following cocaine ingestion E. Vasculitis involving the left anterior descending artery IX-48.  Which of the following is required for the diagnosis of Behçet’s disease? A. B. C. D. E.

Large-vessel vasculitis Pathergy test Recurrent oral ulceration Recurrent genital ulceration Uveitis

IX-49.  A 25-year-old female presents with a complaint of painful mouth ulcerations. She describes these lesions as shallow ulcers that last for 1–2 weeks. The ulcers have been appearing for the last 6 months. For the last 2 days, the patient has had a painful red eye. She has had no genital ulcerations, arthritis, skin rashes, or photosensitivity. On physical examination, the patient appears well developed and in no distress. She has a temperature of 37.6°C (99.7°F), heart rate of 86 beats/min, blood pressure of 126/72 mmHg, and respiratory rate of 16 breaths/min. Examination of the oral mucosa reveals two shallow ulcers with a yellow base on the buccal mucosa. The ophthalmologic examination is consistent with anterior uveitis. The cardiopulmonary examination is normal. She has no arthritis, but medially on the right thigh there is a palpable cord in the saphenous vein. Laboratory studies reveal an erythrocyte sedimentation rate of 68 seconds. White blood cell count is 10,230/μL with a differential of 68% polymorphonuclear cells, 28% lymphocytes, and 4% monocytes. The antinuclear antibody and anti-dsDNA antibody are negative. C3 is 89 mg/dL, and C4 is 24 mg/dL. What is the most likely diagnosis? 384

A. B. C. D. E.

Behçet’s syndrome Cicatricial pemphigoid Discoid lupus erythematosus Sjögren’s syndrome Systemic lupus erythematosus

IX-50.  What is the best initial treatment for the patient in question IX-49? A. B. C. D. E.

Colchicine Intralesional interferon α Systemic glucocorticoids and azathioprine Thalidomide Topical glucocorticoids including ophthalmic prednisolone

IX-51.  Relapsing polychondritis may be a primary disease or may be associated with other rheumatologic diseases. All of the following conditions are associated with relapsing polychondritis EXCEPT: A. B. C. D. E.

Myelodysplastic syndrome Primary biliary cirrhosis Scleroderma Spondyloarthritides Systemic lupus erythematosus

IX-52.  A 47-year-old man is evaluated for 1 year of recurrent episodes of bilateral ear swelling. The ear is painful during these events, and the right ear has become floppy. He is otherwise healthy and reports no illicit habits. He works in an office and his only sport is tennis. On examination, the left ear has a beefy red color, and the pinna is tender and swollen; the earlobe appears minimally swollen but is neither red nor tender. Which of the following is the most likely explanation for this finding? A. B. C. D. E.

Behçet’s syndrome Cogan’s syndrome Hemoglobinopathy Recurrent trauma Relapsing polychondritis

IX-53.  A 25-year-old African-American woman is evaluated for bilateral hilar lymphadenopathy found on a routine chest radiograph performed before a laparoscopic cholecystectomy. She undergoes mediastinoscopy, and multiple noncaseating granulomas are identified in her lymph nodes. All of the following may explain this finding EXCEPT: A. B. C. D. E. F.

Alveolar proteinosis Atypical mycobacteria Beryllium exposure Histoplasmosis Malignancy Sarcoidosis

IX-54.  A 34-year-old woman has a history of cutaneous sarcoidosis that has been managed with hydroxychloroquine for the last 5 years. After an episode of right flank pain and hematuria, she is diagnosed with renal calculus. Which of the following statements regarding her renal calculus is true?

A. B. C. D. E.

Etanercept Hydroxychloroquine Infliximab Methotrexate Prednisone

IX-56.  All of the following statements regarding the clinical manifestations of sarcoidosis are true EXCEPT: A. Cardiac involvement occurs in 25% of patients. B. Eye involvement is typically anterior uveitis. C. Liver involvement is typically manifest by elevation of alkaline phosphatase. D. Lung involvement occurs in over 90% of cases. E. Skin involvement occurs in approximately one-third of patients. IX-57.  You are seeing a 56-year-old woman for complaints of joint pain and stiffness. All of the following signs or symptoms would be indicative of inflammatory causes of arthritis EXCEPT: A. B. C. D. E.

Elevations in erythrocyte sedimentation rate Fatigue, fever, or weight loss Persistence for longer than 6 weeks Presence of soft-tissue swelling around affected joints Prolonged morning stiffness

IX-58.  A 22-year-old man is seen for a shoulder injury that occurred while pitching in a baseball game. He describes feeling a snap then acute pain in the shoulder of his left arm while throwing the ball. Which of the following findings would be most concerning for a tear of one of the rotator cuff muscles? A. Inability to hold the arm at 90° following passive abduction B. Inability to actively raise the arm more than 90° with forward flexion C. Pain with palpation over the bicipital groove while rotating the arm internally and externally D. Pain with palpation while applying pressure anteriorly along the joint and rotating the arm internally and externally E. Pain with passive abduction of the arm

A. B. C. D. E.

QUESTIONS

IX-55.  All of the following agents have been shown to improve symptoms or function in patients with sarcoidosis EXCEPT:

IX-59.  A 62-year-old white male presents with a chief complaint of right knee pain and swelling. Past medical history is significant for obesity with a body mass index (BMI) of 34 kg/m2, diet-controlled Type 2 diabetes mellitus, and hypertension. His medications include hydrochlorothiazide and acetaminophen as needed for pain. Physical examination is remarkable for a moderately sized effusion of the right knee, with range of motion limited to 90° of flexion and 160° of extension. There is minimal warmth and no redness. He has crepitus with range of motion. With weight bearing, he has outward bowing of the legs bilaterally. A radiogram of the right knee shows osteophytes and joint space narrowing. Which of the following is the most likely finding on joint fluid examination?

SECTION IX

A. Exogenous vitamin D and sunlight exposure in patients with sarcoidosis may exacerbate hypercalcemia and associated renal calculus. B. Hypercalcemia is rare in sarcoidosis and is unlikely to contribute to the patient’s calculus. C. Hypercalcemia in sarcoidosis occurs through increased production of 25-dihydroxyvitamin D by the skin. D. If she is to begin therapy with oral calcium to treat the renal stone, a 24-hour urine phosphate should be obtained before and after initiation of therapy. E. None of the above.

A Gram stain showing gram-positive cocci in clusters A white blood cell count of 1110/μL A white blood cell count of 22,000/μL Positively birefringent crystals on polarizing light microscopy Negatively birefringent crystals on polarizing light microscopy

IX-60.  A 62-year-old woman presents complaining of hand pain bilaterally that has been gradually progressive over the past year. She has previously worked as a seamstress in a factory making gloves for more than 35 years. You suspect osteoarthritis. All of the following factors on history or physical examination are characteristic of this diagnosis EXCEPT: A. Evidence of bilateral swelling and warmth affecting the wrists only B. Joint space narrowing and osteophytes at the proximal and distal interphalangeal joints on x-ray C. Pain that becomes worse when preparing meals D. Presence of Heberden’s nodes E. Stiffness that is worse after brief periods of rest with occasional locking of the more affecting joints IX-61.  A 73-year-old woman with a medical history of obesity and diabetes mellitus presents to your clinic complaining of right knee pain that has been progressive and is worse with walking or standing. She has taken over-thecounter nonsteroidal anti-inflammatory drugs without relief. She wants to know what is wrong with her knee and what may have caused it. X-rays are performed and reveal cartilage loss and osteophyte formation. Which of the following represents the most potent risk factor for the development of osteoarthritis? A. B. C. D. E.

Age Gender Genetic susceptibility Obesity Previous joint injury

385

SECTION IX Rheumatology and Immunology

IX-62.  A 53-year-old man presents to your clinic complaining of bilateral knee pain. He states that the pain worsens with walking and is not present at rest. He has been experiencing knee pain intermittently for many months and has had no relief from over-the-counter analgesics. He has a history of hypertension and obesity. When he was in high school and college, he played football and basketball. Which of the following represents the best initial treatment strategy for this patient? A. B. C. D. E.

Avoidance of walking for several weeks Light daily walking exercises Low-dose, long-acting narcotics Oral steroid pulse Weight loss

IX-63.  A 74-year-old man is seen by his primary care provider 6 weeks following an acute gout attack. He has a prior history of gout presenting similarly on two prior occasions within the past 6 months. His past medical history is significant for congestive heart failure, hypercholesterolemia, and stage III chronic kidney disease. He is taking pravastatin, aspirin, furosemide, metolazone, lisinopril, and metoprolol XL. His glomerular filtration rate is 38 mL/min, creatinine is 2.2 mg/dL, and uric acid level is 9.3 mg/dL. He is wondering if there is any therapy that might lessen his likelihood of repeated gout attacks. Which of the following medication regimens is most appropriate for the treatment of this patient? A. B. C. D. E.

Allopurinol 800 mg daily Colchicine 0.6 mg bid Febuxostat 40 mg daily Indomethacin 25 mg twice daily Probenecid 250 mg twice daily

IX-64.  A 64-year-old man with congestive heart failure presents to the emergency department complaining of acute onset of severe pain in his right foot. The pain began during the night and awoke him from a deep sleep. He reports the pain to be so severe that he could not wear a shoe or sock to the hospital. His current medications are furosemide 40 mg twice daily, carvedilol 6.25 mg twice daily, candesartan 8 mg once daily, and aspirin 325 mg once daily. On examination, he is febrile to 38.5°C (101.3°F). The first toe of the right foot is erythematous and exquisitely tender to touch. There is significant swelling and effusion of the first metatarsophalangeal joint on the right foot. No other joints are affected. Which of the following findings would be expected on arthrocentesis? A. Glucose level of less than 25 mg/dL B. Positive Gram stain C. Presence of strongly negatively birefringent needleshaped crystals under polarized light microscopy D. Presence of weakly positively birefringent rhomboidal crystals under polarized light microscopy E. White blood cell (WBC) count greater than 100,000/μL IX-65.  A 24-year-old woman is admitted to the hospital with symptoms of fever and a swollen, painful right knee. About 3 weeks prior to the current syndrome, the patient had 386

systemic symptoms including fevers, chills, and migratory joint pains affecting the hands, wrists, knees, hips, and ankles. At that time, she noticed a few small papules on her upper chest and hands. These have subsequently resolved. She has no significant past medical history. She currently works as a landscape designer and does not recall any recent tick or insect bites. Her only medication is an oral contraceptive. She is unmarried and has multiple sexual partners. On physical examination, the patient has a temperature of 38.4°C (101.2°F), heart rate of 124 beats/min, respiratory rate of 24 breaths/min, and blood pressure of 102/68 mmHg. Her right knee demonstrates redness, warmth, swelling, and pain with movement. An arthrocentesis demonstrates a white blood cell count of 66,000/μL (90% neutrophils). No crystals or organisms are seen. Which of the following would be most likely to yield the correct diagnosis? A. B. C. D. E.

Bacterial cultures of the cervix Bacterial cultures of the synovial fluid Blood cultures IgG directed against Borrelia burgdorferi Rheumatoid factor

IX-66.  A 66-year-old woman with a history of rheumatoid arthritis and frequent attacks of pseudogout in her left knee presents with night sweats and a 2-day history of left knee pain. Her medications include methotrexate 15 mg weekly, folate 1 mg daily, prednisone 5 mg daily, and ibuprofen 800 mg three times daily as needed for pain. On physical examination, her temperature is 38.6°C (101.5°F), heart rate is 110 beats/min, blood pressure is 104/78 mmHg, and oxygen saturation is 97% on room air. Her left knee is swollen, red, painful, and warm. With 5° of flexion or extension, she develops extreme pain. She has evidence of chronic joint deformity in her hands, knees, and spine. Peripheral white blood cell (WBC) count is 16,700 cells/μL with 95% neutrophils. A diagnostic tap of her left knee reveals 168,300 WBCs per μL and 99% neutrophils, and diffuse needle-shaped birefringent crystals are present. Gram stain shows rare gram-positive cocci in clusters. Management includes all of the following EXCEPT: A. B. C. D. E.

Blood cultures Glucocorticoids Needle aspiration of joint fluid Orthopedic surgery consult Vancomycin

IX-67.  A 42-year-old woman is seen in her primary care doctor’s office complaining of diffuse pains and fatigue. She has a difficult time localizing the pain to any particular joint or location, but reports that it affects her upper and lower extremities, neck, and hips. It is described as achy and 10 out of 10 in intensity. She feels that her joints are stiff but does not notice that it is worse in the morning. The pain has been present for the past 6 months and is increasing in intensity. She has tried both over-the-counter ibuprofen and acetaminophen without significant relief. The patient feels as if the pain is interfering with her ability to get restful sleep and is making it difficult for her

IX-68.  A 36-year-old woman presents to your office with diffuse pain throughout her body associated with fatigue, insomnia, and difficulty concentrating. She finds the pain difficult to localize, but reports that it is 7–8 out of 10 in intensity and is not relieved by nonsteroidal anti-inflammatory medications. She has a long-standing history of generalized anxiety disorder and is treated with sertraline 100 mg daily as well as clonazepam 1 mg twice daily. On examination, she has pain with palpation at several musculoskeletal sites. Her laboratory examination demonstrates a normal complete blood count, basic metabolic panel, erythrocyte sedimentation rate, and rheumatoid factor. You diagnose her with fibromyalgia. All of the following therapies are recommended as part of the treatment plan for fibromyalgia EXCEPT: A. An exercise program that includes strength training, aerobic exercise, and yoga B. Cognitive-behavioral therapy for insomnia C. Milnacipran D. Oxycodone E. Pregabalin IX-69.  A 53-year-old woman presents to your clinic complaining of fatigue and generalized pain that have worsened over 2 years. She also describes irritability and poor sleep, and is concerned that she is depressed. She reveals that she was recently separated from her husband and has been stressed at work. Which of the following elements in her history and physical examination would meet American College of Rheumatology criteria for diagnosis of fibromyalgia?

IX-70.  A 42-year-old man is found to have the following finding on a physical examination (Figure IX-70). All of the following conditions are associated with this finding EXCEPT:

QUESTIONS

A. Cognitive dysfunction, sleep disturbance, anxiety, and depression are common comorbid neuropsychological conditions. B. Pain in this syndrome is associated with increased evoked pain sensitivity. C. Pain in this syndrome is often localized to specific joints. D. This syndrome is present in 2–5% of the general population, but increases in prevalence to 20% or more of patients with degenerative or inflammatory rheumatic disorders. E. Women are nine times more likely than men to be affected by this syndrome.

A. Diffuse chronic pain and abnormal sleep B. Diffuse pain without other etiology and evidence of major depression C. Major depression, life stressor, chronic pain, and female gender D. Major depression and pain on palpation at 6 of 18 tender point sites E. Widespread chronic pain and pain on palpation at 11 of 18 tender point sites

SECTION IX

to concentrate. She has missed multiple days of work as a waitress and fears that she will lose her job. There is a medical history of depression and obesity. The patient currently is taking venlafaxine sustained release 150 mg daily. She has a family history of rheumatoid arthritis in her mother. She smokes 1 pack of cigarettes daily. On physical examination vital signs are normal. Body mass index is 36 kg/m2. Joint examination demonstrates no erythema, swelling, or effusions. There is diffuse pain with palpation at the insertion points of the suboccipital muscles, at the midpoint of the upper border of the trapezius muscle, along the second costochondral junction, at the lateral epicondyles, and along the medial fat pad of the knees. All of the following statements regarding the cause of this patient’s diffuse pain syndrome are true EXCEPT:

FIGURE IX-70  (Reprinted from the Clinical Slide Collection on the Rheumatic Diseases, Copyright 1991, 1995. Used by permission of the American College of Rheumatology.)

A. B. C. D. E.

Chronic obstructive pulmonary disease Cyanotic congenital heart disease Cystic fibrosis Hepatocellular carcinoma Hyperthyroidism

IX-71.  A 64-year-old woman sees her primary care physician complaining of hip pain for about 1 week. She localizes the pain to the lateral aspect of her right hip and describes it as sharp. It is worse with movement, and she finds it difficult to lie on her right side. The pain began soon after the patient planted her garden. She has a medical history of obesity, osteoarthritis of the knees, and hypertension. Her medications include losartan 50 mg daily and hydrochlorothiazide 25 mg daily. For the pain, she has taken ibuprofen 600 mg as needed with mild to moderate relief of pain. On physical examination, the patient is not febrile and her vital signs are unremarkable. On examination of the hip, pain is elicited with external rotation and resisted abduction of the hip. Direct palpation over the lateral aspect of the upper portion of the femur near the hip joint reproduces the pain. What is the most likely diagnosis in this patient? A. B. C. D. E.

Avascular necrosis of the hip Iliotibial band syndrome Meralgia paresthetica Septic arthritis Trochanteric bursitis 387

SECTION IX Rheumatology and Immunology

IX-72.  A 32-year-old woman is seen in the clinic with a complaint of left knee pain. She enjoys running long distances and is currently training for a marathon. She is running on average 30–40 miles weekly. She currently is experiencing an aching pain on the lateral aspect of her left knee. There is a burning sensation that also continues up the lateral aspect of her thigh. She denies any injury to her knee, and she has not felt that it was hot or swollen. She is otherwise healthy and takes no medications other than herbal supplements. Physical examination of the knee reveals point tenderness over the lateral femoral condyle that is worse with flexing the knee. The patient is asked to lie on her right side with her right knee and hip flexed at 90°. Her left leg is extended at the hip and slowly lowered into adduction behind the bottom leg, reproducing the patient’s left knee pain. All of the following treatments can be recommended for this patient EXCEPT: A. Assessment of the patient’s running shoes to ensure a proper fit B. Glucocorticoid injection so as not to interfere with the patient’s continued preparation for the upcoming marathon C. Ibuprofen 600–800 mg every 6 hours as needed for pain D. Referral for physical therapy E. Referral for surgical release if conservative therapy fails IX-73.  A 58-year-old female presents complaining of right shoulder pain. She does not recall any prior injury but notes that the shoulder has been getting progressively stiffer over the last several months. She previously had

several episodes of bursitis of the right shoulder that were treated successfully with NSAIDs and steroid injections. The patient’s past medical history is also significant for diabetes mellitus, for which she takes metformin and glyburide. On physical examination, the right shoulder is not warm or red but is tender to touch. Passive and active range of motion is limited in flexion, extension, and abduction. A right shoulder radiogram shows osteopenia without evidence of joint erosion or osteophytes. What is the most likely diagnosis? A. B. C. D. E.

Adhesive capsulitis Avascular necrosis Bicipital tendinitis Osteoarthritis Rotator cuff tear

IX-74.  A 32-year-old woman presents to the clinic with right thumb and wrist pain that has worsened over several weeks. She has pain when she pinches her thumb against her other fingers. Her only other history is that she is a new mother with an 8-week-old infant at home. On physical examination she has mild swelling and tenderness over the radial styloid process, and pain is elicited when she places her thumb in her palm and grasps it with her fingers. A Phalen maneuver is negative. Which condition is most likely? A. B. C. D. E.

Carpal tunnel syndrome De Quervain’s tenosynovitis Gouty arthritis of the first metacarpophalangeal joint Palmar fasciitis Rheumatoid arthritis

ANSWERS IX-1. The answer is A. (Chap. 314) The innate immune system is phylogenetically the oldest form of immunologic defense system, inherited from invertebrates. This defense system uses germ line–encoded proteins to recognize pathogen-associated molecular patterns. Cells of the innate immune system include macrophages, dendritic cells, and natural killer lymphocytes. The critical components of the innate immune system include recognition by germ line–encoded host molecules, recognition of key microbe virulence factors but not recognition of self molecules, and nonrecognition of benign foreign molecules or microbes. Adaptive immunity is found only in vertebrate animals and is based on the generation of antigen receptors on T and B lymphocytes by gene rearrangements, such that individual T or B cells express unique antigen receptors on their surface capable of recognizing diverse environmental antigens. IX-2.

388

The answer is A. (Chap. 314) Complement activity, which results from the sequential interaction of a large number of plasma and cell membrane proteins, plays an important role in the inflammatory response. The classic pathway of complement activation is initiated by an antibody–antigen interaction. The first complement component (C1, a complex composed of three proteins) binds to immune complexes with activation mediated

The answer is C. (Chap. 315) The human major histocompatibility complex genes are located on a 4-megabase region on chromosome 6. The major function of the MHC complex genes is to produce proteins that are important in developing immunologic specificity through their role in binding antigen for presentation to T cells. This process is nonspecific, and the ability of an HLA molecule to bind to a particular protein depends on the molecular fit between the amino acid sequence of a particular protein and the corresponding domain on the MHC molecule. Once a peptide has bound, the MHC-peptide complex binds to the T-cell receptor, after which the T cell must determine if an immune response should be generated. If an antigen is similar to an endogenous protein, the potential antigen will be recognized as a self-peptide and tolerance to the antigen will be continued. The MHC I and II complexes have been implicated in the development of many autoimmune diseases, which occur when T cells fail to recognize a peptide as a self-peptide and an immune response is allowed to develop. MHC I and II genes also play a major role in tissue compatibility for transplantation and are important in generating immune-mediated rejection. The other answers listed refer to functions of immunoglobulins. The variable region of the immunoglobulin is a B cell–specific response to an antigen to promote neutralization of the antigen through agglutination and precipitation. The constant region of the immunoglobulin is able to nonspecifically activate the immune system through complement activation and promotion of phagocytosis by neutrophils and macrophages.

IX-4.

The answer is B. (Chap. 317) The patient has classic symptoms of angioedema with rapid onset of facial swelling often involving the lips, frequently with preceding limb symptoms. Angioedema and urticaria are grouped by the underlying etiology. In this case, ACE inhibitor use is associated with increased levels of bradykinin that in a predisposed individual can result in angioedema. Hereditary angioedema is associated with chronically depressed levels of C1 inhibitor that is involved in the degradation of bradykinin. IgEmediated angioedema occurs due to specific antigen sensitivity. Complement-mediated disease may be due to vasculitis, serum sickness, or reactions to blood products. Non­ immunologic causes of angioedema include direct mast cell–releasing agents such as opiates and agents that alter arachidonic acid metabolism, most commonly NSAIDs. IgE levels are not elevated in bradykinin-mediated angioedema. Because of the potentially life-threatening nature of the disease, rechallenge with a second ACE inhibitor is not recommended.

IX-5.

The answer is D. (Chap. 317) Urticaria and angioedema are common disorders, affecting approximately 20% of the population. In acute urticarial angioedema, attacks of swelling are of less than 6 weeks’ duration; chronic urticarial angioedema is by definition more long standing. Urticaria usually is pruritic and affects the trunk and proximal extremities. Angioedema is generally less pruritic and affects the hands, feet, genitalia, and face. This female has chronic urticaria, which probably is due to a cutaneous necrotizing vasculitis. The clues to the diagnosis are the arthralgias, the presence of residual skin discoloration, and the elevated sedimentation rate, which would be uncharacteristic of other urticarial diseases. The diagnosis can be confirmed by skin biopsy. Chronic urticaria rarely has an allergic cause; hence, allergy skin tests and measurement of total IgE levels are not

ANSWERS

IX-3.

SECTION IX

by C1q. Active C1 then initiates the cleavage and concomitant activation of components C4 and C2. The activated C1 is destroyed by a plasma protease inhibitor termed C1 esterase inhibitor. This molecule also regulates clotting factor XI and kallikrein. Patients with a deficiency of C1 esterase inhibitor may develop angioedema, sometimes leading to death by asphyxia. Attacks may be precipitated by stress or trauma. In addition to low antigenic or functional levels of C1 esterase inhibitor, patients with this autosomaldominant condition may have normal levels of C1 and C3 but low levels of C4 and C2. Danazol therapy produces a striking increase in the level of this important inhibitor and alleviates the symptoms in many patients. An acquired form of angioedema caused by a deficiency of C1 esterase inhibitor has been described in patients with autoimmune or malignant disease.

389

SECTION IX

helpful. Measurement of C1 esterase inhibitor activity is useful in diagnosing hereditary angioedema, a disease that is not associated with urticaria. Patch tests are used to diagnose contact dermatitis. The answer is D. (Chap. 317) This patient presents with symptoms of cold urticaria, an IgE-dependent urticarial reaction to cold exposure. After exposure to cold, urticarial lesions appear in exposed areas and usually last for less than 2 hours. Histologic examination of the urticarial lesion would demonstrate mast cell degranulation with edema of the dermis and subcutaneous tissues. In experimental exposure to a cold challenge such as an ice water bath, elevated levels of histamine in venous blood may be demonstrated if assessed in the extremity exposed to a cold environment, whereas the histamine levels would be normal in a nonexposed extremity. The appearance of a linear wheal after a firm stroke is indicative of dermatographism. This condition can be seen in 1–4% of the population and is often found in individuals with cold urticaria. In general, cold urticaria is a localized process without adverse consequences. However, vascular collapse may occur if an individual is submerged in cold water. Many individuals request treatment because they are embarrassed by their condition or are symptomatic from the recurrent urticaria and pruritus. Treatment with H1 histamine receptor blockers is usually adequate for symptom control. Cyproheptadine or hydroxyzine can be added to therapy if H1 antihistamines are inadequate. In this patient, there is a clear precipitant for developing urticaria—cold exposure. Thus, no other evaluation is necessary. In the evaluation and management of chronic urticaria, the identification and elimination of precipitating factors is important. Possible etiologic factors include foods, pollens, molds, and medications. In this case the urticaria predates the use of oral contraceptive medications; thus, stopping oral contraceptives is unlikely to be helpful. Assessment of antithyroglobulin and antimicrosomal antibodies can be helpful in individuals with chronic urticaria in whom a cause is not otherwise identified. Deficiency of C1 or the presence of a C1 inhibitor presents as recurrent angioedema rather than urticaria.

IX-7.

The answer is E. (Chap. 317) Allergic rhinitis is a common problem in the United States and North America. It is estimated that about 1 in 5 individuals experiences allergic rhinitis. The incidence is greatest in childhood and adolescence, and the symptoms tend to regress with aging. Complete remissions, however, are uncommon. Many individuals experience seasonal symptoms only. These symptoms are due to pollen production by weeds, grasses, and trees that are dependent on wind currents, rather than insects, for cross-pollination. The timing of the pollination events predicts the seasonal severity of symptoms and varies little from year to year within a particular locale. Based on this pattern, one is able to predict which allergens are most likely responsible for a patient’s symptoms. In the temperate regions of North America, trees pollinate in the spring, and ragweed pollinates in the fall. Grasses are responsible for seasonal allergic symptoms in the summer months. Mold allergens can have a variable pattern of symptoms, depending on climatic conditions that allow them to sporulate. Perennial rhinitis does not have a seasonal pattern and is more continually present. Allergens that cause perennial rhinitis include animal dander, dust, and cockroachderived proteins.

IX-8.

The answer is A. (Chap. 319) Antinuclear antibodies are nearly ubiquitous in patients with systemic lupus erythematosus, with demonstration in 90% of affected patients. There are many other antibodies that can be demonstrated. The next most common antibodies are anti-dsDNA and anti-histone. Anti-dsDNA is very specific to SLE and may correlate with disease activity, nephritis, and vasculitis. Antihistone is more frequent in druginduced SLE. Antiphospholipid antibodies can be demonstrated in about half of affected patients, while the remainder is present in less than half of SLE cases.

IX-9.

The answer is B. (Chap. 319) There are well-published, strict diagnostic criteria for systemic lupus erythematosus. They include four or more of the following criteria from Table IX-9.

Rheumatology and Immunology

IX-6.

390

TABLE IX-9  Diagnostic Criteria for Systemic Lupus Erythematosus

Photosensitivity Oral ulcers Arthritis

Immunologic disorder Antinuclear antibodies

ANSWERS

Serositis Renal disorder Neurologic disorder Hematologic disorder

Fixed erythema, flat or raised, over the malar eminences Erythematous circular raised patches with adherent keratotic scaling and   follicular plugging; atrophic scarring may occur Exposure to ultraviolet light causes rash Includes oral and nasopharyngeal ulcers, observed by physician Nonerosive arthritis of two or more peripheral joints, with tenderness,   swelling, or effusion Pleuritis or pericarditis documented by ECG or rub, or evidence of effusion Proteinuria >0.5 g/d or ≥3+ by dipstick or cellular casts Seizures or psychosis without other causes Hemolytic anemia or leukopenia (